Download as pdf or txt
Download as pdf or txt
You are on page 1of 717

Directions Q.

1-5): Study the following information carefully and answer the questions given below:
Twelve persons N, P, Q, R, S, T, U, V, W, X, Y and Z are sitting in two parallel rows. Row 1 is facing west and row 2
is facing east. P, Q, R, S, T and U are facing west. Each person of row 1 is facing the other person of row 2.
V sits at one of the ends of a row and second to the left of Y. There are two persons between Z and N. There is only
one person between T and S. U sits third to the right of Q and is not opposite either V or Y. S and T are not opposite
V. P and S are adjacent to each other. N is not adjacent to W, who is not opposite T.
1). Who among the following is opposite Q?
a) Y
b) Z
c) W
d) N
e) None of these
2). How many persons are there between S and R?
a) One
b) Two
c) Three
d) None
e) None of these
3). Which of the following statements is/are true about P?
a) S does not sit adjacent to P.
b) Z sits opposite the person who sits third to the left of P.

www.bankingpdf.com
c) Y sits opposite P.
d) N does not sit opposite the person who sits on the immediate right of P.
e) None of these
4). Who among the following sit at the extreme ends of row 2?
a) R,V
b) V, X
c) T, U
d) N, Z
e) None of these
5). Who among the following sits second to the right of N?
a) W
b) V
c) Z
d) X
e) None of these

Directions (Q. 06-10): Study the following information carefully and answer the questions given below:
S, T, U, V, W, X, Y and Z are eight family members. Four of them are females and four are males. There are three
couples, two daughters and two sons in the family. They all have a different profession, viz Engineer, Teacher,
Lawyer, Judge, Clerk, Doctor, CS and CA. All are sitting in a circle facing the centre. Z is father of X and U. The CS

www.bankingpdf.com
is married to the Judge. Neither Z nor Y is a Judge. V is the Teacher and is sitting between the Lawyer and the CS.
No male is a Teacher. T is father of Y and is sitting between two males. W is daughter of X, who is sitting second to
the right of her son. S is the Clerk and is married to the Doctor, who sits opposite her. W is the CA and is sitting
second to the left of her father. Z and Y are not adjacent to each other. All females sit together.
6). Which of the following pairs is not correctly matched?
a) T — Judge
b) Z—CS
c) Y— Lawyer
d) V -- Teacher
e) None of these
7). Who among the following sits second to the left of Z?
a) Y
b) S
c) T
d) W
e) None of these
8). Who among the following is a Lawyer?
a) Z
b) Y
c) U
d) T

www.bankingpdf.com
e) None of these
9). Who among the following is an Engineer?
a) X
b) Z
c) W
d) T
e) None of these
10). Who among the following is the wife of U?
a) V
b) W
c) S
d) X

e) None of these

Explanation:
Direction (01 to 05):

www.bankingpdf.com
Answers: 1)b 2)b 3)c 4)b 5)a

Direction (06 to 10):

www.bankingpdf.com
Answers: 6)b 7)b 8)b 9)b 10)c

10-20 Questions :

Directions (Q. 1-5): Read the following information carefully and answer the questions given below.
Shatrujit, Shaukat, Shaunak, Shaurav, Sheetal, Shekhar, Shikha and Shikhar are eight persons. Each is a doctor of a
different specialization like Nephrology, Neurology, Neurosurgery, Ophthalmology, Orthodontics, Orthopaedics, Radiology
and Urology. Each of them has different cars like Maruti Suzuki, Hyundai, Fiat, Toyota, Honda, Chevrolet, Skoda and
Mercedes-benz but not necessarily in the same order. Shatrujit who likes Skoda is a doctor of neither Urology nor
Orthodontics. Shaukat is a doctor of Ophthalmology and does not like either Maruti Suzuki or Fiat. The one who is a
doctor of Nephrology likes Toyota. The one who likes Mercedes-benz is a doctor of Neurology and the one who is a
doctor of Radiology likes Hyundai. Shikhar doesn’t like Honda. Sheetal and Shekhar like Hyundai and Chevrolet, though
not necessarily in the same order. Shaurav likes Mercedes-benz. Shikha is a doctor of Orthopaedics and doesn’t like
Maruti Suzuki. The one who is a doctor of Orthodontics likes Chevrolet. Shaunak is not a doctor of Urology. The one who
is a doctor of Neurosurgery doesn’t like either Fiat or Honda.

www.bankingpdf.com
1). Which of the following combinations is definitely correct?
a) Shaunak-Nephrology-Hyundai
b) Shekhar-Radiology-Hyundai
c) Shaurav-Orthopaedics-Maruti Suzuki
d) Shaukat-Ophthalmology-Honda
e) None of these

2). Who among of the following likes Maruti Suzuki?


a) Shatrujit
b) Shaukat
c) Shaurav
d) Shikhar
e) None of these

3). Shaunak is a doctor of which of the following specializations?


a) Nephrology
b) Orthodontics
c) Orthopaedics
d) Neurosurgery

www.bankingpdf.com
e) None of these

4). Which of the following combinations is incorrect?


a) Shaurav-Mercedes-benz
b) Shatrujit-Neurosurgery
c) Shikha-Chevrolet
d) Shaunak-Toyota
e) None of these

5). Who among of the following is a doctor of Urology?


a) Sheetal
b) Shekhar
c) Shaukat
d) Shikhar
e) None of these

Directions (Q. Nos. 6-10) Study the following statements and conclusion carefully and answer the syllogism questions
given below it

6). Statements: All furniture is jungles

www.bankingpdf.com
No jungle is road
Some roads are hills
Conclusions:
(i) Some roads are furniture
(ii) Some jungles are furniture
(iii) Some hills are jungles
a) None follows
b) Only i follow
c) Only ii follows
d) Only iii follows
e) Only i and ii follows.

7). Statements: All the locks are keys.


All the keys are bats.
Some watches are bats.
Conclusions:
(i) Some bats are locks.
(ii) Some watches are keys.
(iii) All the keys are locks.
a) None follows
b) Only i follow
c) Only ii follows

www.bankingpdf.com
d) Only iii follows
e) Only i and ii follows.

8). Statements: Some keys are staplers.


Some staplers are stickers.
All the stickers are pens.
Conclusions:
(i) Some pens are staplers.
(ii) Some stickers are keys.
(iii) No sticker is keys.
(iv) Some staplers are keys.
a) All follows
b) Only either i or ii and both iii and iv follows
c) Only either ii or iii and both i and iv follows
d) Only either i or iv follows
e) None of these

9). Statements: Some questions are answers.


Some answers are writers.
All the writers are poets.
Conclusions:
(i) Some writers are answers.

www.bankingpdf.com
(ii) Some poets are questions.
(iii) All the questions are poets.
(iv) Some poets are answers.
a) Only i and ii follows
b) Only i and iii follows
c) Only iii and iv follows
d) Only i and iv follows
e) None of these

10). Statements: Some envelops are gums.


Some gums are seals.
Some seals are adhesives.
Conclusions:
(i) Some envelopes are seals.
(ii) Some gums are adhesives.
(iii) Some adhesives are seals.
(iv) Some adhesives are gums.
a) None follows
b) Only i follows
c) Only iv follows
d) Only iii follows

www.bankingpdf.com
e) Only ii follows
Answers:
1)d 2)d 3)a 4)c 5)d 6)c 7)b 8)c 9)d 10)d

Explanation:
Directions (Q. 1-5):

1).
Answer: d)
2).
Answer: d)
3).

www.bankingpdf.com
Answer: a)
4).
Answer: c)
5).
Answer: d)

6).

Answer: c)
7).

www.bankingpdf.com
Answer: b)
8).

Answer: c)
9).

www.bankingpdf.com
Answer: d)
10).

Answer: d)

www.bankingpdf.com
20-30 Questions :

Directions (Q. 1-5): P,Q,R,S,T,U,V and W are eight members of a family. They belong to three different generations.
There are three married couples. All of them are sitting around a circular table, facing the centre but not necessarily in the
same order.
 P and T are a married couple. T the mother of R and sits second to the left of her husband.
 R is the brother of W, who is not an immediate neighbour of his mother.
 There is only one person sitting between R and his daughter S, but that person is not S’s Uncle.
 U, the nephew of W, sits third to the right of Q's husband, but neither to the opposite nor to the immediate left of his
father.
 S is not an immediate neighbour of her mother V. No three females are sitting together.
 V and his sister-in-law are immediate neighbour.

1). Who among the following is S’s father?


a) P
b) R
c) W
d) U
e) Can’t be determined

www.bankingpdf.com
2). How many females are there in the family?
a) Four
b) Three
c) Five
d) Can’t be determined
e) None of these

3). What is the position of T with respect to her granddaughter?


a) Second to the left
b) Fourth to the right
c) Third to the right
d) Fourth to the left
e) Immediate left

4). Who among the following is the aunt of U?


a) T
b) V
c) W
d) Q
e) Can’t be determined

www.bankingpdf.com
5). Which of the following represents a group of Female members of the family?
a) SP
b) RU
c) WV
d) TQ
e) Can’t be determined

Directions (Q. 6-10): A seven story building is rented for exhibition by 7 different mobile companies namely Apple, Lenovo,
Micromax, Samsung, HTC, Sony, and BlackBerry but not in the same order. Ground floor is numbered 1, first floor is
numbered. 2 and so on until the topmost floor is numbered 7 .
I. Only two companies can rent the floor which is above Lenovo company.
II. There are two floors between BlackBerry and HTC also BlackBerry has rented the floor which is above Lenovo company.
III. Sony has rented the floor which is immediately above Micromax company , Who has rented an odd number floor.
IV. The number of floors rented above and below Samsung were same.

6). Which of the following company has rented the 3rd Floor?
a) BlackBerry
b) Lenovo
c) HTC

www.bankingpdf.com
d) Samsung
e) Sony

7). Which company has rented the floor immediately below Lenovo?
a) Sony
b) Lenovo
c) BlackBerry
d) Samsung
e) Micromax

8). One of the company from the below option which does not belong to Group?
a) Sony
b) Lenovo
c) Apple
d) HTC
e) Micromax

9). What is the number of floor which apple Company has rented?
a) 2nd floor
b) 7th Floor

www.bankingpdf.com
c) 3rd Floor
d) 5th Floor
e) None of these

10). Which of the following statement is true according to BlackBerry Company?


a) There are two floors above BlackBerry.
b) There are only one floor between Blackberry and Sony.
c) Lenovo and HTC were the immediate neighbours .
d) The 6th Floor is rented by BlackBerry.
e) None of these.

Answers:
1)b 2)a 3)c 4)d 5)d 6)c 7)d 8)a 9)b 10)d

Solution:
Directions (Q. 1-5):

www.bankingpdf.com
Directions (Q. 6-10):

www.bankingpdf.com
30-40 Questions :

Direction (Q. 1-5): Study the given information and answer the following questions:
When a word and number arrangement machine is given an input line of words and numbers, it arranges them following a
particular rule. The following is an illustration of an input and rearrangement:

Input: 45 native charge 33 48 dark total freeze 62 88 98 gold office 21


Step I: 45 native charge 33 48 dark freeze 62 88 98 gold office total 21
Step II: 45 native charge 48 dark freeze 62 88 98 gold total 21 office 33
Step III: charge 48 dark freeze 62 88 98 gold total 21 office 33 native 45
Step IV: charge dark freeze 62 88 98 total 21 office 33 native 45 gold 48
Step V: charge dark 88 98 total 21 office 33 native 45 gold 48 freeze 62
Step VI: charge 98 total 21 office 33 native 45 gold 48 freeze 62 dark 88

www.bankingpdf.com
Step VII: total 21 office 33 native 45 gold 48 freeze 62 dark 88 charge 98
Step VII is the last step of the above arrangement as the intended arrangement is obtained. As per the rules followed in
the given steps, find out the appropriate steps for the given input.
Input: 35 quite head clear 50 65 98 slow giant 71 82 19 oliver music

1). Which element is fourth to the right of ‘giant’ in Step V?


a) clear
b) 19
c) quite
d) 98
e) 35

2). How many steps will be required to complete the given input?
a) Five
b) Six
c) Seven
d) Eight
e) None of these

3). Which of the following is Step III of the given input?


a) quite 82 head 50 giant 35 clear 19 65 98 slow 71 oliver music

www.bankingpdf.com
b) head clear 65 98 giant 71 82 music slow 19 quite 35 oliver 50
c) head clear 98 giant 71 82 slow 19 quite 35 oliver 50 music 65
d) head clear 98 65 giant 71 82 music slow 19 quite 35 oliver 50
e) None of these

4). Which of the following is the fourth element from the left end of Step VI?
a) clear
b) music
c) 71
d) oliver
e) None of these

5). What is the position of ‘music’ from the left end in the final step?
a) Sixth
b) Seventh
c) Eighth
d) Fifth
e) Ninth

Directions (Q. 6-10): Read the information/ statement given in each question carefully and answer the questions.

www.bankingpdf.com
6). In which of the following expressions does the expression ‘P > S’ hold true?
a) P = Q < R ≤ S
b) S ≥ Q > R > P
c) Q = S > R ≥ P
d) P ≥ R >Q = S
e) S ≤ Q > P > R

7). Which of the following expressions will be true if the expression ‘E ≥ C < O = S’ is definitely true?
a) S < C
b) E ≥ O
c) E < S
d) O = E
e) None is true

8). In which of the following expressions will the expression ‘A < B’ be definitely true?
a) T < A ≥ Y = B
b) A > T ≥ Y = B
c) B = Y ≥ T > A
d) A ≥ T > Y < B
e) None of these

www.bankingpdf.com
9). Which of the following expressions may not be true if the expression ‘H ≥ R = U ≤ G’ is definitely true?
a) U ≤ H
b) G ≥ H
c) R ≤ G
d) Only (b) and (c)
e) All are true

10). Which of the following expressions will be true if the expression ‘C ≥ F > V ≥ W’ is definitely true?
a) W ≤ C
b) C = V
c) C < W
d) F ≥ W
e) None is true

Answers:
1).c) 2).c) 3).b) 4).e) 5).b) 6).d) 7).e) 8).c) 9).b) 10).e)

40-50 Questions :

www.bankingpdf.com
Directions (Q. 1-5):
Words are arranged in reverse alphabetical order and the numbers are arranged in ascending order. One word and one
number is arranged in each step. The arrangement is done from right to left.
Input: 35 quite head clear 50 65 98 slow giant 71 82 19 oliver music
Step I. 35 quite head clear 50 65 98 giant 71 82 oliver music slow 19
Step II. head clear 50 65 98 giant 71 82 oliver music slow 19 quite 35
Step III. head clear 65 98 giant 71 82 music slow 19 quite 35 oliver 50
Step IV. head clear 98 giant 71 82 slow 19 quite 35 oliver 50 music 65
Step V. clear 98 giant 82 slow 19 quite 35 oliver 50 music 65 head 71
Step VI. clear 98 slow 19 quite 35 oliver 50 music 65 head 71 giant 82
Step VII. slow 19 quite 35 oliver 50 music 65 head 71 giant 82 clear 98.
1).
Answer: c)
2).
Answer: c)
3).
Answer: b)
4).
Answer: e)
5).
Answer: b)

www.bankingpdf.com
6). In the expression P ≥ R>Q= S, expression P > S hold true.
Answer: d)

7). None of the given expression is definitely true from the expression E≥ C <O = S
Answer: e)

8). In the expression B = Y ≥ T > A, expression A < B is definitely true


Answer: c)

9). Expression X≥ H is definitely false within the expression H≥ R = U≤ G


Answer: b)

10). None of the given expression is definitely true from the expression, C≥ F>V≥ W.

Answer: e)

50-60 Questions

www.bankingpdf.com
Directions (Q. 1-5): Read the following information and answer the questions given below it:
(i) L, M, N, O, P, Q, R and S are sitting around a circle facing the centre.
(ii) 'N', who is third to the left of 'P', is not a neighbour of 'R' and 'M'.
(iii) S is the neighbour of 'O' and 'R' and is third to the right of 'M’.
(iv) ‘L' is not the neighbour of ‘O', who is second to the left of 'N'.

1). What is the position of ‘Q'?


a) Immediate right of R
b) Immediate left of N
c) Third to the right of M
d) Second to the left of S
e) None of these
2). Which of the following is the pair of adjacent persons?
a) R and M
b) M and N
c) R and O
d) L and P
e) None of these
3). Which of the following pairs has the first person sitting second to the left of the second person?
a) N,M
b) Q, S

www.bankingpdf.com
c) L,Q
d) S, P
e) None of these
4). If N and O interchange their positions, then which of the following will be the correct statement?
a) S is on the immediate right of N.
b) R is second to the right of O.
c) O is between L and Q.
d) N is between L and Q.
e) None of these
5). Which of the following has the second person sitting between the remaining two?
a) M,L,P
b) N, L,Q
c) P,S,R
d) Q, N, O
e) None of these

Directions (Q. 6-10): Study the following information carefully and answer the questions given below:
A, B, C, D, E, F and G arc sitting in a straight line facing north, but not necessarily in the same order. There is only one
person between F and C. E sits between A and D. There are only two persons between E and G. F sits on the immediate
left of A, who sits in the middle of the row.
6). How many persons are there between E and F?

www.bankingpdf.com
a) One
b) Two
c) Three
d) Can't be determined
e) None of these
7). Who among the following sit at the extreme ends of the row?
a) D,F
b) G,C
c) B, C
d) Can't be determined
e) None of these
8). Who among the following sits on the immediate right of D?
a) G
b) E
c) F
d) B
e) None of these
9). Who among the following sits third to the right of A?
a) C
b) G
c) B
d) E

www.bankingpdf.com
e) None of these
10). Which of the following statements is true with regard to B?
a) B is second to the right of A.
b) B is fourth to the left of G
c) B sits at the extreme right end of the row.
d) B sits at the extreme left end of the row.
e) None of these

Answers:

1).b) 2).e) 3).a) 4).c) 5).e) 6).a) 7).c) 8).d) 9).c) 10).c)

Explanation:

Directions (Q. 1-5):

www.bankingpdf.com
Directions (Q. 6-10):

60-70 Questions

1). Statement: B > D < T ≥ V = M ≥ X > Z


Conclusions:
I. T > X

www.bankingpdf.com
II. T = X
III. T ≥ Z
IV. M > T
a) None follows
b) Only III follows
c) Only IV follows
d) Only either I or II follows
e) None of these

2). Statement: Q > H = U ≥ C = J < E


Conclusions:
I. Q > J
II. E > H
III. H ≥ J
IV. U > E
a) Only I and III follow
b) Only I and IV follow
c) Only II and III follow
d) Only II, III and IV follow
e) None of these

www.bankingpdf.com
3). Statement: W > H = I ≥ C = L < E
Conclusions:
I. E < W
II. I ≥ L
III. E > I
IV. W > L follows
a) Only I and IV follow
b) Only II and IV follow
c) Only I and III follow
d) Only I, II and IV follows
e) None of these

4). Statement: P = R ≥ E < S = N ≥ T


Conclusions:
I. P > T
II. N > R
III. S > P
IV. T > R
a) None follows
b) Only III follows
c) Only IV follows

www.bankingpdf.com
d) Only either III or IV follows
e) None of these

5). Statements: A > F ≥ G, D < H = G


Conclusions:
I. A ≥ H
II. F ≥ D
III. D ≥ G
IV. A = D
a) None follows
b) Only I follows
c) Only II follows
d) Only III follows
e) Only IV follows

6). Statements: C > B ≥ L, Q = E > P = C


Conclusions:
I. Q > B
II. L < E
III. Q > L
IV. P > L

www.bankingpdf.com
a) Only I, II and III follows
b) Only II, III and IV follow
c) Only I, III and IV follows
d) All I, II, III and IV follows
e) None of these

7). Statements: O ≤ U < L, P ≥ I < C = L


Conclusions:
I. U > I
II. C > O
III. L > I
IV. P > O
a) Only I and II follows
b) Only I, II and III follows
c) Only II and III follows
d) Only II, III and IV follows
e) None of these

8). Statement: D ≤ H = J ≤ K ≥ P > R


Conclusions:
I. D = K

www.bankingpdf.com
II. K > D
III. K > R
IV. J > R
a) None follows
b) Only III follows
c) Only IV follows
d) Only either I or II and III follows
e) None of these

9). Statement: A ≥ R > S = Y < W < V


Conclusions:
I. A > Y
II. W > R
III. V > S
IV. Y ≤ A
a) Only I and III follows
b) Only I and IV follows
c) Only II and III follows
d) Only I, II and IV follows
e) None of these

www.bankingpdf.com
10). Statement: R ≥ U > F = E ≥ X > Z
Conclusions:
I. R > E
II. U > X
III. F > Z
IV. R > Z
a) Only I, II and III follows
b) Only II, III and IV follows
c) Only I, III and IV follows
d) All I, II, III and IV follows
e) None of these

Answer:
1)d 2)a 3)b 4)a 5)a 6)d 7)c 8)d 9)a 10)d

Explanations:
1). B > D < T ≥ V = M ≥ X > Z
T>X
T=X
T≥Z
M>T

www.bankingpdf.com
Either I or II follows.
Answer: d)

2). Q > H = U ≥ C = J < E


Q > J follows
E > H not follows
H ≥ J follows
U > E not follows
Answer: a)

3). W > H = I ≥ C = L < E


E < W not follows
I ≥ L follows
E > I not follows
W > L follows
Answer: b)

4). P = R ≥ E < S = N ≥ T
P>T
N>R

www.bankingpdf.com
S>P
T>R
None follows
Answer: a)

5). A > F ≥ G, D < H = G


A≥H
F≥D
D≥G
A=D
None follows
Answer: a)

6). C > B ≥ L, Q = E > P = C


Q>B
L<E
Q>L
P>L
All follows
Answer: d)

www.bankingpdf.com
7). O ≤ U < L, P ≥ I < C = L
U > I not follows
C > O follows
L > I follows
P > O not follows
Answer: c)

8). D ≤ H = J ≤ K ≥ P > R
D=K
K>D
K > R follows
J > R not follows
Either I or II and III follows
Answer: d)

9). A ≥ R > S = Y < W < V


A > Y follows
W > R not follows
V > S follows

www.bankingpdf.com
Y ≤ A not follows
Answer: a)

10). R ≥ U > F = E ≥ X > Z


R>E
U>X
F>Z
R>Z
All follows

Answer: d)

70-80 Questions

Directions (Q. 89-95); These questions are based on the following arrangement of numbers, symbols and letters.
F$=2GP☸L9Q£@7RTU?©68BEβ3A3#4JKM
1). ‘$2FP' is to ‘☸9P£’ in the same way as ‘____’ is to '8E6A'.
a) 1)T?R6
b) 2) U?R6

www.bankingpdf.com
c) 3)T©R6
d) 4)T?76
e) 5) None of these
2). How many such numbers are there in the above arrangement each of which is either immediately followed by a
symbol or immediately preceded by a letter, but not both?
a) Three
b) None
c) One
d) 4)Two
e) None of these
3). How many such symbols are there in the above arrangement each of which is either immediately followed by a
letter or immediately preceded by a number, but not both?
a) One
b) Three
c) Two
d) Four
e) More than four
4). If all the symbols in the arrangement are removed, then which of the following will be the seventh to the right of
the 14th element from the right end?
a) 1)P
b) 2)E

www.bankingpdf.com
c) 3)A
d) 4)3
e) 5) None of these
5). If all the numbers in the arrangement are removed, then which of the following will be exactly midway between the
fourth element from the right end and the fourth element from the left end?
a) 1)R
b) 2) U
c) 3) ?
d) 4) ©
e) 5) None of these
6). How would the word ‘PIMENTO' be written if each of its consonants is substituted by a letter third to its right in the
English alphabet and each of its vowels is substituted by the fourth consonant to its right in the English alphabet?
a) I ) SMPIQWR
b) 2) SMPJQWR
c) 3) SMPIQWS
d) 4) SMPJQWS
e) 5) None of these
7). In the case of how many letters in the word 'UNSOCIABLE', their serial order in the word will not differ from their
serial order in the arrangement where the letters of the word are arranged alphabetically?
a) Two
b) 2) None

www.bankingpdf.com
c) 3) One
d) 4) Three
e) 5) Four
Directions (8-10): Study the following informatI011 carefully and answer the questions given below.
In a certain code language, 'spring is the best season' is written as 'ni ra tin su da', 'nature blossoms in spring' is
written as 'ye zo ga tin', 'this is nature' is written as 'zo ra pa'. 'love blossoms this season' is written as 'ga mo su pa',
'best month' is written as 'ki da'.
8). What is the code for 'the'?
a) 1)ni
b) 2)ra
c) 3) tin
d) 4) su
e) 5) da
9). Which of the following does 'pa' stands for?
a) blossoms
b) 2) love
c) 3) season
d) 4) this
e) 5) Cannot be determined
10). Which of the following is the code for 'nature the best'?
a) zo pa da

www.bankingpdf.com
b) 2) ni da zo
c) 3) ra ino da
d) 4) da ga ra
e) 5) None of these

Answer:
1)a 2)c 3)b 4)b 5)e 6)d 7)b 8)a 9)d 10)b

Solutions
1). $, 2, F and P are nth, (n +2)th, (n-1)th and (n +4)th elements respectively from the left end of the given sequence.
☸, 9, P, £ are mth, (m + 2)th, (m -1)th and (m+4)th elements respectively from the left end of the given sequence.
Similarly,
8, E, 6 and A are xth, (x + 2)th, (x - 1)th and (x +4)th elements respectively from the left end of the given sequence.
The above relationship is found only in the option a),ieT?R6.
Answer: a)
2). Look at the underlined number. F$=2 G P ☸ L 9 Q £ @ 7 R T U ? © 6 8 B E β A 3 # 4 J K M
Answer: c)
3). 2; Look at the underlined symbols.
F$= 2 G P ☸ L 9 Q £ @ 7 R T U ? © 6 8 B E βA 3 # 4 J K M There are only three such symbols.
Answer: b)

www.bankingpdf.com
4). When all the symbols in the given arrangement are removed then the new sequence of the elements remaining
will be as follows:
F2GPL9Q7RTU68BEA34JKM
Now, we have to find out (14 - 7 =) 7th element from the right end.
Answer: b)
5). When all the numbers in the given arrangement are removed then the new sequence of the elements remaining
will be as follows:
F$ = G P ☸ L Q £ @ R T U ? © B E β A # J K M Now, we have to fmd out the element which is exactly midway
between G and # in the above sequence. Hence, the required element is 'T'.
Answer: e)
6).
P I M E N T O
+3 +4 +3 +4 +3 +3 +4
S M P J Q W S
Note: The fourth consonant after E is J.
Answer: d)
7).
U N S O C I A B L E
A B C E I L N O SU
Answer: b)
Directions(8-10):

www.bankingpdf.com
spring is the best season  ni ra tin su da ..… (1)
nature blossoms in spring  ye zo ga tin ….. (2)
this is nature  zo ra pa …… (3)
love blossoms this season  ga mo su pa ..... (4)
best month ki da --(5)
From (1) and (2), spring  tin
From (1) and (3), is ra
From (1) and (4), season  su
From (1) and (5), best  da
From (1), the  ni
From (2) and (3), nature  zo
From (3), this  pa
From (2) and (4), blossoms  ga
From (4), love  mo
From (5), month  ki
From (2), in  ye
8).
Answer: a)
9).
Answer: d)
10).

www.bankingpdf.com
Answer: b)
Nature the best
zo ni da

80-90 Questions

1). Statements: Some sticks are lamps


Some flowers are lamps
Some lamps are dresses
All dresses are shirts
Conclusions:
(i) Some shirts are sticks
(ii) Some shirts are flowers
(iii) Some flowers are sticks
(iv) Some dresses are sticks
a) Only i and iii follows
b) Either ii or iv follows
c) None follows
d) Only iv follows
e) None of these

www.bankingpdf.com
2). Statements: Some buses are cars
All cars are roads
All roads are codes
Some codes are dresses
Conclusions:
(i) Some dresses are cars
(ii) Some codes are buses
(iii) Some roads are buses
(iv) Some dresses are buses
a) None follows
b) Only i follows
c) Only i and ii follows
d) Only ii and iii follows
e) All follows

3). Statements: All fans are halls


No halls is green
Some rooms are green
Conclusions:
(i) Some rooms are fans

www.bankingpdf.com
(ii) Some rooms are halls
(iii) Some fans are green
(iv) No green is fan
a) None follows
b) All follow
c) Only iii and iv follow
d) Only iv follow
e) None of these

4). Statements: Some colours are kites


Some kites are desks
All desks are notes
All notes are mountains
Conclusions:
(i) Some mountains are colours
(ii) Some notes are colours
(iii) Some mountains are desks
(iv) Some notes are kites
a) All follows
b) Only i and ii follows
c) Only iii and iv follows

www.bankingpdf.com
d) d) Only i and iv follows
e) None follows

5). Statements: Some benches are horses


All horses are houses
Some houses are lions
All lions are roads
Conclusions:
(i) Some roads are benches
(ii) Some lions are horses
(iii) Some houses are benches
(iv) Some roads are houses
a) Only i and iii follows
b) Only i and ii follows
c) Only iii and iv follows
d) Only ii and iii follows
e) None of these

6). Statements: All papers are boxes


Some boxes are boards
Some boards are blacks

www.bankingpdf.com
All blacks are roads
Conclusions:
(i) Some roads are boards
(ii) Some blacks are boxes
(iii) Some boards are papers
(iv) Some roads are boxes
a) None follows
b) Only i follows
c) Only ii follows
d) Only iii follows
e) Only iv follows

7). Statements: Some boxes are walls


Some walls are horses
All horses are jungles
Conclusions
(i) Some jungles are boxes
(ii) Some jungles are walls
(iii) Some horses are boxes
(iv) No jungles is box
a) Only i and ii follows

www.bankingpdf.com
b) Only i, ii and iii follows
c) Only either i or iv and ii follows
d) Only either i or iv and iii follows
e) None of these

8). Statements: All pens are walls


Some walls are tyres
Some tyres are rounds
Some rounds are buses
Conclusions:
(i) Some buses are tyres
(ii) Some rounds are walls
(iii) Some rounds are pens
(iv) Some buses are walls
a) None follows
b) All follows
c) Only i and ii follows
d) Only ii and iv follows
e) None of these

9). Statements: All flowers are buses

www.bankingpdf.com
Some buses are cats
All cats are tigers
Conclusions:
(i) Some tigers are buses
(ii) Some tigers are flowers
(iii) Some cats are flowers
(iv) Some buses are flowers
a) All follow
b) Only i and iii follows
c) Only ii and iv follows
d) Only i and iv follows
e) None of these

10). Statements: All stones are Sheets


No sheets is ring
Some rings are woods
All woods are windows
Conclusions:
(i) Some windows are stones
(ii) Some windows are rings
(iii) No window is stone

www.bankingpdf.com
(iv) Some rings are stones
a) Only i , iii and iv follows
b) Only ii and iii follows
c) Only either i or ii follows
d) Only either i or iii and ii follows
e) None of these.

Explanation:
1).

Answer: c)
2).

www.bankingpdf.com
Answer: d)
3).

Answer: d)
4).

Answer: c
5).

www.bankingpdf.com
Answer: c)
6).

Answer: b)
7).

Answer: c)
8).

www.bankingpdf.com
Answer: a)
9).

Answer: d)
10).

Answer: d)

www.bankingpdf.com
90-100 Questions

Directions (Q. 1-5): Study the following information carefully and answer the questions given below.
In a certain code language. 'India provide financial help' is written as 'pa Ii ka sa'. 'help for other country' is written as ‘sa ri
jo sh’, 'provide for poor country' is written its ‘jo ri ha pa' and 'Pakistan needs financial help' is written as ‘ta sa si li’.

1). What is the code for 'India help Pakistan' in the given code language?
a) ka sa ta
b) li pa ta
c) sa jo ka
d) Cannot be determined
e) None of these

2). What does ' Ii' stand for?


a) Pakistan
b) financial
c) help
d) poor

www.bankingpdf.com
e) None of these

3). What is the code for 'help' in the given code language?
a) si
b) ri
c) sa
d) Can't be determined
e) None of these

4). What may be the possible code for 'financial needs' in the given code language?
a) sa si
b) li si
c) ka ta
d) Can't be determined
e) None of these

5). What is the code for 'country' in the given code language?
a) pa
b) sh
c) jo

www.bankingpdf.com
d) cannot be determined
e) None of these

6). In a row of girls, Divi and Pavi occupy the ninth place from the right end and tenth place from the left end, respectively.
If they interchange their places, then Divi and Pavi occupy seventeenth place from the right and eighteenth place from the
left respectively. How many girls are there in the row?
a) 22
b) 24
c) 26
d) 28
e) Cannot be determined

7). Students line up in a queue in which Ashwin stands fifteenth from the left and Saran is seventh from the right. If they
interchange their places, Saran would be fifteenth from the right. How many students are there in the queue?
a) 21
b) 22
c) 28
d) 29
e) 30

www.bankingpdf.com
8). Ruban ranks eighteenth in a class of 49 students. What is his rank from the last?
a) 18
b) 19
c) 31
d) 32
e) 17

9). N ranks fifth in a class. S is eighth from the last. If T is sixth after N and just in the middle of N and S, then how many
students are there in the class?
a) 21
b) 23
c) 24
d) 25
e) 20

10). Vinoth is sixth from the left end and Penchal is tenth from the right end in a row of boys. If there are eight boys
between Vinoth and Penchala, how many boys are there in the row?
a) 21
b) 23
c) 24
d) 25

www.bankingpdf.com
e) 26

Answers:
1).d) 2).b) 3).c) 4).b) 5).d) 6).c) 7).d) 8).d) 9).c) 10).c)

Solution:
Directions (Q. 1-5):
India provide financial help pa li ka sa (i)
help for other country sa ri jo sh (ii)
provide for poor country jo ri ha pa ... (iii)
Pakistan needs financial help to sa si li ... (iv)
From (i), (ii) and (iv), help - > sa (v)
From (i) and (iii), provide pa ... (vi)
From (i), (iv) and (v) financial li (vii)
From (i), (v), (vi) and (vii), India ka... (viii)
From (ii) and (iii), for/country jo/ri ... (ix)
From (ii), (v) and (ix) other sh (x)
From (iii), (vi) and (ix) poor ha ... (xi)
From (iv), (v) and (vii) Pakistan/needs ta/si …(x ii)

www.bankingpdf.com
6). c) Since Divi and Pavi exchange places, so Pavi’s new position is the same as Divi’s earlier position. This position is
17th from the right and 10th from the left. Therefore Number of girls in the row = (16 + 1 + 9) = 26.
7). d). Saran’s new position is 15th from the right as well as the left end of the row.
Therefore number of students in the queue = ( 14 + 1 + 14 ) = 29.
8). d). Number of students behind Ruban in rank = ( 49 - 18 ) = 31.
So, Ruban is 32nd from the last.
9). c) Number of students in the class = ( 4 + 1 + 5 + 1 + 5 + 1 + 7 ) = 24.
10). c). Clearly, number of boys in the row = ( 6 + 10 + 8) = 24.

100-110 Questions :

Directions (Q. 01-05): Study the following information to answer the given questions.
Seven friends, viz Manjit, Navin, Parul, Hitesh, Tarun, Raghu and Sahil, are pursuing their PhD in different subjects—
Maths, History, Political Science, Chemistry, Economic, Accounts and Geography, but not necessarily in the same
order. They study in three different universities, viz P, Q and S. At least two friends study in the same university.
Hitesh is studying Political Science in university P. The one who is studying History is not from university S. Sahil is
studying Geography in university Q with only Navin. Manjit is not studying Economics and is not from university P.
Raghu is studying Accounts and is not from university P. Tarun is studying Chemistry but not from university P. No
one studies Maths or Economics in university P.

1). Which of the following represents the group that studies in university S?

www.bankingpdf.com
a) Hitesh, Navin
b) Parul, Raghu, Tarun
c) 3 ) Manjit, Tarun
d) Tarun, Raghu, Manjit
e) None of these
2). Parul is studying in which of the following universities?
a) P
b) Q
c) S
d) Can't be determined
e) None of these
3). Which of the following combinations is true?
a) Manjit—Economics--university S
b) Navin—History—university Q
c) Parul—History—university S
d) Tarun—Chemistry—university Q
e) None of these
4). Navin studies which subject in PhD?
a) History
b) Maths
c) Economics

www.bankingpdf.com
d) Can't be determined
e) None of these
5). Who among the following is studying History in PhD?
a) Navin
b) Parul
c) Raghu
d) Manjit
e) None of these

Directions (Q. 06-10): Study the following information to answer the given questions.
Six persons L, M, N, O, P and R are sitting around a circular table facing towards the centre in a hotel. They ordered
different items like Juice, Shake, Ice cream, Fruits, Patties and Pastries for their breakfast, not necessarily in the
same order. Each of them wears a shirt of a different brand, viz Levis, Woodland, Lee, Mufti, Raymond and UCB, not
necessarily according to the order given.
i.) The persons who ordered Juice, Ice cream, and Pastries are wearing neither Levis nor Woodland shirts.
ii.) The persons in Lee and Raymond shirts have ordered neither Juice nor Ice cream.
iii.) L is not wearing a Levis shirt and is not on the immediate left of the person who has ordered Fruits.
iv.) The only person who is sitting between P and R likes Shake. The person who is on the left side of the person in
Levis shirt does not eat Patties.
v.) O ordered Fruits and he is wearing Lee shirt. He is facing the person who has ordered Shake.

www.bankingpdf.com
vi.) The one who has ordered Juice is sitting opposite the person in UCB shirt while the person in Lee shirt is on the
left side of the person who ordered Pastries.
vii.) The one who ordered Patties is on the immediate right of the person in Levis shirt but on the immediate left of the
person who ordered for Ice cream.
viii.) N has not ordered Ice cream while R has not ordered Juice.
6). Who among the following is wearing Levis shirt?
a) L
b) M
c) N
d) P
e) None of these
7). The only person who is sitting between P and O is wearing the shirt of which of the following brands?
a) Mufti
b) UCB
c) Woodland
d) Raymond
e) None of these
8). Who among the following has ordered Pastries?
a) 1)M
b) 2)O
c) 3)N

www.bankingpdf.com
d) 4)P
e) 5) None of these
9). Which of the following is correctly matched?
a) L—Raymond—Pastries
b) M—Mufti--Ice cream
c) P—Mufti--Ice cream
d) R—Woodland—Pastries
e) None of these
10). The person who ordered patties is wearing the shirt of which of the following brands?
a) Mufti
b) Raymond
c) UCB
d) Woodland
e) None of these

Answers:
1).d) 2).a) 3).e) 4).c) 5).b) 6).c) 7).d) 8).e) 9).a) 10).d)

www.bankingpdf.com
Solutions:
Direction (01 to 05):
Friend University Subject

Manjit S Maths

Navin Q Economics

Parul P History

Hitesh P Political Science

Raghu S Accounts

Sahil Q Geography

Tarun S Chemistry

Direction (06 to 10):

www.bankingpdf.com
110-120 Questions :

Q 1-5: Study the following information carefully and answer the questions given below.
In a sports event, different games are scheduled to be held on seven days, starting on Sunday and ending on
Saturday. Two games to be held on Friday as well as Saturday and one game on each of the remaining five days.

www.bankingpdf.com
The games to be held in these seven days are Boxing, Wrestling, Tennis, Football, Volleyball, Shooting and Basket
ball, but not necessarily in the same order. Shooting is scheduled on Wednesday. Boxing and Football are scheduled
to be held on the same day. Wrestling is scheduled to be held three days before basketball, i.e., two sports are
scheduled between wrestling and basketball. Wrestling is not scheduled to be held on Tuesday. Tennis is scheduled
immediately after the football is scheduled. Cycling is scheduled immediately after the day wrestling is scheduled.
Swimming is not scheduled on Sunday.
1). Which of the following game is scheduled to be held on Tuesday?
a) Wrestling
b) Shooting
c) Cycling
d) Boxing
e) Tennis

2). On which of the following days is swimming scheduled?


a) Monday
b) Wednesday
c) Tuesday
d) Saturday
e) Sunday

3). Which of the following is true?


a) Monday – cycling

www.bankingpdf.com
b) Sunday – Tennis
c) Friday – Volleyball
d) Wednesday – Boxing
e) Thursday – Basket ball

4). Cycling is related to Wrestling, in the same way as Basket ball is related to?
a) Shooting
b) Boxing
c) Tennis
d) Volleyball
e) Football

5). What are the games that are scheduled on Saturday?


a) Tennis
b) Boxing
c) Football
d) Both a & c
e) Both b & c

Directions (Q. 6- 10): Now in each of the following questions, assuming the given statements to be true, find which of
the two conclusions I and II given below them is / are definitely true. Give answers-
a) if only conclusion I is true.

www.bankingpdf.com
b) if only conclusion II is true.
c) if either conclusion I or II is true.
d) if neither conclusion I nor II is true.
e) if both conclusions I and II are true.

ANSWERS:
Directions(01-05) :

1. C 2. D 3. E 4. A 5. e
Directions (06 to 10):
6. B
For conclusion I : H ≤ K = I > L → FALSE
For conclusion II : H ≤ K ≤ J → TRUE
7. D
For conclusion I : P≥ Q ≥ R < S → FALSE

www.bankingpdf.com
For conclusion II : P≥ Q ≥ R< S ≥ T → FALSE
8. A
For conclusion I : A = B > D→ TRUE
For conclusion II : A = B ≤ C → FALSE
9. C
For conclusion I : E ≥ F ≥ I
For conclusion II : E ≥ F ≥ I
10. E
For conclusion I : X ≤ Y ≤ Z ≤ W → TRUE
For conclusion II : Y ≤ Z ≤ W ≤ V→ TRUE

6). H ≤ K = I; J ≥ K; L < I
I. L > K
II. H ≤ J
7). P≥ Q ≥ R; R < S; S ≥ T
I. S > R
II. Q ≥ T
8). A = B ≤ C; B > D; A ≥ E
I. A > D
II. A > C
9). E ≥ F = H; F ≥ I; E ≤ K

www.bankingpdf.com
I. E > I
II. E = I
10). X ≤ Y ≤ Z; Z ≤ W; W ≤ V
I. X ≤ W
II. V ≥ Y

120-130 Questions :

Q 1-5: Study the following information carefully and answer the questions given below.
I. P, Q, R, S, T, U and V are sitting on a wall and all of them are facing West.
II. S is on the immediate left of R.
III. T is at an extreme end and has Q as his neighbor.
IV. V is between Q and U.
V. S is sitting third from the north end.
1). Who is sitting to the left of S ?
a) Q
b) U
c) T
d) R

www.bankingpdf.com
e) P
2). Which of the following pairs of people are sitting at the extreme ends ?
a) QV
b) PR
c) TP
d) ST
e) VP
3). Name the person who should change places with R such that he gets the fourth place from the south end.
a) P
b) S
c) Q
d) T
e) U
4). Immediately between which of the following pairs of people S is sitting ?
a) UR
b) PQ
c) VP
d) TU
e) RV
5). Which of the conditions given above are not required to find out the place in which P is sitting ?
a) I

www.bankingpdf.com
b) II
c) IV
d) III
e) All required

Q 6-10: Study the following information carefully and answer the questions given below.
I. There are seven books one each on Tamil, Hindi, English, History, Economics, Science and Accountancy, lying on
a table one above the other.
II. Tamil is on the top of all the books.
III. Accountancy is immediately below Science which is immediately below Tamil.
IV. Economics is immediately above History but not in the middle.
V. History is immediately above Hindi.
6). Economics is between which of the following books?
a) Accountancy and Science
b) History and Hindi
c) English and Tamil
d) Tamil and History
e) English and History
7). Which three books are between Accountancy and Hindi?
a) English, Economics and History
b) Economics, History and Science.

www.bankingpdf.com
c) Economics, History and Hindi
d) Science, Hindi and Economics
e) None of these
8). If Tamil and English, Accountancy and Hindi and Education and History interchange their positions, which book
will be between Tamil and History?
a) Accountancy
b) History
c) Economics
d) Hindi
e) Science
9). After interchanging the position of History and Tamil, how many books will be between Science and Tamil?
a) 2
b) 3
c) 4
d) 1
e) None
10). Which book is at fifth position from bottom after interchanging Accountancy and Economics?
a) Accountancy
b) Hindi
c) Economics
d) History

www.bankingpdf.com
e) None of these

ANSWER :
Q 1-5: SEQUENCE: Facing West

1. b 2. c 3. e 4. a 5. e
Q 6-10: SEQUENCE: Top to Bottom

www.bankingpdf.com
6. e 7. a 8. d 9. b 10. c

130-140 Questions :

Directions for Q 1 -5 : Study the following information to answer the questions that follow.
In a certain code `ti se lee’ means `Jasmines are white’, `di ree’ means `yellow petals’ , ‘lee mee se’ means ‘
Jasmines are Beautiful’ and `lee nut ree’ means `petals are greens’.

1). How is `greens are white petals’ written in that code?


a) di ree lee se
b) ti lee nut ree

www.bankingpdf.com
c) lee ti nut mee
d) Cannot be determined
e) None of these

2). How is `Jasmines’ written in that code?


a) ti
b) ree
c) se
d) lee
e) Cannot be determined

3). How is `Yellow are white’ written in that code?


a) lee di ti
b) di ree ti
c) ti se nut
d) Cannot be determined
e) None of these

4). Which of the following codes stands for ‘Beautiful’?

www.bankingpdf.com
a) ree
b) lee
c) se
d) mee
e) Cannot be determined

5). Which code stands for ‘Petals are nice’?


a) ti se hi
b) nut ree ti
c) se lee di
d) di ki lee
e) lee mi ree

6). If PATIENT is coded as 7654325 and EXCEL is coded as 38931, then how would you encode PEACE?
a) 73693
b) 76393
c) 79363
d) 79633
e) 73396

www.bankingpdf.com
7). If green means white, white means yellow, yellow means blue, blue means violet, violet means red, then what will
be the color of SUNFLOWER?
a) White
b) Violet
c) Green
d) Blue
e) Red

8). If white is called blue, blue is called red, red is called yellow, yellow is called green, green is called black, black is
called violet and violet is called orange, what would be the color of human blood?
a) Violet
b) Green
c) Red
d) Blue
e) Yellow

9). If in a certain code, DOCTOR is written as GQFVRT, how would BULLOCK be written in the same code ?
a) EONWREN

www.bankingpdf.com
b) ENORWNE
c) EWONREN
d) EREWONN
e) EWONREN

10). According to a military code, PEPPER is PEPREP and NEARER is AENRER, What is the code for
COMPUTER?
a) COMPRETU
b) PMOCRETU
c) PMOCUTER
d) RETUPMOC
e) None of these

ANSWERS:
1). b; greens are white petals
↓ ↓ ↓ ↓
Nut lee ti ree

2). c; Jasmines →se


3). a; yellow are white
↓ ↓ ↓

www.bankingpdf.com
di lee ti

4). d; Beautiful → mee

5). e; Petals are nice


↓ ↓ ↓
ree lee mi

6). a; P E A C E
↓ ↓ ↓ ↓ ↓
7 3 6 9 3

7). d

8). e

9). c;

10). b; First half is written in reverse and then second half is written in reverse.

140-150 Questions :

www.bankingpdf.com
Directions (Q. 1-5): Read the following information carefully and answer the questions given below.
Veni, Veera, Viraj, Velan, Vendan, Varun and Venu live on 7 different floors of building but not necessarily in the same
order. The lowermost floor is numbered 1, then 2,3 and so on. Each one of them also likes a different company sims
namely Airtel, Aircel, Reliance, Docomo, Idea, Vodafone and Bsnl. Veni lives on an odd numbered floor but not on the
floor numbered 3. The one who likes Idea lives immediately above Veni. Only 2 persons live between Velan and the one
who likes Idea. The one who likes Aircel lives on one of the odd numbered floor above Velan, Only 3 persons live
between Viraj and the one who likes Aircel. The one who likes Reliance lives immediately above Viraj. The one who likes
Airtel lives immediately above one who likes Bsnl. Venu lives on an odd numbered floor, Only one person lives between
Veera and Vendan. Veera lives on one of the floors above Vendan. Neither Viraj nor Veni likes Docomo. Vendan does not
like Reliance.

1). Who lives between Vendan and Varun?


a) Veni
b) Venu
c) Viraj
d) Veera
e) Velan

2). Which of the following sim does Vendan likes?


a) Bsnl
b) Airtel

www.bankingpdf.com
c) Vodafone
d) Docomo
e) None of these

3). Which of the following combination is not true?


a) Velan – 3th floor
b) Venu – 7th floor
c) Vendan – 4th floor
d) Viraj – 5th floor
e) Veni – 5th floor

4). Who among the following lives on the floor numbered 5?


a) Varun
b) Velan
c) Veera
d) Veni
e) Venu

5). Which of the following combination is correct?


a) Veera - Vodafone

www.bankingpdf.com
b) Venu – Airtel
c) Veni – Reliance
d) Varun – Aircel
e) Viraj – Vodafone

Directions (Q. 6-10): Read the following information carefully and answer the questions given below.
YH4T$FO9V*R5QM@IE7K+C2ZWA#N

6). Position wise $ is related to M and Y is the same way as 5 is related to which of the following?
a) 9 and +
b) + and V
c) K and I
d) 4 and I
e) None of These

7). If all the vowels in the above arrangement are dropped which of the following will be the middle element?
a) Q
b) R
c) 5
d) M

www.bankingpdf.com
e) None of these

8). How many such elements are there in the above arrangement, each of which is immediately followed by a vowel and
immediately preceded by a symbol?
a) None
b) One
c) Two
d) Three
e) More than three

9). If all the numbers in the above arrangement are arranged in ascending order from left to right how many numbers will
retain their position within the arrangement after rearrangement?
a) One
b) Two
c) Three
d) Four
e) None of those

www.bankingpdf.com
10). If the positions of the elements in the above arrangement are interchanged i.e. first and twenty sixth, second and
twenty-fifth and so on and the position of the last element is left unchanged which of the following will be tenth from the
left end after rearrangement?
a) *
b) V
c) 7
d) +
e) E
Answers:
1)e 2)b 3)d 4)d 5)e 6)d 7)a 8)c 9)b 10)e

Explanation:

Directions (Q. 1-5):


Floor Person Sims

7 Venu Docomo

6 Veera Idea

5 Veni Aircel

www.bankingpdf.com
4 Vendan Airtel

3 Velan BSNL

2 Varun Reliance

1 Viraj Vodafone

6). $ - M = +8
Y – 5 = +10
4 – I = +12
Answer: d)

7). According to question


YH4T$F9V*R5QM@7K+C2ZW#N
Q will be in middle.
Answer: a)

8). According to question


Symbol – Element – Vowel

www.bankingpdf.com
$–F–O
@–I–E
Two pairs
Answer: c)

9). According to question


YH2T$FO4V*R5QM@IE7K+C9ZWA#N
Two number remains same position
Answer: b)

10). After rearrangement


# AW Z 2 C + K 7 E I @ M Q 5 R * V 9 0 F $ T 4 H Y N
E will be 10th after the rearrangement.
Answer: e)

150-160 Questions :

Directions (Q. 1-5): Read the following information carefully and answer the questions given below.

www.bankingpdf.com
Balachandra, Dharmendra, Harmendra, Jitendra, Mahendra, Ravindra, Virendra and Yogendra live on 8 different floors of
buildings but not necessarily in the same order. The lower most floor of the building numbered 1, the one above that is
numbered 2 and so on. Each of them lives on a different floor also likes a different mobiles – Samsung, Apple, Microsoft,
LG, Lenovo, Xiaomi, Micromax and Oppo.
The one who likes Lenovo lives on an even numbered floor. Only 3 people live between the one who likes Lenovo and
Harmendra. Only 2 people live between Harmendra and Jitendra. Jitendra does not live on the lowermost floor. Only 3
people live between Jitendra and the one who likes Xiaomi. Mahendra lives immediately above Balachandra. Mahendra
lives on an even numbered floor. Balachandra does not like Xiaomi. Balachandra lives in neither on the floor 3 nor 5. Only
2 people live between Balachandra and the one who likes Micromax. Only one person lives between the one who likes
Micromax and Apple. Dharmendra lives immediately above Virendra. Only one person lives between Virendra and the
one who likes LG. The one who likes Microsoft lives on immediately above the one who likes Samsung. Ravindra does
not like Lenovo, Balachandra does not like Apple.

1). Which of the following pair is correct?


a) Ravindra - Apple
b) Jitendra-Oppo
c) Virendra- Lenovo
d) Balachandra – Micromax
e) Mahendra-Microsoft

www.bankingpdf.com
2). Which of the following mobile does Yogendra likes?
a) Xiaomi
b) Lenovo
c) LG
d) Microsoft
e) None of these

3). Who lives below the person one who likes LG?
a) Ravindra
b) Yogendra
c) Dharmendra
d) Harmendra
e) None of these

4). Jitendra lives on which of the following floor numbers?


a) 5
b) 4
c) 3
d) 6

www.bankingpdf.com
e) None of these

5). Who among the following likes Xiaomi?


a) Yogendra
b) Mahendra
c) Harmendra
d) Ravindra
e) None of these

Directions (Q. 6-10): Read the following information carefully and answer the questions given below.

Seven trainees Rajesh, Shalini,Suganya, Rahaman, Sankar, Vinoth and Gokul have been recruited by TCS Ltd. to handle
various projects. They come from different cities - Pune, Agra, Jaipur and Mumbai and have specialised in different fields
i.e. Marketing, Tourism, Banking and Economics Not more than two trainees have been selected with the same
specialisation or from the same city
Shalini and one Tourism specialised belongs to same city.Gokul is from Pune and has specialised in Banking. Rajesh
comes from Jaipur and has not specialised in Banking. Tourism trainees were selected from Agra and Mumbai. No female
trainee has specialised in Economics. A trainee who has specialised in Economics does not come from Jaipur. Vinoth
lives in the same city as Rajesh but does not have the same specialisation. Rahaman and Suganya come from the same
city but not in Mumbai, with different specialisation. Shalini has specialised in Marketing.

www.bankingpdf.com
6). Who comes from Agra?
a) Rahaman
b) Sankar
c) Shalini
d) Cannot be determined
e) None of these

7). Who has studied Marketing?


a) Rajesh
b) Suganya
c) Rahaman
d) Vinoth
e) None of these

8). What has Suganya specialised in?


a) Banking
b) Tourism
c) Marketing
d) Data inadequate

www.bankingpdf.com
e) None of these

9). Which of the following combinations is definitely incorrect?


a) Banking-Pune
b) Economics-Agra
c) Marketing- Jaipur
d) Marketing - Pune
e) None of these

10). Which of the following must be TRUE?


a) Sankar specialises in Tourism
b) Suganya specialises in Economics
c) Rahaman specialises in Tourism
d) Sankar comes from Pune
e) None of these

Answers:
1)e 2)b 3)c 4)a 5)d 6)a 7)a 8)b 9)d 10)a

www.bankingpdf.com
Directions (Q. 1-5):
Floor Person Mobiles

8 Mahendra Microsoft

7 Balachandra Samsung

6 Yogendra Lenovo

5 Jitendra LG

4 Dharmendra Micromax

3 Virendra Oppo

2 Harmendra Apple

1 Ravindra Xiaomi

Directions (Q. 6-10):


Trainees City Field

www.bankingpdf.com
Rajesh Jaipur Marketing

Shalini Mumbai Marketing

Suganya Agra Tourism

Rahaman Agra Economics

Sankar Mumbai Tourism

Vinoth Jaipur Banking

Gokul Pune Banking

www.bankingpdf.com
160-170 Questions ;

Directions (Q. Nos. 1-5): Study the following information carefully and answer the questions given below it.

Digits in the numbers are to be coded as follows

Letters J A L S E Q G D K M B I N P R

Number / 8 5 # 9 2 £ 6 % 3 7 * 4 @ $ 1
Symbol Code

Conditions
(i) If the first letter is a vowel and the last a consonant, the codes for the first and the last are to be interchanged.
(ii) If the first letter is a consonant and the last a vowel, both are to be coded as the code of the vowel.
(iii) If no vowel is present in the group of letters, the second and the fifth letters are to be coded as ©.

1). EMANRB
a) *75@12
b) 275@1*
c) ©75@2©
d) *75@1*

www.bankingpdf.com
e) None of these

2). QDBGRM
a) £%*617
b) £©*6©7
c) £%*167
d) % £*61©
e) None of these

3). KQAPJE
a) 3£5$82
b) 3£58$2
c) 2£5$82
d) 2£5$83
e) None of these

4). JAQDKP
a) 85£%3$
b) $5£%3$
c) $5£%38

www.bankingpdf.com
d) $5£3%8
e) None of these

5). IKQLMS
a) 43£#74
b) ©3£#7©
c) 4£3#74
d) 93£#74
e) None of these

Directions (6-10): In these questions, certain symbols have been used to indicate relationships between elements as
follows:
A * B means A is either equal to or greater than B
A $ B means A is equal to B
A £ B means A is either equal to or smaller than B
A & B means A is smaller than B
A @ B means A is greater than B

6). Statements: M @ T, O @ R, O & T


Conclusions:

www.bankingpdf.com
I. O £ M
II. R & T
III. M @ R
a) Only I and II follows
b) Only II and III follows
c) Only I and III follows
d) None follows
e) None of these

7). Statements: R * H, T & H, H * B


Conclusions:
I. R $ B
II. R @ B
III. R @ T
a) I and II only follows
b) Either I or II and III only follows
c) Only I follow
d) II and III only follows
e) None of these

8). Statements: E @ R, M @ L, E & L

www.bankingpdf.com
Conclusions:
I. L @ R
II. E & M
III. M @ R
a) Only I and II follows
b) Only II and III follows
c) Only I and III follows
d) None follows
e) All follows

9). Statements: Y $ V, H $ S, V * S
Conclusions:
I. S £ H
II. H £ V
III. Y * S
a) Only II follows
b) Only I and II follows
c) c) Only III follows
d) Only II and III follows
e) None of these

www.bankingpdf.com
10). Statements: J & C, C * B, S * C
Conclusions:
I. B * J
II. S £ J
III. S @ B
a) None follows
b) Only I follow
c) Only II follows
d) Only III follows
e) Only II & III follows

Answers:
1)a 2)e 3)c 4)b 5)d 6)b 7)b 8)e 9)d 10)d

Explanation:
1). EMANRB - *75@12
Condition ii follows
Answer: a)
2). QDBGRM - £©*6©7
Condition iii follows
Answer: e)

www.bankingpdf.com
3). KQAPJE - 2£5$82
Condition ii follows
Answer: c)
4). JAQDKP - 85£%3$
No condition follows
Answer: b)
5). IKQLMS - 93£#74
Condition I follows
Answer: d)
Directions (Q. 6-10):
A * B means “A ≥ B”
A $ B means “A = B”
A £ B means “A ≤ B”
A & B means “A < B”
A @ B means “A > B”
6). M > T, O > R, O < T
O ≤ M not follows
R < T follows
M > R follows
Answer: b)
7). R ≥ H, T < H, H ≥ B
R=B

www.bankingpdf.com
R>B
R>T
Either I or II and III follows
Answer: b)
8). E > R, M > L, E < L
L>R
E<M
M>R
All follows
Answer: e)
9). Y = V, H = S, V ≥ S
S ≤ H not follows
H ≤ V follows
Y ≥ S follows
Answer: d)
10). J < C, C ≥ B, S ≥ C
B ≥ J not follows
S ≤ J not follows
S > B follows
Answer: d)

www.bankingpdf.com
170-180 Questions :

Directions (Q.1-4): In each question below are given three statements followed by four conclusions numbered I, II,. You
have to take the given statements to be true even if they seem to be at variance with commonly known facts. Read all the
conclusions and then decide which of the given conclusions logically follows from the given statements, disregarding
commonly known facts.

a) If only conclusion I follow


b) If only conclusion II follow
c) If either conclusion I or II follow
d) If neither conclusion I nor II follow
e) If both conclusion I and II follow
1). Statement: Some boards are clocks.
All boards are boxes
No box is a wall
Conclusion: I. Some boxes are clocks
II. All clocks being boxes is a possibility.

2). Statement: No home is a hut.


Some huts are houses.
Some houses are table.

www.bankingpdf.com
Conclusion: I. Some houses are not homes.
II. All tables being huts is a possibility.

3). Statement: Some cats are rats.


Some rats are fishes.
All fishes are birds.
Conclusion: I. Some fishes are rats.
II. All cats being birds is a possibility.

4). Statement: Some flowers are roses.


No rose is red.
All red are leaves.
Conclusion: I. Some flowers are definitely not red.
II. Some leaves are definitely not roses.

Directions (Q. 5-8): In each of the following questions, assuming the given statements to be true, find which of the given
two conclusions numbered I, II and II is/are definitely true and give your answer accordingly .
a) Only conclusion I is true.
b) Only conclusion II is true.

www.bankingpdf.com
c) Either conclusion I or II is true
d) Both Conclusion I and II is true
e) Neither Conclusion I and II is true.

5). Statement: L = I ≥ N < E; N ≥ S


Conclusion: I. S < E
II. E > L

6). Statement: V < E > B = H ≥ N; B ≤ T


Conclusion: I. T > V
II. N ≤T

7). Statement: L = I ≥ N < E; N ≥ S


Conclusion: I. S = L
II. L > S

8). Statement: B > E ≥ A < L; T ≥ A > S


Conclusion: I. B > S

www.bankingpdf.com
II. L > S

Directions (Q. 9-10): Study the following information and answer the questions given below.
‘A + B’ means ‘A’ is father of ‘B’
‘A ÷ B’ means ‘B’ is brother of ‘A’
‘A × B’ means ‘A’ is husband of ‘B’
‘A - B’ means ‘A’ is sister of ‘B’
9). In the expression A + B × C – D ÷ E, how is D related to B?
a) Brother in law
b) Sister in law
c) Nephew
d) Brother
e) Cannot be determined

10). Which of the following expressions shows that ‘R is sister of Q’?


a) P + Q – R + Z
b) T × Q + Z – R
c) P + Q – R × Z
d) None of these
e) Cannot be determined

www.bankingpdf.com
Answers:
1).e) 2).e) 3).e) 4).e) 5).a) 6).b) 7).c) 8).d) 9).e) 10).d)

Solution:
5). Given Statements:
L = I ≥ N < E;
N≥S
Combining above both we get
S ≤N , E
Thus S < E is true.
Hence I is true.
Again from I we can’t compare L and E
hence II (E > L) is not true
Answer: a)
6). Given Statements:
V < E > B = H ≥ N;
B≤T
Combining above both we get
V<E>B≤T
thus, we can’t compare V and T.
hence I (T > V) is not true.

www.bankingpdf.com
Again, from (I) and (ii), we get
T≥B=H≥N
thus, T ≥ N or N ≤ T is true
hence II is true
Answer: b)
7). Given Statements
L = I ≥ N < E;
N≥S
Combining above both we get
L=I≥N≥S
thus, L ≥ S or S ≤ L is true
it means L > S may be true or L = S may be true.
Hence conclusion I and II make a complementary pair.
so, either I or II is true
Answer: c)
8). Given statements:
B > E ≥ A < L; T ≥ A > S
Combining above we get
B>E≥A<L
thus B > S is true
hence I is true
Again S < A < L

www.bankingpdf.com
So, S < L or L > S is true
hence both I and II is true
Answer: d)
9). hence D may be either male or female so, we cannot be determined.
Answer: e)
10).

Answer: d)

180-190 Questions :

Directions (Q.1-5): Study the following information carefully and answer the questions given below:
Ten members Q,R,S,T,U,V,W,X,Y and Z are made to sit in two rows- I and II, five members in each row. They are facing
each other in such a way that the members of row I are facing the members of row II. The members of row I are facing
south. T is third to the right of Q and is exactly opposite W.R sits on the immediate right of W and is facing south. X is third
to the right of R and is exactly opposite V. S sits between Q and U and is opposite Y.
1).Who among the following sits second to the left of S?
a) T
b) V

www.bankingpdf.com
c) Q
d) Z
e) None of these

2).Which of the following groups of persons sit in the same row?


a) U,X,Y
b) Q,R,S
c) W,Y,Z
d) W,T,Y
e) None of these

3).Who among the following sits on the immediate right of R?


a) S
b) Y
c) Z
d) U
e) None of these

4).Who among the following sits exactly opposite Z?

www.bankingpdf.com
a) Q
b) T
c) V
d) Can’t be determined
e) None of these

5).Who among the following sit on the extreme ends of row I?


a) W,X
b) Z,Y
c) V,T
d) S,Y
e) None of these

Directions (Q.6-10): Study the following information carefully and answer the questions given below:
Eight friends Shahil, Sonali, Nitesh, Vidya, Rohit, Gita, Sanchita and Deepak are made to sit in two rows I and II
with four members in each row. Shahil, Gita, Sanchita and Deepak are facing south and sit in row I. Sonali, Nitesh, Vidya
and Rohit are facing north and sit in row II. Each members of row I is facing a member from row II. There are two Doctors,
two Engineers, two Judges and two Commissioners in the group. No two persons belonging to the same profession sit in
the same row.

www.bankingpdf.com
The Engineers of row I sits on the immediate right of Deepak. The Doctor of row II faces the immediate neighbor
of the Engineer of row I. Vidya faces the immediate neighbor of Gita, who is not and Engineer. There is only one person
between the Judge and the Commissioner in both rows. The Commissioner of row I faces the Engineer of row II. Sonali is
not a Judge.
Gita sits on the immediate left of Deepak. Nitesh is not the Judge and he does not sit at any ends of the row. The
judge of row II faces Shahil. Vidya is not an immediate neighbor of Rohit.

6).Who among the following sits between Sonali and Vidya?


a) Sanchita, who is a judge
b) Sonali, who is an engineer
c) Rohit, who is a judge
d) Nitesh, who is a commissioner
e) None of these

7).Who among the following sit at the ends of the rows?


a) Commissioner of row II and Judge of row I
b) Judge of row II and Doctor of row I
c) Engineer of row I and Engineer of row II
d) Doctor of row II and Judge of row I
e) None of these

www.bankingpdf.com
8).Which of the following is the pair of Doctors?
a) Sonali,Deepak
b) Gita,Sonali
c) Nitesh,Vidya
d) Shahil,Sanchita
e) None of these

9).Who among the following sits diagonally opposite the Judge of row II?
a) Gita
b) Sanchita
c) Shahil
d) Vidya
e) None of these

10). Four of the following five are alike in a certain way and hence form a group. who among the following do not belong
to that group?
a) Shahil,Vidya
b) Rohit,Sanchita

www.bankingpdf.com
c) Sonali,Gita
d) Nitesh,Sanchita
e) Rohit,Deepak

Answer:
1)b 2)C 3)b 4)a 5)a 6)d 7)c 8)b 9)b 10)b

Explanation:
Directions (Q. 1-5):

Directions (Q. 6-10):

www.bankingpdf.com
190-200 Questions :

Directions (Q. 1-5): Study the following information carefully and answer the question given below.
Ten persons J,K,L,M,N,P,Q,R,S and T are sitting in two rows with five persons in each row. The persons in row one are
facing south and the persons in row two are facing north. Each person in row one faces a person from the other row.
All of them like a different soft drink ,viz sprite, slice, pepsi, maaza, fanta, mirinda, limca, 7up, nimbooz and cola, but not
necessary in the same order.
The persons who like fanta and mirinda sit opposite each other. P sits opposite J, who likes sprite. The one who likes slice
sits opposite the one who likes 7up.T is not facing north but sits third to the left of Q, who likes slice. There is only one
person between K and L. N sits at one of the ends of the row and likes mirinda. The one who likes 7up is on the

www.bankingpdf.com
immediate right of M, who does not like cola. The persons who like pepsi and maaza respectively are not facing north. L
likes limka. The one who likes maaza sits opposite the one who is second to the right of K. S does not like cola. N sits
opposite the one who sits second to the left of the one who likes pepsi.
1). Who among the following likes cola?
a) R
b) T
c) P
d) S
e) None of these
2). Who among the following sit at the extreme ends of the row and are facing south?
a) M,N
b) S,L
c) T,N
d) R,T
e) None of these
3). How many persons sit between M and N?
a) one
b) two
c) three
d) can’t be determined
e) None of these
4). K likes which of the following soft drinks?

www.bankingpdf.com
a) Fanta
b) 7up
c) Cola
d) Maaza
e) None of these
5). which of the following statements is/are true?
a) R likes Cola and sits at one of the ends of the row.
b) P is the immediate neighbour of Q and the person who likes fanta.
c) The one who likes limca sits on the immediate left of the one who likes mirinda.
d) only a) and c)are true
e) None of these.
6). Four of the following five are alike in a certain way and hence form a group.Which is the one that does not belong to
that group?
a) R,Q
b) N,L
c) M,K
d) T,S
e) S,L

Directions (Q. 7-9): Read the following information carefully and answer the questions which follow:

www.bankingpdf.com
A, B, C, D, E and F live on different floors in the same building having six floors numbered one to six (the ground floor is
numbered 1, the floor above it, numbered 2 and so on and the topmost floor is numbered 6).
A lives an odd numbered floor. F lives on even numbered floor neither 6 nor 2. C lives immediately above the floor on
which E lives who does not live odd number floor. C does not live on an even numbered floor. There are 3 persons lives
between C and D. A lives immediately below E.
7). On which of the following floors does E live?
a) 2nd
b) 4th
c) 6th
d) 3rd
e) cannot be determined
8). Who amongst the following lives on the topmost floor?
a) B
b) E
c) F
d) D
e) cannot be determined
9). Who amongst the following lives on the floor exactly between the floors on which B and C live?
a) A
b) D
c) E
d) F

www.bankingpdf.com
e) Either D or E
10). How many persons lives between C and F?
a) None
b) one
c) two
d) three
e) more than three

Answer:
1)a 2)d 3)c 4)b 5)d 6)e 7)a 8)d 9)d 10)a

Explanation:
Directions (Q. 1-6):
Directions (Q. 7-10):

Floor Person

6 D

5 B

www.bankingpdf.com
4 F

3 C

2 E

1 A

www.bankingpdf.com
200-210 Questions :

1). Rachna walked from A to B in the East 10 feet. Then she turned to the right and walked 3 feet. Again she turned to the
right and walked 14 feet. How far is she from A?
a) 4 feet
b) 5 feet
c) 24 feet
d) 27 feet
e) None of these

2). One morning after sunrise Nivedita and Niharika were talking to each other face to face at Dalphin crossing. If
Niharika's shadow was exactly to the right of Nivedita, Which direction Niharika was facing?
a) North
b) South
c) East
d) Data is inadequate
e) None of these

www.bankingpdf.com
3). Ranveer left home and cycled 10 km towards South, then turned right and cycled 5 km and then again turned right and
cycled 10 km. After this he turned left and cycled 10 km. How many kilometers will he have to cycle to reach his home
straight?
a) 10 km
b) 15 km
c) 20 km
d) 25 km
e) None of these

Directions (Q. 4-5): In the following questions, the symbols *, $, @, %, and # are used with the following meanings:

P * Q means P is greater than Q.


P $ Q means P is either greater than or equal to Q.
P @ Q means P is equal to Q.
P % Q means P is smaller than Q.
P # Q means P is either smaller than or equal to Q.
Now in each of the following questions, assuming the given statement to be true, find which of the two conclusions I and II
given below them is/are definitely true. Give answer.
a) if only conclusion I is true;
b) if only conclusion II is true;
c) if either I or II is true;

www.bankingpdf.com
d) if neither I NOR II is true; and
e) if both I and II are true.

4). Statements: M*N, N#Q, Q#D


Conclusions: I. M*D II.Q@D

5). Statements: V*X, U#Z, X@U


Conclusions: I. V*Z II. X@Z

Directions (Q. 1-3):


A + B means A is the father of B ;
A - B means A is the wife of B ;
A x B means A is the brother of B ;
A ÷ B means A is the daughter of B.

6). If P x R ÷ Q, which of the following statements is true ?


a) P is the uncle of Q
b) P is the father of Q
c) P is the brother of Q
d) P is the son of Q

www.bankingpdf.com
e) None of these

7). If P x R - Q, which of the following is true ?


a) P is the brother-in-law of Q
b) P is the brother of Q
c) P is the uncle of Q
d) P is the father of Q
e) None of these

8). If P x R + Q, which of the following is true ?


a) P is the uncle of Q
b) P is the father of Q
c) P is the brother-in-law of Q
d) P is the grandfather of Q
e) None of these

9). If A + B means A is the mother of B; A – B means A is the brother B; A % B means A is the father of B and A x B
means A is the sister of B, which of the following shows that P is the maternal uncle of Q?
a) Q – N + M x P
b) P + S x N – Q

www.bankingpdf.com
c) P – M + N x Q
d) Q – S % P
e) None of these

10). If A + B means A is the brother of B; A x B means A is the son of B; and A % B means B is the daughter of A then
which of the following means M is the maternal uncle of N?
a) M + O x N
b) M % O x N + P
c) M + O % N
d) Cannot be determined
e) None of these

Answer:
1)b 2)a 3)b 4)d 5)d 6)d 7)a 8)a 9)c 10)d

Solution:
1).

www.bankingpdf.com
Answer: b)

2).

In the morning sun rises in the East. Hence then any shadow falls in the West. Since Nikharia's shadow was exactly to the
right of Nivedita. Hence Nikharia is facing towards North.
Answer: a)

www.bankingpdf.com
3).

Answer: b)

4).
Answer: d)

5).
Answer: d)

6). P X R ÷ Q means P is the brother of R who is the daughter of Q i.e. P is the son of Q.

www.bankingpdf.com
Answer: d)

7). P x R - Q means P is the brother of R who is the wife of Q i.e. P is the brother-in-law of Q.
Answer: a)

8). P is the uncle of Q


P x R + Q means P is the brother of R who is the father of Q i.e. P is the uncle of Q
Answer: a)

9). P – M → P is the brother of MM + N → M is the mother of NN x Q → N is the sister of Q Therefore, P is the maternal
uncle of Q.
Answer: c)

10). Because the sex of O is not known.


Answer: d)

www.bankingpdf.com
210-220 Questions :

Directions (1-5): Study the following information carefully and answer the questions given below it :
(i) Eleven students A, B, C, D, E, F, G, H, I, J and K are sitting in a row of the class facing the teacher.
(ii) D, who is to the immediate left of F, is second to the right of C.
(iii) A, is second to the right of E, who is at one of the ends.
(iv) J is the immediate neighbour of A and B and third to the left of G.
(v) H is to the immediate left of D and third to the right of I.

1). Who is sitting in the middle of the row ?


a) C
b) I
c) B
d) G
e) None of these

2). Which of the following group of friends is sitting to the right of G ?


a) IBJA
b) ICHDF
c) CHDF
d) CHDE
e) None of these

www.bankingpdf.com
3). In the above sitting arrangement, which of the following statements could be dispessed with?
a) (i)
b) (ii)
c) (iii)
d) (iv)
e) None

4). Which of the following statements is true in the above sitting arrangement ?
a) There are three students sitting between D and G.
b) G and C are neighbours sitting to the immediate right of H.
c) B is sitting between J and I.
d) K is sitting between A and J.
e) None of these.

5). If E and D, C and B, A and H and K and F interchange their positions, which of the following pair of students is sitting
at the end ?
a) D and E
b) E and F
c) D and K

www.bankingpdf.com
d) K and F
e) None of these

Directions (6-11): Study the following information carefully and answer the question given below:
Eight friends Heena , Chandan, Nita, Gaurav, Bhavesh, Pranav, Dinesh and Isha are sitting in a row facing north .All of
them like different colours , viz Red,Orange,Green,Yellow,Black,Violet and Blue.
 There is only one person between nita and the one who likes Violet.
 Dinesh is neither an immediate neighbour of Nita nor he likes Green.
 Heena sits fourth to the left of the one who likes violet but she does not like Pink.
 The person who likes Black is third to the right of the one who likes Green and sits on the immediate right of Heena.
 The one who likes Green sits at one of the extreme ends of the row. Chandran does not like Green.
 Pranav is an immediate neighbour of both Dinesh and Nita.
 Isha sits at one of the extreme ends of the row but she does not like Green.
 The one who likes Blue sits second to the right of the one who likes Orange.
 The ones who like Black and Pink are immediate neighbour.
 Bhavesh sits third to the left of Nita and likes Yellow.
 There is only one person between the persons who like Yellow and Black.

6). How many persons are there between Gaurav and Chandra?
a) Two
b) Three

www.bankingpdf.com
c) Four
d) Five
e) None of these

7). Who among the following is an immediate neighbour of the person who like Black and Yellow?
a) Dinesh
b) Gaurav
c) Heena
d) Chandan
e) None of these

8). Who among the following likes Orange?


a) Nita
b) Pranav
c) Dinesh
d) Bhavesh
e) None of these

9). Who among the following sits on the immediate left of the one who likes Blue?

www.bankingpdf.com
a) The one who likes Orange
b) Either Pranav or Nita
c) The one who likes violet
d) The one who likes Black
e) None of these

10). Who among the following sits third to the Right of the person who likes Red?
a) Nita
b) Gaurav
c) Dinesh
d) Pranav
e) None of these

11). Which of the fallowing statements is /are true?


a) Bhavesh is on the immediate right of Gaurav and likes Green.
b) Heena sits second to the right of nita.
c) Dinesh is an immediate neighbour of Isha and Nita.
d) Only 1) and 3)are true
e) None of these

www.bankingpdf.com
Answer:
1)b 2) c 3) e 4) c 5) c 6) a 7) c 8) b 9) c 10) d 11) e

Solutions:
Directions (Q. 1-5):

Directions (Q. 6-11):

www.bankingpdf.com
220-230 Questions :

1). Kunal walks 10 Kilometers towards North. From there, he walks 6 kilometres towards South. Then, he walks 3
kilometres towards East. How far and in which direction is he with reference to his starting point ?

a) 5 kilometres West
b) 5 kilometres North-east
c) 7 kilometres East
d) 7 kilometres West
e) None of these

2). A person starts from a point A and travels 3 km eastwards to B and then turns left and travels thrice that distance to
reach C.He again turns left and travels five times the distance he covered between between A and B and reaches his
destination D. The shortest distance between the starting point and the destination is
a) 12 km
b) 15km
c) 16 km
d) 18 km
e) None of these

www.bankingpdf.com
3). A man walks 1 km towards East and then he turns to South and walks 5 km. Again he turns to East and walks 2 km,
after this he turns to North and walks 9 km. Now, how far is he from his starting point ?
a) 3 km
b) 4 km
c) 5 km
d) 7 km
e) None of these

4). Laxman went 15 kms to the west from my house, then turned left and walked 20 kms. He then turned East and walked
25 kms and finally turning left covered 20 kms. How far was he from his house?
a) 5 km
b) 10 km
c) 40 km
d) 80 km
e) None of these

5). A walks 10 metres in front and 10 metres to the right. Then every time turning to his left, he walks 5, 15 and 15 metres
respectively. How far is he now from his starting point ?
a) 5 metres
b) 10 metres

www.bankingpdf.com
c) 15 metres
d) 20 metres
e) 23 metres

Directions-(6-10) Study the following information to answer the given questions:


In a certain code 'best way to win' is written as 'ad mi ja no', ' the way to hell' is written as 'ku ja ig ad'. 'win of the day' is
written as 'be ku zo mi' and 'to sell of night' is written as 'be Ii ya ja'.

6). What is the code for 'sell'?


a) be
b) Ii
c) ya
d) ja
e) Cannot be determined

7). Which of the following may represent 'hell is way'?


a) ig ad no
b) ig py ya
c) re ad be

www.bankingpdf.com
d) ig li re
e) ad re ig

8). ‘mi’ is the code for


a) to
b) win
c) way
d) of
e) Cannot be determined

9). What is the code for' best'?


a) ad
b) mi
c) no
d) ja
e) Cannot be determined

10). Which of the following represents 'of the way'?


a) ku be ad

www.bankingpdf.com
b) rni be no
c) ku be ya
d) mi ku be
e) be mi ad

Answer:
1)b 2)b 3)c 4)b 5)a 6)e 7)e 8)b 9)c 10)a

Solutions
Directions (1-5):

www.bankingpdf.com
Directions (6-10):
word best way to win the hell of day sell night

code no ad ja mi ku ig be zo li/ya ya/li

230-240 Questions :

Directions (Q1-5): Study the following information carefully and answer the question given below
There are twelve friends sitting in two parallel rows containing six people each. In row-1: L, M, N, O, P and Q are seated
and all of them are facing south. In row-2; Z, Y, X, W, V and U are seated and all of them are facing north. Therefore, in
the given seating arrangement each member seated in a row faces another member of the other row.
P faces W, who does not sit at extreme ends. Z sits third to the right of W.Z does not sit at extreme ends. Q does not face
Z ,and he does not sit at either of the extreme ends. Q is not on immediate neighbour of P.Y sits alone of the extreme
ends only two persons sits between Y & V .V does not face Q. Two persons sits between N & M. N is not an immediate
neighbor of P. X does not face Q. l is not at an immediate neighbour of N.

1). Who among the following faces Z?


a) L
b) M

www.bankingpdf.com
c) N
d) O
e) P

2). How many persons are sitting between P & O?


a) Two
b) Three
c) Four
d) None of these
e) One

3). Who among the following sits at the extreme ends of the rows?
a) Q, O
b) L, N
c) X, Z
d) N, Q
e) Y, V

4). N is related to M in the same way as O is related to P .Which of the following U related to, following the same pattern?
a) Q

www.bankingpdf.com
b) W
c) X
d) Y
e) None of these

5). Who among the following sits exactly between V & Z?


a) W
b) U
c) Y
d) Cannot be determined
e) None of these

Directions (Q 6-10): Study the following information carefully and answer the question given below:
Eight friends B,C,D,E,G,H,N and O are sitting in a row facing north. All of them working different banks, viz ICICI,
HDFC,IDBI ,SBI ,BOI ,OBC ,CITY BANK and DENA BANK, but not necessarily in the same order.
H sits fourth to the left of the person who works in IDBI. N sits between O and C but does not works in IDBI.B, who works
in HDFC, sits second to the left of C, who works in OBC.B does not sit on the extreme ends of the row.
The person who works in BOI sits on the immediate left of the, who works in SBI.E works in CITY BANK and is not an
immediate neighbor of B. The person who works in DENA BANK is not an immediate neighbour of the one who works in
OBC. D does not work in DENA BANK. G sits on one end of the row.

www.bankingpdf.com
6). Who among the following sit on the extreme ends of the row?
a) Those who work in IDBI and SBI
b) The person who work in CITY BANK and DENA BANK
c) G and the one who works in ICICI
d) Cannot be determined
e) None of these

7). How many persons are there between E and the one who works in BOI?
a) Two
b) One
c) Three
d) Five
e) None of these

8). Based on the given arrangement, which of the following is true with respect to D?
a) Only one person sits to the left of D.
b) Only one person sits between D and C.
c) None of the given options is true.
d) Both who work in SBI and CITY BANK are immediate neighbour of D.

www.bankingpdf.com
e) N sits second to the right of D

9). Who among the following is exactly between the person who work in IDBI and OBC?
a) N,O
b) C,N
c) H,O
d) B,N,E
e) None of these

10). N works in which of the following banks?


a) ICICI
b) DENA BANK
c) BOI
d) Cannot be determined
e) None of these

Answer:
1)d 2)a 3)b 4)c 5)b 6)b 7)a 8)d 9)a 10)c

www.bankingpdf.com
Solutions
Direction -(1-5)

Directions (6-10):

www.bankingpdf.com
240-250 Questions :

Directions (Q.1-5): In each of the questions given below three statements are followed by two conclusions numbered
I and II. You have to take the given statements to be true even if they seem to be at variance with commonly known
facts. Read all the conclusions and then decide which of the given conclusions and then decide which of the given
conclusions logically follows from the given statements, disregarding commonly known facts. Give answer
a) If only conclusion I follows
b) If only conclusion II follows
c) If either conclusion I or conclusion II follows
d) If neither conclusion I nor conclusion II follows
e) If both conclusions I and II follow.
1). Statements: All right are wrong
Some rights are false
All false are true
Conclusion: I. Some false are wrong
II. All true being wrong is a possibility.
2). Statement: No toy is a tree.
Some toys are tough.
All trees are trains.
Conclusions: I. All trees being tough is a possibility.
Ii. All trains being toys is a possibility.
3). Statements: No book is a ball.

www.bankingpdf.com
Some books are boys.
All boys are baskets.
Conclusion: I. All baskets being books is a possibility
II. All balls being boys is a possibility.
4). Statements: Some foods are forests.
Some floods are flutes.
All forests are frozen.
Conclusions: I. Some foods are not frozen is a possibility.
II. Some frozen are forests.
5). Statements: All ports are airports.
Some airports are harbours.
Some harbours are docks.
Conclusions : I. All docks being airports is a possibility.
II. All ports are harbours.

Directions (Q. 6-8): Study the following information and answer the given questions.
M is the mother of B. A is the husband of M. N is the only brother of B. C is married to N. Q is the only child of C. N
does not have any sister. J is the father of A.
6). If A does not have any grandson, then how is Q related to B?
a) Cannot be determined

www.bankingpdf.com
b) Sister in law
c) Daughter in law
d) Niece
e) Nephew
7). How is A related to C?
a) Uncle
b) Cannot be determined
c) Father in law
d) Nephew
e) Brother in law
8). How is B related to J?
a) Father
b) Nephew
c) Brother in law
d) Brother
e) Grandson

Directions (9-10): Study the following information carefully and answer the questions given below:

www.bankingpdf.com
There are seven family members A, B, C, D, E, F and G in which there are two married couples. A is sister of B, who
is maternal grandson of E. the father of D has two maternal grandchildren. C is sister in law of G, who is father of A.
F is a female.
9). What is the relation between D and F?
a) F is mother of D
b) D is brother of F
c) D is son of F
d) D is daughter in law of F
e) Cannot be determined
10). Who among the following is husband of F?
a) G
b) D
c) A
d) E
e) None of these

Answers with Solution:


1). Some rights are false (I)  conversion  Some false are right (I) + All rights are wrong (A) = I + A = I = Some
false are wrong (I). hence, conclusion I follows. Since there is no negative statement, conclusion II also follows.
Answer: e)

www.bankingpdf.com
2). No toy is a tree Conversion  no tree is a toy(E) + Some toys are tough (I) = E + I = O*= Some tough are not
trees (O*). Thus, there is a possibility that all trees are tough. Hence, conclusion I follows. Again, No toy is a tree (E)
+ All trees are trains (A) = E + A = O* = Some trains are not toys. So, there is no possibility that All trains are toys.
Hence, conclusion Ii does not follow.
Answer: a)

3). Some books are boys (I) + All boys are baskets (A) = I + A = I = Some books are baskets. Hence, All baskets a
being books is a possibility. Thus, conclusion I follows. Some books are boys  conversion  Some boys are
books(I) + No book is a ball (E) = I + E = O = Some boys are not balls. Thus, All boys being balls is not a possibility.
Hence, conclusion II does not follow.
Answer: a)

4). Some foods are forests (I) + All forests are frozen (A) = I + A = I = Some foods are frozen. So, there is a
possibility that some foods are not frozen. Thus, conclusion I follows. Again, All forests are frozen
(A)  conversion  Some frozen are forests (I). Thus, conclusion II also follows.
Answer: e)

5). Since there is no negative statement. All docks being airports is a possibility. Hence, conclusion I follows. Again,
All ports are airports (A) + Some airports are harbours (I) = No conclusion. Hence, conclusion, II does not follow.

www.bankingpdf.com
Answer: a)

Direction (6-8):

Answer: d)
Answer: c)
Answer: e)
Direction (9-10):

www.bankingpdf.com
Answer: a)
Answer: d)

250-260 Questions :

Directions (1-5) : Study the following information and answer the questions given below.
Eight people E, F, G, H, J, K, L and M are sitting around a circular table facing the centre. Each of them is of a different
profession Chartered Accountant, Columnist, Doctor, Engineer, Financial Analyst, Lawyer, Professor and Scientist but not
necessarily in the same order. F is sitting second to the left of K. The Scientist is an immediate neighbour of K. There are
only three people between the Scientist and E. Only one person sits between the Engineer and E. The Columnist is to the
immediate right of the Engineer. M is second to the right of K. H is the Scientist. G and J are immediate neighbours of
each other. Neither G nor J is an Engineer. The Financial Analyst is to the immediate left of F. The Lawyer is second to
the right of the Columnist. The Professor is an immediate neighbour of the Engineer. G is second to the right of the
Chartered Accountant.

1). Who is sitting second to the right of E?


a) The Lawyer
b) G

www.bankingpdf.com
c) The Engineer
d) F
e) K

2). Who amongst the following is the Professor?


a) F
b) L
c) M
d) K
e) J

3). Four of the following five are alike in a certain way based on the given arrangement and hence from a group. Which of
the following does not belong to that group?
a) Chartered Accountant - H
b) (2) M - Doctor
c) (3) J - Engineer
d) (4) FinancialAnalyst - L
e) (5) Lawyer - K

4). What is the position of L with respect to the Scientist ?

www.bankingpdf.com
a) Third to the left
b) Second to the right
c) Second to the left
d) Third to the right
e) Immediate right

5). Which of the following statements is true according to the given arrangement?
a) The Lawyer is second to the left of the Doctor
b) E is an immediate neighbour of the Financial Analyst
c) H sits exactly between F and the Financial Analyst
d) Only four people sit between the Columnist and F
e) All of the given statements are true

Directions (Q. 6-10): Study the information below and answer the following question: –
In a certain code ‘facing problem with money’ is coded as ‘st np cg rt’, ‘money problem very serious’ is coded as ‘nt st np
vw’, ‘serious with every person’ is coded as ‘rt nt pr ab’, ‘facing person each day’ is coded as ‘cg ab no cd’.
6). What is the code for ‘pr’?
a) every
b) with
c) serious
d) person

www.bankingpdf.com
e) None of these

7). what is the code for ‘rt nt np’?


a) with serious problem
b) with serious money
c) serious money facing
d) either a or b
e) None of these

8). Which of the following is the code for ‘facing’?


a) np
b) st
c) cg
d) vw
e) None of these

9). What is the code for ‘Person with money’?


a) ab rt cg
b) ab cg no
c) ab rt st

www.bankingpdf.com
d) st np cg
e) st pr vw

10). Which of the following is the code for ‘facing’?


a) ab
b) cg
c) no
d) nt
e) None of these

Answers:
1).b) 2).d) 3).c) 4).b) 5).a) 6).a) 7).d) 8).c) 9).c) 10).b)

Explanation:
Directions (Q. 1-5):

www.bankingpdf.com
Directions (Q. 6-10):
Facing = cg
With = rt
Serious = nt
Person = ab
Very = vw
Every = pr
Money problem = st np
Each day = no cd

www.bankingpdf.com
260-270 Questions :

Directions (Q. 1-5): Study the following information carefully to answer the given questions
Seven Persons – M, N, O, P, Q, R and S – live on separate floors of a seven storeyed building, but not in the same order.
The ground floor of the building is numbered 1, the floor above it 2 and so on until the topmost floor is numbered 7.Each
person likes different cartoon characters, viz, Doraemon, Pokemon, Ben 10, Tom and Jerry, Ninja Hattori, Looney Tunes
and Chhota Bheem, but not necessarily in the same order.

The person who likes Tom and Jerry lives on floor numbered 4. Only two persons live between P and the one who likes
Tom and Jerry. M does not live on the lowermost floor. M lives on any odd numbered floor below the one who likes Tom
and Jerry.
S lives on an even numbered floor but neither immediately above nor immediately below the floor of M. Only two persons
live between M and the person who likes Chhota Bheem. Only one person lives between N and R. R lives on an even
numbered floor and does not like Tom and Jerry.

Only three persons live between the persons who like Doraemon and Ben 10 respectively. The person who likes
Doraemon live on any floor above the N’s floor. The person who likes Doraemon does not live on the topmost floor.
O does not like Doraemon or Ben 10. The person who likes Ninja Hattori lives on the floor immediately above the floor of
the person who likes Looney Tunes.

1). How many persons live between the floors on which S and P live?
a) Three

www.bankingpdf.com
b) Two
c) Four
d) Five
e) No one

2). Which of the following statements is/are true according to the given information?
a) Q lives on floor numbered 5 and he does not like Tom and Jerry
b) M likes Ninja Hattori and he does not live on floor numbered 4
c) O likes Pokemon and he lives on the topmost floor
d) Only two persons live between the floors of Q and R
e) All the statements are true.

3). Who among the following lives on the floor immediately above the floor of M?
a) N
b) R
c) S
d) O
e) No one

4). Who among the following lives exactly between the floors on which S and N live?

www.bankingpdf.com
a) P
b) R
c) M
d) Q
e) No one

5). Who among the following does like cartoon character Ben 10?
a) R
b) P
c) N
d) Q
e) S

Directions (Q. 6-7): Study the following information and answer the given questions.
G is the mother of R. R is the mother of S. S is the daughter of T. T is the brother of J. J is the mother of L. L is the
daughter of Q. Q is the Son of D.

6). How is T related to G?


a) Son
b) Daughter
c) Son-in-law

www.bankingpdf.com
d) Sister
e) Daughter-in-law

7). How is S related to J?


a) Son
b) Niece
c) Daughter
d) Nephew
e) Cannot be determined

Directions (Q. 8-10): Study the following information and answer the question.
Six people A, B, C, D, E and F obtained different marks in an exam. A obtained more marks than only two people. B
obtained more marks than D but not the highest. F obtained more marks than only C. The one with second highest marks
obtained 480 marks. The one with second lowest marks obtained 350 marks.

8). Which of the following is true with respect to E as per the given information?
a) E obtained less marks than only two people
b) E possibly obtained 500 marks
c) E obtained more marks than D but less than B
d) All the given statements are true

www.bankingpdf.com
e) E definitely obtained 480 marks

9). Who amongst the following obtained third highest marks?


a) D
b) C
c) E
d) F
e) A

10). Who amongst the following possibly obtained 400 marks?


a) A
b) Either C or A
c) D
d) E
e) Either C or E

Answers:
1).c) 2).e) 3).a) 4).d) 5).b) 6).c) 7).b) 8).b) 9).a) 10).b)

www.bankingpdf.com
Directions (Q. 1-5):
Floor No Person Cartoon Character

7 O Pokemon

6 S Chhota Bheem

5 Q Chipmunk

4 N Tom and Jerry

3 M Ninja Hattori

2 R Looney Tunes

1 P Ben 10

www.bankingpdf.com
270-280 Questions :

Directions (Q. 1-5): Read the following information carefully and answer the questions given below.
Seven aspirants—Haresh, Harjeet, Harshad, Hafiza, Hemant, Haridas, and Hemavati—belong to three different cities—
Mumbai, Hyderabad and Bangalore. Three of them write IBPS and two each write SSC and PSC. One IBPS aspirant is
from each of these cities.

Harshad writes SSC and is from Bangalore. Hafiza and Hemavati are the only aspirants from Mumbai and Hafiza writes
PSC. No two aspirants belonging to the same city write the same exam. None of the PSC aspirants is from Bangalore.
Haridas is from Bangalore. Hemant does not same as Haridas city. Harjeet doesn’t write either IBPS or SSC. Hemant
does not write SSC.Harjeet and Haresh is not from Bangalore.

1). Which of the following pairs of aspirants write IBPS?


a) Hemavati, Hemant, Haresh
b) Haresh, Hemant, Hafiza
c) Hemant, Harshad, Hemavati
d) Hemant, Haridas, Haresh
e) Haridas, Hemavati, Hemant

2). Which exam does Haridas write?


a) SSC

www.bankingpdf.com
b) IBPS
c) PSC
d) Data inadequate
e) None of these

3). Which of the following pairs of aspirants write all the three exams?
a) Haresh, Hemant, Harshad
b) Hemant, Haridas, Haresh
c) Hemavati, Hemant, Haresh
d) Hemant, Harjeet, Haresh
e) None of these

4). In which city all the exams are wrote by the aspirants?
a) Mumbai
b) Hyderabad
c) Bangalore
d) Data inadequate
e) None of these

5). Which exam does Haresh write?

www.bankingpdf.com
a) IBPS
b) PSC
c) SSC
d) None of these
e) Data inadequate

6). Statements: All dolls are diaries


Some diaries are pinks
Some pinks are drawers
All drawers are chairs

Conclusions:
(i) Some drawers are diaries
(ii) Some chairs are pinks
(iii) Some pinks are dolls
(iv) Some diaries are dolls

a) All follows
a) Only i and ii follows
b) Only iii and iv follows
c) Only ii and iv follows
d) None follows

www.bankingpdf.com
7). Statements: Some buildings are rivers
Some mountains are both buildings and rivers
Some roads are buildings
All roads are trucks

Conclusions:
(i) Some mountains are roads
(ii) Some buildings are trucks
(iii) Some rivers are roads
(iv) Some trucks are rivers

a) None follows
a) Only i follows
b) Only ii follows
c) Only iii follows
d) Only iv follows

8). Statements: All tables are round


Some bills are round
Some rivers are bills

www.bankingpdf.com
All rivers are conical

Conclusions:
(i) Some rivers are round
(ii) Some bills are conical
(iii) Some rivers are both bills and round
(iv) Some tables are conical

a) None follows
a) Only i follows
b) Only ii follows
c) Only iii follows
d) Only iv follows

9). Statements: All tins are books


All books are waters
Some waters are desks
Some desks are tables

Conclusions:
(i) Some tables are waters
(ii) Some desks are tins

www.bankingpdf.com
(iii) Some waters are tins
(iv) All tins are waters

a) Only i and iii follows


b) Only i and ii follows
c) Only iii and iv follows
d) All follow
e) None of these

10). Statements: Some trains are buses


Some buses are trucks
Some trucks are boats
Some boats are cars

Conclusions:
(i) Some trucks are trains
(ii) Some cars are trucks
(iii) Some boats are buses
(iv) Some boats are trains

a) All follows
a) Only i and ii follows

www.bankingpdf.com
b) Only iii and iv follows
c) Only i and iv follows
d) None follows

Answers:
1).e) 2).b) 3).d) 4).b) 5).c) 6).d) 7).c) 8).c) 9).c) 10).e)

Solution:
Directions (Q. 1-5):

1). IBPS.
Answer: e)
2). Hyderabad.
Answer: b)
3). Haridas, Hemavati, Hemant.
Answer: d)
4). Hemant, Harjeet, Haresh.

www.bankingpdf.com
Answer: b)
5). SSC.
Answer: c)

6.

Answer: d)
7.

Answer: c)

www.bankingpdf.com
8.

Answer: c)
9.

Answer: c)
10.

www.bankingpdf.com
Answer: e)

www.bankingpdf.com
280-290 Questions :

Directions (Q. 1-5): Read the following information carefully and answer the questions given below.
Reena, Ramya, Ravi, Raja, Roshni, Rakesh, Rana and Rani are eight friends travelling by three different names of
Airlines, viz GoAir, IndiGo and SpiceJet. Each Airline is used by at least one female. The Airlines go to three places –
Australia, Japan and Russia. There are three days of travel – Thursday, Friday and Saturday. Each place is visited by at
least one female. On each day at least one female travels. Reena and Raja travel to Australia on the same day by the
same model of car. Rakesh travels on Thursday in GoAir, but not to Russia and not to same place on Raja travel. A
person travelling to Russia uses IndiGo on Thursday. SpiceJet goes only to Japan and only on Friday. Rana travels to
Japan in GoAir on the day before the day on which Reena travels. Roshni travels to Russia but she likes to travel in
GoAir. No male number travels in SpiceJet. Reena travels on the same day on which Ramya travels. Ravi travels to
Japan on Saturday in IndiGo. Rani does not travel on Friday. No person travels to Australia on Thursday. Raja travels by
the same airline by which Rana travels. Roshni and Ravi travel on the same day.

1). Which of the following combinations is/are true?


a) Reena – SpiceJet – Australia - Friday
b) Ravi – IndiGo – Japan - Saturday
c) Roshni – GoAir – Australia - Thursday
d) Rani – GoAir – Japan - Saturday
e) None of these

www.bankingpdf.com
2). Ramya travels in which of the following airlines?
a) IndiGo
b) GoAir
c) SpiceJet
d) Either GoAir or IndiGo
e) None of these

3). Which of the following statements is/are true?


a) Rana goes to Japan on Thursday
b) Raja travels in IndiGo on Friday
c) Rakesh travels in GoAir to Russia
d) Ramya goes to Japan on Saturday
e) None of these

4). Roshni travels on which of the following day?


a) Thursday
b) Saturday
c) Either Thursday or Friday
d) Friday
e) None of these

www.bankingpdf.com
5). Who among the following travel to Japan on Thursday?
a) Rakesh and Rani
b) Rana and Rakesh
c) Ramya and Rana
d) Rani and Ramya
e) None of these

Directions (Q. 6-10): Read the following information carefully and answer the questions given below.
(All the numbers are two digits numbers)
Input : tools 74 32 mark brick 98 93 55 world kind ice 41 77 cost
Step I : 32 tools 74 mark 98 93 55 world kind ice 41 77 cost brick
Step II : 41 32 tools 74 mark 98 93 55 world kind ice 77 brick cost
Step III : 55 41 32 tools 74 mark 98 93 world kind 77 brick cost ice
Step IV : 74 55 41 32 tools mark 98 93 world 77 brick cost ice kind
Step V : 77 74 55 41 32 tools 98 93 world brick cost ice kind mark
Step VI : 93 77 74 55 41 32 98 world brick cost ice kind mark tools
Step VII : 98 93 77 74 55 41 32 brick cost ice kind mark tools world
Step VII is the last step of the above input, as the desired arrangement is obtained.

Input: 79 what rest 27 43 nest door line glass 59 31 unique 66 83

www.bankingpdf.com
6). Which step number is the following output?
43 31 27 79 what rest nest 59 unique 83 66 glass door line
a) Step V
b) Step VI
c) Step IV
d) Step III
e) There is no such step

7). Which word/number would be at 5th position from the right in Step V ?
a) 27
b) 83
c) glass
d) door
e) what

8). How many elements (words or numbers) are there between 'glass' and '43' as they appear in the last step of the
output?
a) One
b) Three

www.bankingpdf.com
c) Four
d) Five
e) Seven

9). Which of the following represents the position of 'what' in the fourth step?
a) Eighth from the left
b) Fifth from the right
c) Sixth from the left
d) Fifth from the left
e) Seventh from the left

10). Which of the following would be step IV?


a) 27 79 what rest 43 nest line glass 59 31 unique 66 83 door
b) 66 59 43 31 27 79 what unique 83 door glass line nest rest
c) 59 43 31 27 79 what rest unique 66 83 door glass line nest
d) 83 79 66 59 43 31 27 door glass line nest rest unique what
e) None of these

Answers:
1).b) 2).c) 3).a) 4).b) 5).b) 6).e) 7).d) 8).b) 9).c) 10).c)

www.bankingpdf.com
Solution:
Directions (Q. 1-5):
Name Airlines Place Day

Reena GoAir Australia Friday

Ramya SpiceJet Japan Friday

Ravi IndiGo Japan Saturday

Raja GoAir Australia Friday

Roshni GoAir Russia Saturday

Rakesh GoAir Japan Thursday

Rana GoAir Japan Thursday

Rani IndiGo Russia Thursday

Directions (Q. 6-10):

www.bankingpdf.com
Input: 79 what rest 27 43 nest door line glass 59 31 unique 66 83

Step I : 27 79 what rest 43 nest line glass 59 31 unique 66 83 door


Step II : 31 27 79 what rest 43 nest line 59 unique 66 83 door glass
Step III : 43 31 27 79 what rest nest 59 unique 66 83 door glass line
Step IV : 59 43 31 27 79 what rest unique 66 83 door glass line nest
Step V : 66 59 43 31 27 79 what unique 83 door glass line nest rest
Step VI : 79 66 59 43 31 27 what 83 door glass line nest rest unique
Step VII : 83 79 66 59 43 31 27 door glass line nest rest unique what

www.bankingpdf.com
290-300 Questions :

Directions (1-5): These questions are based on the following letter / number/ symbol arrangement. Study it carefully and
answer the questions that follow:

W1R%4JE#7MTUI9BH3A$9FQ5DG6USP

1). Which element will be seventh to the right of eighteenth from the right end?
a) A
b) E
c) $
d) #
e) None of these

2). If the positions of E and A are interchanged and similarly the positions of R and U are interchanged then how many
symbols will be there each of which is either preceded or followed by a vowel?
a) None
b) One
c) Two
d) Three

www.bankingpdf.com
e) More than three

3). Four of the following are alike in a certain way based on their positions in the above arrangement and so form a group.
Which is the one that does not belong to that group?
a) RW4
b) 5FG
c) 9QA
d) 3B$
e) 7ET

4). Which element will be eighth to the left of twentieth from the right end?
a) W
b) R
c) %
d) 4
e) None of these

5). If the order of the last 15 elements is reversed which of the following will be fifth to the right of twelfth from the left end?
a) U
b) $

www.bankingpdf.com
c) 3
d) 6
e) None of these

Directions (Q. 6-10): Read the following information carefully and answer the questions given below.
There are seven shops denoted by seven consecutive words M, N, O, P, Q, R and S. All the shops sell different things,
viz – Computers, Vegetables, Oils, Books, Fruits, Toys, and Biscuits. Seven persons are working in these shops, viz
Adarsh, Adesh, Adhik, Adhipa, Adhita, Aditeya and Aditya, But not necessarily in the same order.
Adesh works in shop O and Adhipa does not work at the Computer shop
Aditya does not work in the Biscuit shop. The Biscuit shop is next to the Book shop
Adhita works at the Oils shop
The shop at which Adarsh works is shop R
The shop at which Adhipa works is immediately before the shop at which Aditya works
The Toys shop is immediately after the Computer shop and immediately before the Vegetables shop
Aditeya works in the Book shop, which is shop Q

6). Which of the following combinations is correct?


a) Shop N – Aditya - Computer
b) Shop R – Adesh - Oils
c) Shop O – Adesh - Computer
d) Shop P – Aditeya - Book

www.bankingpdf.com
e) None of these

7). In which of the following shops does Adhipa work?


a) Toys
b) Vegetables
c) Fruits
d) Cannot be determined
e) None of these

8). In which shop does Adhita work?


a) Shop N
b) Shop O
c) Shop R
d) Shop S
e) None of these

9). The shop which is immediately before the Book shop is _____________
a) Biscuit shop
b) Vegetables shop
c) Fruit shop

www.bankingpdf.com
d) Toys shop
e) None of these

10). Which of the following shops is denoted by word R?


a) Vegetables
b) Toys
c) Fruits
d) Computer
e) None of these

Answers:

1).c) 2).d) 3).c) 4).e) 5).a) 6).a) 7).c) 8).d) 9).b) 10).e)

Solution:
1). Seventh to the right of eighteenth from the right end = 18 – 7 = 11th from right end = $
Answer: c)

2). New arrangement


W1U%4JA#7MTRI9BH3E$9FQ5DG6RSP
“U %”, “A #” and “E $”

www.bankingpdf.com
Answer: d)

3). R – 2 = W ; R + 2 = 4
5–2=F;5+2=G
9+2=Q;9-2=A
3–2=B;3+2=$
7–2=E;7+2=T
So 9QA is not belong to that group
Answer: c)

4). Eighth to the left of twentieth from the right end = 8 + 20 = 28th from right end = 1
Answer: e)

5). New arrangement is


W1R%4JE#7MTUI9PSU6GD5QF9$A3HB
Fifth to the right of twelfth from the left end = 5 + 12 = 17 th from left end = U
Answer: a)

Directions (Q. 6-10):

www.bankingpdf.com
Order of shop

www.bankingpdf.com
300-310 Questions :

Directions (Q. 1-5): A coded statement is followed by two conclusions. Decode the statement and conclusion
The symbols have to be decoded as follows:
A (a) B means A is smaller than B.
A (b) B means A is greater or equal to B.
A (c) B means A is greater than B.
A (d) B means A is equal to B.
A (e) B means A is shorter or equal to B.
a) If only conclusion I is true
b) If only conclusion II is true
c) If conclusion I or II is true
d) If neither conclusion I nor II is true
e) If both conclusions are true

1). Statement : V (a) W, W (b) L, L(c)Y


Inference: I. L (c) V
II. V (a) Y

2). Statement : G (b) H, H (d) O, O (a) F


Inference: I. G (c) F
II. G (a) F

www.bankingpdf.com
3). Statement : D (e) K, K (c) P, P (b) Q
Inference: I. K (b) Q
II. D (e) Q

4). Statement : S (c) R,R (d) T, T (a) V


Inference: I. V (a) S
II. V (a) R

5). Statement : N (a) L, L(d) P, P (e) H


Inference: I. L (e) N
II. H (c) N

Directions (Q. 6-10): A word and number arrangement machine when given an input line of words and numbers
rearranges them following a particular rule in each step. The following is an illustration of input and rearrangement?
Input : go now 52 38 17 for again 65
Step I: 65 go now 52 38 17 for again
Step II: 65 again go now 52 38 17 for
Step III: 65 again 52 go now 38 17 for

www.bankingpdf.com
Step IV: 65 again 52 for go now 38 17
Step V: 65 again 52 for 38 go now 17
Step VI : 65 again 52 for 38 go 17 now
Step VI is the last step of the rearrangement.
As per the rules followed in the above steps, find out in each of the following questions the appropriate step for the given
input.

Input: home turn 39 24 86 44 roll over


6). Which of the following steps will be the last?
a) X
b) IX
c) VIII
d) VII
e) None of these

7). Step III of input is : 94 car 86 window shut 52 31 house


Which of the following is definitely the input?
a) 94 car window 86 shut 52 31 house
b) 86 window 94 car shut 52 31 house
c) Car shut window 86 53 31 house 94
d) Cannot be determined

www.bankingpdf.com
e) None of these

8). Step IV of an input is : 58 box 47 dew 15 21 town pot


Which of the following steps will be the last?
a) VII
b) VI
c) VIII
d) IX
e) None of these

9). Input: show 51 36 new far 81 46 goal


Which of the following steps will be the last but one?
a) VII
b) VIII
c) VI
d) V
e) None of these

10). Input : buy win task 52 38 43 door 12


Which of the following will be step IV?

www.bankingpdf.com
a) 52 buy 43 door 38 task 12 win
b) 52 buy 43 door 38 task win 12
c) 52 buy 43 door task win 38 12
d) There will be no such step
e) None of these

Answers:
1)a 2)b 3)d 4)d 5)e 6)e 7)d 8)b 9)c 10)e

Solution:
1). V < W, W ≥ L, L > Y
or, W > V, W ≥ L, L > Y
or, W =L > V, W > L > V, W = L > Y
there is no clear indication between V and Y.
Answer: a)

2). G ≥ H, H = O, O < F
or, G ≥ H, H = O, F > O
or, G > H, G = H = O, F > G = H = O.
Answer: b)

www.bankingpdf.com
3). D ≤ K, K > P, P ≥ Q
or K ≥ D, K > P, P ≥ Q
or, K > D, K = D > P, P ≥ Q
or, K > P ≥ Q, K = D > P ≥ Q
thus, I and II both are not true
Answer: d)

4). S > R, R = T, T < V


or, S > R =T, V > R = T
thus, I and II both are not true
Answer: d)

5). N < L, L =P, P ≤ H


or, L>N, L = P-, H≥P
or, L = P > N, H ≥ P = L
Answer: e)

6). Input : home turn 39 24 86 44 roll over

www.bankingpdf.com
Step I: 86 home turn 39 24 44 roll over
Step II: 86 home 44 turn 39 24 roll over
Step III: 86 home 44 over turn 39 24 roll
Step IV: 86 home 44 over 39 turn 24 roll
Step V: 86 home 44 over 39 roll turn 24
Step VI : 86 home 44 over 39 roll 24 turn
Answer: e)

7). input of the given sequence cannot be determined, as we don’t know about the initial position of the elements
Answer: d)

8). Step V : 58 box 47 dew 21 15 town pot


Step VI : 58 box 47 dew 21 pot 15 town
hence step VI is the last step
Answer: b)

9). Step I: 81 show 51 36 new far 46 goal


Step II: 81 far show 51 36 new 46 goal
Step III: 81 far 51 show 36 new 46 goal
Step IV: 81 far 51 goal show 36 new 46

www.bankingpdf.com
Step V: 81 far 51 goal 46 show 36 new
Step VI : 81 far 51 goal 46 new show 36
Step VII: 81 far 51 goal 46 new 36 show
hence, step VI is the last but one step.
Answer: c)

10). Input – buy win task 52 38 43 door 12


IVth step 52 buy 43 door 38 win task 12
Step I: 52 buy win task 38 43 door 12
Step II: 52 buy 43 win task 38 door 12
Step III: 52 buy 43 door win task 38 12
Step IV: 52 buy 43 door 38 win task 12
Answer: e)

www.bankingpdf.com
310-320 Questions :

Directions (Q. 1-5): In each question below are given two statements followed by two conclusions numbered I and II. You
have to take the two given statements to be true even if they seem to be at variance with commonly known facts. Read
the conclusions and then decide which of the given conclusions logically follows from the given statements disregarding
commonly known facts.
a) If only conclusion I follows.
b) If only conclusion II follows.
c) If either I or II follows.
d) If neither I nor II follows.
e) If both I and II follow.

1). Statements : All humans are civilized.


Some animals are mammals.
All humans are mammals.
Conclusion:
I.All civilized are mammals.
II.Some animals are civilized.

2). Statements: Smita is a girl.

www.bankingpdf.com
Some girls are beautiful.
All girls are student.
Conclusion:
I.Smita is an student.
II.Some students are beautiful.

3). Statements: All rivers are streams.


Some streams are wells.
No well is sea.
Conclusion:
I.Some rivers are wells.
II.Some seas are not stream.

4). Statements: Some kites are Bird.


All animals are kites.
Some animals are insects.
Conclusion:
I.No bird is insect.
II.Some insect are kites.

www.bankingpdf.com
5). Statements: Some doctors are professional.
Raj is a professional.
Raj is not young.
Conclusion:
I.Some doctors are young.
II.Some young doctors are not professional.

Directions (Q.6-10): Read the following information carefully and answer the questions, which follow:
‘P÷ Q’ means ‘P is son of Q’.
‘P × Q’ means ‘P is sister of Q’.
‘P + Q’ means ‘P is brother of Q’.
‘P-Q’ means ‘P is mother of Q’.

6). How is T related to S in the expression ‘T × R + V ÷ S’?


a) Sister
b) Mother
c) Aunt
d) Uncle
e) None of these

www.bankingpdf.com
7). How is T related to S in the expression ‘T × R ÷ V - S’?
a) Father
b) Sister
c) Daughter
d) Aunt
e) None of these

8). How is S related to T in the expression ‘T + R – V+ S’?


a) Uncle
b) Nephew
c) Son
d) Cannot be determined
e) None of these

9). Which of the following means that ‘S is the husband of T’?


a) T × R –V + S
b) T-R ÷V × S
c) T – R + V ÷ S
d) T ÷ R × V + S
e) None of these

www.bankingpdf.com
10). How is V related to T in the expression ‘T ÷ R + V × S’?
a) Aunt
b) Nephew
c) Niece
d) Uncle
e) None of these

Answers:
1)d 2)e 3)d 4)b 5)d 6)e 7)b 8)d 9)c 10)a

Explanation:
1).

www.bankingpdf.com
Answer: d)

2).

Answer: e)

3).

www.bankingpdf.com
Answer: d)

4).

www.bankingpdf.com
Answer: b)

5).

None of the conclusions are valid. Hence,


Answer: d)

6).

m-male, f-female
Hence, T is daughter of S.
Answer: e)

www.bankingpdf.com
7).

m-male, f-female
Hence, T is sister of S.
Answer: b)

8).

m-male, f-female
S’s sex is not given.
Answer: d)

www.bankingpdf.com
9).

m-male, f-female
Answer: c)

10).

Hence, V is Aunt of T.
Answer: a)

www.bankingpdf.com
320-330 Questions :

1). Tarun moves 10 m towards north from point A, Then he takes a left and moves 10 m and reaches a point B. At point B
he climbs some stairs to reach a height of 5 m, after reaching the top he turns towards his left and proceeds to a distance
of 10m and reaches a point D. Find the distance between A and D.
a) 5
b) 10√5
c) 5√5
d) 20
e) 25

2). Sunil goes 5m in the north-east direction, then he turns 135 degrees clockwise & moves 4m, then he turns
Towards his right & walks 3m, then he turns 45 degrees clockwise & moves 5m, Again he turns 135 degrees
anticlockwise & moves 4m & reaches a point X. In which direction is X located with respect to the starting point.
a) North – west
b) North
c) West
d) South – west
e) South – east

www.bankingpdf.com
Directions (Q. 3-5): In a particular system different calculations are written as following:
(i)a & b # c means product of a and b has been added to the square of c.
(ii) a * b % c means b has been divided by c and result has been subtracted from a.
(iii) a © b @ c means the square of b has been multiplied to the square root of c and the result has been added to a.
(iv)a ! b € c means a has been multiplied to b and the result has been divided by c.
In each of the questions, the value that is to come at (?) has to be found.

3). 76 *7 6 8 % 12 =x
24! X € 36 = ?
a) 16
b) 8
c) 18
d) Cannot be calculated
e) None of these

4). 15 © 3 @ 25=p
P * 46 % 2 = ?
a) 60
b) 33
c) 45
d) 83

www.bankingpdf.com
e) 37

5). 8 © 4 @ 49= m
M * 204 % 17 = ?
a) 132
b) 106
c) 112
d) 108
e) None of these

Directions (Q.6-10): Study the following information carefully and answer the questions given below.
Seven colleagues A,H,J,S,V,D and R work in four different departments of an organization Marketing, Finance, HR and
IT. Not more than two work in any department. Each of them is wearing different coloured shirt, which is White, Blue,
Cream, Rosy, Black, Yellow, and Pink, not necessarily in that order. A works with J in the Marketing department and is not
wearing white coloured shirt. V is wearing Yellow coloured shirt and does not work in the Finance department. H works
with S in the same department and is wearing Black shirt. Only D works in the HR department. No one in the Marketing or
IT department is wearing Cream shirt. J is wearing Rosy shirt. The one who is wearing the Pink shirt is working in the
Finance department.

6).Who work in the Finance department?

www.bankingpdf.com
a) V,R
b) A,H
c) S,H
d) Data inadequate
e) None of these

7).Who is wearing Cream shirt?


a) R
b) A
c) S
d) D
e) None of these

8).What coloured shirt is R wearing?


a) Black
b) White
c) Cream
d) Data inadequate
e) None of these

www.bankingpdf.com
9).In which department is R working?
a) IT
b) Marketing
c) Finance
d) HR
e) Data inadequate

10).what coloured shirt is A wearing?


a) Cream
b) White
c) Blue
d) Data inadequate
e) None of these

Answers:
1)c 2)c 3)b 4)e 5)d 6)c 7)d 8)b 9)a 10)c

Explanation:
1).

www.bankingpdf.com
Using the Pythagoras theorem RD2 + AR2 =AD2
AD2 = 102+52 =175.
Hence AD=√125=5√5.
Answer: c)

2).

From the following diagram we see that X is located in west direction with respect to starting point.

www.bankingpdf.com
Answer: c)

3).From 1st expression, x = 76 – (768/12)=76-64 =12


From 2nd expression,(24*12)/36=8
Answer: b)

4). From 1st expression, p = 15 + (3*3*5) = 60


From 2nd expression, 60 – (46/2)=37
Answer: e)

5). From 1st expression, m=8 + (4*4*7) = 120


From 2nd expression,120 – (204/17) = 108
Answer: d)

Directions (Q. 6-10):


Person Dept Shirt

A Mkt Blue

www.bankingpdf.com
H F Black

J Mkt Rosy

S F Pink

V IT Yellow

D HR Cream

R IT White

6). H and S work in the Finance department


Answer: c)

7). D is wearing Cream shirt


Answer: d)

8). R is wearing White shirt.


Answer: b)

www.bankingpdf.com
9). R is working in the IT department
Answer: a)

10). A is wearing blue shirt


Answer: c)

330-340 Questions :

Direction (Q. 1-5): Twelve wards N, O, P, Q, R, S, T, U, V, W, X and Y of a hospital are in two parallel buildings. Building
1 is facing west and building – 2 is facing east. Wards P, Q, R, S, T and U are in building 1 and wards N, O, V, W, X, and
Y are in building 2. Each wards of building 1 is exactly opposite the other ward in building 2.
i. Ward V, which is at one of the ends of the building, is second to the left of ward Y.
ii. There are two wards between ward N and O.
iii. There is only one ward between ward S and T.
iv. Ward U is third to the right of Q.
v. Ward U is not opposite either V and Y.
vi. Ward S and T are not opposite V. ward P is adjacent to ward S.
vii. Ward W, which is not opposite T. is not adjacent to N.

www.bankingpdf.com
1). Which ward opposite to Q?
a) N
b) Y
c) W
d) X
e) O

2). How many wards are there between wards R and S?


a) Four
b) Three
c) Two
d) One
e) None of this

3). Ward R is related to ward X in the same way as ward U is related to O, based on the given arrangement. Who among
the following is Q related to, following the same pattern?
a) N
b) V
c) W
d) Y
e) None of this

www.bankingpdf.com
4). Which of the following wards are at the ends of the building?
a) V, O
b) C, Q
c) V, W
d) R, T
e) None of this

5). Which is following statements is true about P?


a) Ward Y is opposite ward P.
b) Ward S is not adjacent to ward P.
c) Ward N is not neighbor of that ward which is opposite P.
d) Ward P is 2nd from the right end
e) Ward O is opposite that ward which is 3rd to the left of ward P.

6). Statement : K > I ≥ T ≥ E; O < R < K


Conclusions : I. R < E II. O < T
a) Neither conclusion I nor II follows
b) Both the conclusion I and II follow
c) Only conclusion II follows

www.bankingpdf.com
d) Either conclusion I or II follows
e) Only conclusion I follows

7). Statement : C < L < O = U = D ≥ S > Y


Conclusions : I. O > Y II. C < D
a) Neither conclusion I nor II follows
b) Both the conclusion I and II follow
c) Only conclusion I follows
d) Only conclusion II follows
e) Either conclusion I or II follows

8). Statement : K ≥ L > M ≥ N


Conclusions : I. N ≤ K II. N < K
a) Both the conclusion I and II follow
b) Neither conclusion I nor II follows
c) Either conclusion I or II follows
d) Only conclusion I follows
e) Only conclusion II follows

9). Statement : Z ≥ Y = W ≤ X

www.bankingpdf.com
Conclusions : I. W < Z II. W = Z
a) Only conclusion I follows
b) Only conclusion II follows
c) Neither conclusion I or II follows
d) Either conclusion I nor II follows
e) Both the conclusion I and II follow

10). Statement : B > A > S < I > C > L > Y


Conclusions : I. B > L II. A > Y
a) Only conclusion I follows
b) Only conclusion II follows
c) Either conclusion I or II follows
d) Neither conclusion I nor II follows
e) Both the conclusion I and II follow

Answer:
1) e 2)c 3)a 4)e 5) a 6)a 7) b 8) e 9)d 10)d

SOLUTIONS:
Directions (Q. 1-5):

www.bankingpdf.com
6). Statement : K > I ≥ T ≥ E; O < R < K
On combining both the statements, we get
O<R<K>I≥T≥E
Conclusions : I. R < E (false)
II. O < T (false)
Thus, neither conclusion I nor II follows
Answer: a)

7). Statement : C < L < O = U = D ≥ S > Y


Here statements are already combined
Conclusions : I. O > Y (true)
II. C < D (true)

www.bankingpdf.com
Thus, both conclusion I and II follow
Answer: b)

8). Statement : K ≥ L > M ≥ N


Here statements are already combined
Conclusions : I. N ≤ K (false)
II. N < K (true)
Thus, only conclusion II follows
Answer: e)

9). Statement : Z ≥ Y = W ≤ X
Here statements are already combined
Conclusions : I. W < Z (may be true)
II. W = Z (may be true)
Thus, either conclusion I or II follows
Answer: d)

10). Statement : B > A > S < I > C > L > Y


Here statements are already combined
Conclusions : I. B > L (false)

www.bankingpdf.com
II. A > Y (false)
Thus, neither conclusion I nor II follows
Answer: d)

340-350 Questions :

Direction (1 – 5):
Seven conferences on Marketing, Finance, HR, Real Estate, Hospitality, Management, and Banking were scheduled
to be held in Mumbai, Pune, Delhi, Hyderabad, Indore, Bhopal and Chennai on one day in a week starting from
Monday and ending on Sunday.
Conference on Hospitality was held in Hyderabad on Friday. Only one conference was held between conference on
hospitality and Finance. Conference on Management was held immediately after HR but immediately before
Banking. Conference on Management was not held in Indore. The conference held on Monday was held in Delhi.
Only one conference was held between conference on Banking and the conference in Mumbai. The conference on
HR was not held in Mumbai. Conference in Pune was held immediately before the conference in Indore. Conference
on Real Estate was not held on Monday. Conference in Chennai was not held after conference in Mumbai.

1). If Delhi is related to Pune and Banking is related to Real Estate in a certain way, then to which of the following
would Indore be related to, following the same pattern?
a) Bhopal

www.bankingpdf.com
b) Chennai
c) Mumbai
d) Delhi
e) None of these
2). In which of the following cities was conference on Banking held?
a) Chennai
b) Mumbai
c) Delhi
d) Bhopal
e) Indore
3). How many conferences were held between conferences on Marketing and Real Estate?
a) None
b) One
c) Two
d) Three
e) Four
4). On which of the following days was the conference of HR held?
a) Monday
b) Tuesday
c) Wednesday
d) Thursday

www.bankingpdf.com
e) Friday
5). Which of the following conferences was held on Monday?
a) Banking
b) Management
c) HR
d) Marketing
e) Can’t be determined

Direction [6 – 8]:
There are five bottles A, E, I, O and U arranged on a table but not necessarily in the same order. Bottle E is to the
north of bottle O. Bottle I is to the south of bottle E. Bottle A and bottle U are in the east and west respectively of
bottle I.
6). In which direction is bottle O with respect to bottle I?
a) South
b) North
c) Either 1) or 2)
d) East
e) North – East
7). If bottle I is exactly between E and O then in which direction is bottle A with respect to bottle O?
a) North

www.bankingpdf.com
b) North – east
c) South – east
d) South
e) North – east
8). In which direction is bottle U with respect to bottle E?
a) South
b) South – east
c) West
d) South – west
e) None of this

Direction [9 – 10]:
A is 10m to the north of Z. who is to the northwest of Y. Z is 15m to the west of B. C is 10m to the east of Y, who is
10m to the south of B.
9). In which direction is Y with respect to A?
a) South
b) Southeast
c) Southwest
d) West
e) None of this

www.bankingpdf.com
10). What is the total distance between point A and Y?
a) 35m
b) 25m
c) 30m
d) 20m
e) None of this
Solutions:
1) c 2) e 3)e 4)b 5)d 6) c 7) b 8) d 9) b 10) b

Explanation:
(1 to 5):

(6 to 8):

www.bankingpdf.com
(9 to 10):

www.bankingpdf.com
350-360 Questions :

Directions (Q. 1-3): A family consists of six members A, B, C, S, T and U. B is the son of C but C is not Mother of B. A and
C are a married couple, T is the brother of C. S is the daughter of A. U is the brother of A.
1).How many children does A have?
a) One
b) Two
c) Three
d) Four
e) Indeterminable

2).How many female members are there in the family?


a) One
b) Two
c) Three
d) Four
e) Indeterminable

3).How is B related to S?
a) Husband

www.bankingpdf.com
b) Father
c) Brother
d) Uncle
e) Indeterminable

Directions (Q. 4-5): Surya starts his journey from his school & moves 10m towards east. Then he takes a left & moves 2m,
then takes a right and moves 22m distance and stops. Then he takes a left & moves 24m distance and reaches a joint X,
Then he turns towards his right and walks 26m distance then again he takes a right and walks 28m distance and reaches a
point Y.

4). How much distance has he travelled from his school?


a) 352m
b) 256m
c) 370m
d) 252m
e) 332m

5).In which direction point Y is located with respect to his school.


a) North-east
b) East

www.bankingpdf.com
c) South-east
d) South
e) South-West

Directions (Q. 6-8): A coded statement is followed by two conclusions. Decode the statement and conclusion
The symbols have to be decoded as follows:
A(a) B means A is smaller than B.
A (b) B means A is greater or equal to B.
A (c)B means A is greater than B.
A (d) B means A is equal to B.
A (e) B means A is shorter or equal to B.
a) If only conclusion I is true
b) If only conclusion II is true
c) If conclusion I or conclusion II is true
d) If neither conclusion I not conclusion II is true
e) If both conclusions are true

6). Statement :D(e)K, K(c)P, P(b)Q


Inference: I. K(b)Q
II.D(e)Q

www.bankingpdf.com
7). Statement: S(c)R, R(d)T, T(a)V
Inference: I. V(a)S
II.V(a)R

8). Statement : N(a)L , L(d)P, P(e)H


Inference: I. L(e)N
II.H(c )N

Directions (Q. 9-10): In each of the following questions the symbols $, % and & have been used having their meaning as
M @ N means M is greater than N
M # N means M is smaller than N
M $ N means M is not greater than N
M % N means M is not shorter than N
M & N means M is equal to N
Assuming the statement given in each of the following question as true, deduce, which of the given five alternatives is
definitely true?

9). Statement :2A @ 4C, 4A & 6B

www.bankingpdf.com
a) C & B
b) C % B
c) C # B
d) C @ B
e) C $ B

10). Statement : 4B & 6C,4A @ 2C


a) B # A
b) B @ A
c) B % A
d) B $ A
e) B & A

Answers:
1).b 2).b 3).c 4).a 5).c 6).d 7).d 8).e 9).c 10).a

Explanation:

www.bankingpdf.com
Directions (Q. 1-3): B is the son of C but C is not the mother. So, C is the father of B. A is married to C. So, A is the wife of
C and the mother of B. S is the daughter of A and hence of C and so she is the sister of B. T is the brother of C and U is
the brother of B.

1). Clearly, B is the son of A and S is the daughter of A. So, A has two children.
Answer: b)

2). There are two females only-mother A and daughter S.


Answer: b)

3). S is the sister of B who is male. So, B is brother of S.


Answer: c)

Directions (Q. 4-5): Refer to the following figure.

www.bankingpdf.com
4). Total distance he travelled from his house = 10+2+4+16+64+256=352m
Answer: a)

5). Point Y located in South-east direction with respect to his school.


Answer: c)

6).D ≤ K, K > P, P≥ Q
Or, K≥D, K > P, P ≥ Q
Or, K > D, K = D> P, P ≥ Q
OR, K > P ≥ Q , K =D > P ≥ Q
Thus, I and II both are not true.
Answer: d)

www.bankingpdf.com
7).S > R , R = T, T<V
Or, S > R =T, V > R = T
Thus, conditions I and II both are not true
Answer: d)

8).N<L, L = P, P≤ H
Or, L>N, L= P, H≥ P
Or,L = P > N,H≥P = L
Hence,
Answer: e)

9).2A > 4C, 4A =6B => 4A > 8C,4A = 6B


Or,4A = 6B > 8C
i.e. B > C or C<B. Thus, C#B.
Answer: c)

10).4B =6C, 4A > 2C => 4B = 6C,12A > 6C


Or,12A > 4B dr. B < A. thus, B # A

www.bankingpdf.com
Answer: a)

360-370 Questions :

Directions(1-5): Nine people L, M, N, O, P, Q, R, S and T stay in a building not necessarily in the same order. The
building has nine floors and only one person stays on one floor. All of them own one each, and each car is of a
different car, i.e. grey, white, black, yellow, green, red, orange and pinknot necessarily in the same order. The ground
floor is numbered 1, the floor above it, number 2 and so on and the topmost floor is numbered 9. S owns a black
coloured car and stays on even numbered floor. L stays on even numbered floor below the floor on which S stays.
The one who own an orange coloured car stays on 4rth floor. P stays on the second floor and owns the white
coloured car. The one who owns a pink coloured car stays on the third floor. L does not own a green coloured car.
There are two floors between the floors on which the people owning the black and red coloured cars stay. N owns a
grey coloured car. There are three floors between the floors on which N and R stays. O stays on a floor immediately
above T’s floor. There is one floor between the floors on which Q and R stay. Q does not own the pink coloured car.
The one who owns the blue car stays on the top–most floor. Q does not stay on the ground floor.

1) How many floors are there between the floor on which Q stays and the floor on which N stays?
a) Six
b) Eight
c) One
d) None

www.bankingpdf.com
e) More than three
2) Which of the following is true as per the given information?
a) M stays on a floor immediately below the floor on which S stays
b) R stays on a floor immediately above the floor on which L stays
c) Q stays on eight floor
d) O owns the orange coloured car and stays on 4rth floor
e) None is true
3) On which of the following floor does M stay?
a) 7th
b) 6th
c) 9th
d) 1st
e) None of these
4) Who amongst the following stays on the top–most floor?
a) M
b) O
c) R
d) S
e) None of these
5) L owns a car of which of the following colours?
a) Pink

www.bankingpdf.com
b) Blue
c) Yellow
d) Orange
e) None of these
6) How many floors are there between the floor on which S stays and the floor on which red coloured car stays?
a) 3
b) 2
c) 1
d) 4
e) More than 4

Directions (Q. 7-12):


Eight candidates A, B, C, D, P, Q, R and S have applied for IBPS exam for different banks, viz BOB, BOI, Dena
Bank, UBI, OBC, Indian Bank, Corporation Bank and UCO Bank but not necessarily in the same order. There are five
male and three female candidates in this group. Each male and each female has applied from his/her city, viz
Chennai, Delhi, Mumbai, Kochi, Patna, Raipur, Gwalior and Agra. No male has applied from Raipur and Kochi.
P has applied for Indian Bank from Agra. The one who has applied for Corporation Bank is neither from Mumbai nor
from Kochi. B has applied for OBC. His sister C has applied for Dena Bank from Chennai. The one who belongs to
Delhi has applied for BOB. The one who has applied from Mumbai is not a female. D has applied for UBI and her
friend has applied for BOB. Q is from Gwalior and has not applied either for Corporation Bank or for UCO Bank. The

www.bankingpdf.com
one who has applied for UCO Bank has applied neither from Patna nor from Mumbai. S has applied from Kochi. A
does not apply from Patna.

7). Which of the following groups is a group of female applicants?


a) B, D, C
b) D, Q, S
c) C, D, S
d) P, Q, R
e) None of these
8). Who among the following has applied from Ranchi?
a) P
b) D
c) S
d) C
e) None of these
9). Which of the following combinations is true?
a) A– UBI – Ranchi
b) R – Corporation Bank – Mumbai
c) C – UCO Bank – Chennai
d) B – OBC – Mumbai
e) None of these

www.bankingpdf.com
10). Who among the following has applied for BOB?
a) A
b) S
c) D
d) Q
e) None of these
11). B belongs to which of the following cities?
a) Delhi
b) Ranchi
c) Mumbai
d) Gwalior
e) None of these
12). Four of the five are alike in a certain way and hence form a group. Which is the one that does not belong to that
group?
a) P
b) R
c) A
d) C
e) Q

Answers:

www.bankingpdf.com
1).e) 2).d) 3).c) 4).a) 5).c) 6).b) 7).c) 8).b) 9).d) 10).a) 11).c) 12).d

Solutions
Directions(1-6): The tabulated is given below to show the arrangements – :
Floor Colour of the person Person
9 Blue M
8 Black S
7 Green Q
6 Yellow L
5 Red R
4 Orange O
3 Pink T
2 White P
1 Grey N

Direction (7 –
12): Based on the information given figure is drawn below

www.bankingpdf.com
370-380 Questions :

1). In a certain code language, ’col tip mot’ means ‘singing is appreciable’; ’mot baj min’ means ‘dancing is good’ and ‘tip
nop baj’ means ‘singing and dancing’, which of the following means ‘good’ in that code language?
a) Not
b) Min
c) Baj
d) Indeterminable
e) None of these

www.bankingpdf.com
2).In certain code language, ‘pre nat bis’ means ‘smoking is harmful’ ; ‘vog dor nat’ means ‘avoid harmful habit’ and ‘dor
bis yel’ means ‘please avoid smoking’, which of the following means ‘habit’ in that code language?
a) Vog
b) Nat
c) Dor
d) Bis
e) None of these

3).If in a certain language MYSTIFY is coded as NZTUJGC, how is NEMESIS coded in that language?
a) MIDIHRDR
b) OFNFTJT
c) ODNHTDR
d) PGOKUGU
e) OFFNJTT

4).If in certain language FORGE is coded as FPTJI, how is CULPRIT coded in that language?
a) CSJNPGR
b) CVSQMTU
c) CVNSVNZ

www.bankingpdf.com
d) CXOSULW
e) CVNVSNZ

5).If SHAHRUKH could be given the code number 94, what code can be given to AMIRABH?
a) 104
b) 94
c) 45
d) 54
e) 72

Directions (Q.6-10) : Study the following information to answer the given questions:
In a group of 5, each person has an exclusive and different preference (has/like) for a pen, a watch and a car. Pen
preferences are Parker, Lamy, Pointer, Lexi and Cello. Car preferences are Innova, Baleno, Alto, Scorpio and Ciaz.
Watch preferences are Timex, Titan, Fastrack, Samay and Citizen.
Suganth has Scorpio and Parker but does not prefer among watches-Titan or Fastrack. The one who has Baleno, likes
Fastrack. Dhanya has preference for Ciaz, Cello and Citizen. Amir has preference for Lamy and Timex. Renu prefers
Innova and Lexi. Vimal’s preference for a watch is not Titan.

6).Which watch is Suganth’s preference?


a) Titan

www.bankingpdf.com
b) Fastrack
c) Samay
d) Cannot be determined
e) None of these

7).Which pen is Vimal’s preference?


a) Lamy
b) Pointer
c) Lexi
d) Cannot be determined
e) None of these

8).Which watch is Vimal’s preference?


a) Samay
b) Fastrack
c) Timex
d) Cannot be determined
e) None of these

9).Who’s preference is Baleno?

www.bankingpdf.com
a) Vimal
b) Amir
c) Renu
d) Cannot be determined
e) None of these

10).Which watch is Renu’s preference?


a) Samay
b) Fastrack
c) Titan
d) Cannot be determined
e) None of these

Answers:
1).b) 2).a) 3).b) 4).c) 5).d) 6).c) 7).b) 8).b) 9).a) 10).c)

Explanation:
1).In the first and second statements, the common code word is ‘mot’ and the common word is ‘is’. So, ‘mot’ means ‘is’.
Similarly ‘baj’ and ‘dancing’ in the second and third statements. Thus, in the second statement, ‘min’ means ‘good’.
Answer: b)

www.bankingpdf.com
2). In the first and second statements, the common code word is ‘nat’ and the common word is ‘harmful’. So, ‘nat’ means ‘
harmful’. Similarly ‘dor’ and ‘avoid’ in the second and third statements. Thus, in the second statement, ‘vog’ means ‘habit’.
Answer: a)

3).Each letter in the word MYSTIFY is moved one step forward to obtain the corresponding letter of the codes So, in
NEMESIS, N will be coded as O, E as F and so on thus the code becomes QFNFTJT.
Answer: b)

4).Here, the first letter in the word FORGE remains same and second, third, fourth and fifth letters are moved one, two,
three, and four steps forward.
Answer: c)

5). S + H + A + H + R + U + K + H =19 + 8 + 1 + 8 + 18 + 21 +11 + 8 = 94


Thus, A + M + I+ T +A + B + H =1 + 13 + 9 + 20 + 1 + 2 + 8 =54
Answer: d)

Directions (Q. 6-10):

www.bankingpdf.com
Person Pen Watch Car

Suganth Parker Samay Scorpio

Dhanya Cello Citizen Ciaz

Amir Lamy Timex Alto

Renu Lexi Titan Innova

Vimal pointer Fastrack Baleno

6). Suganth likes Samay.


Answer: c)

7). Vimal Prefers pointer.


Answer: b)

8). Vimal prefers Fastrack watch


Answer: b)

www.bankingpdf.com
9). Vimal prefer Baleno
Answer: a)

10). Renu prefers Titan


Answer: c)

380-390 Questions :

Directions (1-5): These questions are based on the following letter / number/ symbol arrangement. Study it carefully and
answer the questions that follow:
5DGE«79$F16R%LIAJ3B#4@KP8UM2

1). How many such numbers are there in the above arrangement each of which is immediately followed by a consonant
but not immediately preceded by a vowel?
a) None
b) One
c) Two
d) Three
e) More than three

www.bankingpdf.com
2). Four of the following five are alike in a certain way on the basis of their positions in the above arrangement and so
form a group. Which is the one that does not belong to the group?
a) $9F
b) R6%
c) 8PU
d) #B4
e) 3BJ

3). If all the consonants are removed from the above arrangement which element will be fifth to the left of seventh from the
right?
a) 6
b) $
c) %
d) 1
e) None of these

4). If all the symbols are removed from the above arrangement which element will be third to the left of thirteenth from the
left?
a) L
b) R
c) 6
d) %

www.bankingpdf.com
e) None of these

5). Which element will be fifth to the right of ninth from the right end if all the numbers are removed from the above
arrangement?
a) K
b) @
c) P
d) #
e) None of these

Directions (Q. 6-10): In the following questions the symbol ×, ∂, %, ©, @, are used with the following illustrations.

P % Q means P is not smaller than Q


P © Q means P is neither smaller than nor equal to Q
P x Q means P is neither greater than nor equal to Q
P ∂ Q means P is not greater than Q
P @ Q means P is neither greater than nor smaller than Q

6). Statement – R ∂ K ; K x M ; M @ J
Conclusions:

www.bankingpdf.com
I. J©K
II. M © R
III. R x J

a) Only I & II follows


b) Only II & III follows
c) Only I & III follows
d) All I, II & III follows
e) None of these

7). Statements – Z@M; M©K; KxF


Conclusions:
I. F©Z
II. K x Z
III. F © M

a) None follows
b) Only I follow
c) Only II follows
d) d) Only III follows
e) Only II & III follows

www.bankingpdf.com
8). Statements – V % H ; H @ F ; F ∂ E
Conclusions:
I. F@V
II. F x V
III. E % H

a) Only either I or II follows


b) Only III follows
c) Only I & II follows
d) All I, II & III follows
e) Only either I or II and III

9). Statements – W © T ; T ∂ N ; N % D
Conclusions:
I. DxT
II. W © N
III. D @ T

a) None follows
b) Only I follow

www.bankingpdf.com
c) Only II follows
d) d) Only III follows
e) Only I & II follows

10). Statements – Y@G; G©K; KxR


Conclusions :
I. R©Y
II. K x Y
III. R © G

a) None follows
b) Only I follow
c) Only II follows
d) d) Only III follows
e) Only II & III

Answers:
1). c) 2). e) 3). d) 4). b) 5). a) 6. d) 7. c) 8. e) 9. a) 10. c)

Explanation:

www.bankingpdf.com
1). 1 6 R and J 3 B
Answer: c)

2). $9F this symbols and numbers are arranged in preceded by and followed by manner
So “3BJ” is not arranged in the above manner
Answer: e)

3). New arrangement


5E«79$16%IA3#4@8U2
Fifth to the left of seventh from the right = 5 + 7 = 12th from right = 1
Answer: d)

4).
5DGE79F16RLIAJ3B4KP8UM2
Third to the left of thirteenth from the left = 13 – 3 = 10th from the left = R
Answer: b)

5).
DGE«$FR%LIAJB#@KPUM

www.bankingpdf.com
Fifth to the right of ninth from the right end = 9 – 5 = 4th from right end = K
Answer: a)

Directions (Q. 6-10):


P % Q means P is not smaller than Q ----------------- [ P≥ Q ]
P © Q means P is neither smaller than nor equal to Q --- [ P > Q]
P x Q means P is neither greater than nor equal to Q --------- [ P < Q ]
P ∂ Q means P is not greater than Q ---------- [ P ≤ Q ]
P @ Q means P is neither greater than nor smaller than Q------- [ P=Q ]

6). R ≤ K < M = J
J>K
M> R
R<J
All follows
Answer: d)

7). Z = M > K < F


F > Z not follows
K< Z follows

www.bankingpdf.com
F> M not follows
Answer: c)

8). V ≥ H = F ≤ E
F=V
F< V
E≥H
Either I and II and III follows
Answer: e)

9). W > T , N ≥ T , N ≥ D
D<T
W> N
D=T
None follows
Answer: a)

10). Y = G > K < R


R > Y not follows
K< Y follows

www.bankingpdf.com
R> G not follows

Answer: c)

390-400 Questions :

Directions (Q.1-5): Study the following information to answer the given questions:
(a) Six plays are to be organized from Monday to Sunday –One play each day with one day when there is no play. ’No play’
day is not Monday or Sunday.
(b) The plays are held in sets of 3 plays each in such a way that 3 plays are held without any break i.e. 3 plays are held in
such a way, that there is no ‘No play’ day between them but immediately before this set or immediately after this set it is ‘
No play’ day.
(c) Play T is held on 26th and play X was held on 31st of the same month.
(d) Play U was not held immediately after play S (but was held after S, not necessarily immediately) and play V was held
immediately before W.

(e) All the six plays were held in the same month.

1). Which play was organized on Monday?


a) T
b) V

www.bankingpdf.com
c) W
d) Cannot be determined
e) None of these

2). Which day was play T organized?


a) Tuesday
b) Monday
c) Wednesday
d) Cannot be determined
e) None of these

3). Which date was a ‘No play’ day?


a) 26th
b) 28th
c) 29th
d) Cannot be determined
e) None of these

4). Which of the following is true?


a) Play U is held immediately before play V

www.bankingpdf.com
b) Play T is held after play U
c) There was a gap after 2 plays and then 4 plays were organized
d) First play was organized on the 25th
e) Play U was held on Friday

5). Which day was play W organized?


a) Friday
b) Wednesday
c) Saturday
d) Cannot be determined
e) None of these

Directions (Q.6-10) :
Eight friends; A, B, C, D, E, F, G and H are sitting around a circle facing towards the center. Each of these friends is using
one application in his mobile out of facebook, Twitter, Hike, Snapchat, Line, WhatsApp, Skype and Viber. The persons
who are using Line and Vibre are sitting opposite to each other. F is using Snapchat and is not sitting adjacent to the
person who is using Facebook. C is sitting third to left of the person who is using Twitter. A is using Hike and sitting 2 nd to
the left of D, who is using Skype. E using WhatsApp and sitting opposite to H. Two people are sitting between the persons
who are using Twitter and Skype. H is not using Skype, Line or Viber.

www.bankingpdf.com
6).How many arrangements are possible?
a) Two
b) Four
c) One
d) Six
e) None of these

7).Who is using Facebook?


a) B
b) H
c) C
d) A
e) Can’t be determined

8).Which of the following statements is definitely wrong?


a) The person using Hike is sitting between the persons using Twitter and Viber.
b) F is sitting opposite to D.
c) There is one person is sitting between the persons using Snapchat and Hike.
d) E is sitting to the immediate right of G.
e) None of these

www.bankingpdf.com
9). Who is using Viber?
a) B
b) C
c) H
d) G
e) Can’t be determined

10). Who is sitting third to the right of the person using WhatsApp?
a) C
b) H
c) F
d) B
e) Can’t be determined

Answers:
1).e) 2).a) 3).b) 4).d) 5).c) 6).b) 7).c) 8).d) 9).e) 10).c)

Explanation:

www.bankingpdf.com
Directions (Q. 1-5):
Date Day Play

25 Monday S

26 Tuesday T

27 Wednesday U

28 Thursday No play

29 Friday V

30 Saturday W

31 Sunday X

Directions (Q. 6-10):

www.bankingpdf.com
www.bankingpdf.com
400-410 Questions :

Directions (Q.1-5): Five students; P,Q,R,S and T appeared in a test that comprises of five different languages namely
English, Hindi, Sanskrit, French and German. Each student got a different rank in each of these five subjects and rank of
no two students was the same in a subject. It is known that:
(i). P got 1st rank in German whereas S got 2nd rank in Hindi.
(ii). Q got 5th rank in English and T got 1st rank in French.
(iii). S got 4th rank in Sanskrit and the person who got 3rd rank in German got 5th rank in Sanskrit.
(iv). R’s rank in German was equal to T’s rank in Hindi and Q’s rank in German was equal to R’s rank in Sanskrit.
(v). T’s rank in German was equidistant from both his highest and lowest rank.

1). Who got the lowest rank in Hindi?


a) Q
b) T
c) P
d) R
e) Can’t be determine

2). Who got the first rank in English?


a) P

www.bankingpdf.com
b) R
c) T
d) Q
e) None of these

3). Q’s rank in French was equal to whose rank in German?


a) Q
b) T
c) R
d) P
e) S

4). Who got 1st rank in Sanskrit?


a) Q
b) R
c) P
d) S
e) Can’t be determined

5). P got 2nd rank in which subject?

www.bankingpdf.com
a) Hindi
b) Sanskrit
c) French
d) English
e) Can’t be determined

6).Below in each question are given two statements(A) and (B). These statements may be either independent causes or
may be effects of independent causes or a common cause. One of these statements may be the effect of the other
statement. Read both the statements and decide which of the following answer choices correctly depicts the relationship
between these two statements.
A. The average day temperature of the city has increased by about 2 degrees in the current year over the average of past
ten years.
B. More people living in rural areas of the state have started migrating to the urban areas in comparison with the earlier
year.
a) If statement (A) is the cause and statement (B) is its effect.
b) If statement (B) is the cause and statement (A) is its effect.
c) If both the statements (A) and (B) are independent causes.
d) If both the statements (A) and (B) are independent causes.
e) If both the statements (A) and (B) are effects of some common cause.

www.bankingpdf.com
7). In question below is given a statement lowed by two assumptions numbered I and II. An assumption is something
supposed or taken for granted. You have to consider the statement and following assumptions and decide which of the
assumptions is implicit the statement.

Statement: The government has decided to earmark a separate lane in the metropolis for passenger vehicles with more
than one occupant.
Assumptions:
I. The move may help decongest the roads of the metropolis.
II. Many people may resort to pool car system to avoid traffic snarls.
a) If only assumption I is implicit
b) If only Assumption II is implicit.
c) If either Assumption I or Assumption II is implicit.
d) If neither Assumption I nor Assumption II is implicit.
e) If both Assumption I and Assumption II are implicit.

8).In each question below a statement is given followed by two courses of action numbered I and II. A course of action is
a practicable and feasible step or administrative decision to be taken for follow-up, improvement, or further action in
regard to the problem, policy, etc. On the basis of the information given in the statement, you have to assume everything
in the statement to be true, and decide which of the suggested courses of action logically follows(s) for pursuing.

www.bankingpdf.com
Statement: A university librarian reported increased cases of theft of books from the library.
Courses of action:
I. Stricter security arrangements should be put in place in order to prevent such incidents.
II. All the students in the university should be made to pay a hefty fine in order to replace the lost books.
a) If only I follows
b) If only II follows
c) If either I or II follows
d) If neither I nor II follows
e) If both I and II follow

9).In a certain code language PRECISE is written as FULCBPO. How would MONSOON be written in that code?
a) OQRSNQQ
b) NQQSKML
c) OQRSKML
d) OPQSKML
e) None of these

10). If R is coded as 5, L is coded as 1,N is coded as 6, T is coded as 2, U is coded as 8, F is coded as 3, S is coded as 4,


I is coded as 9,and G is coded as 7,what is the decoded form of 425967?
a) TRINGS
b) SLINGS

www.bankingpdf.com
c) STRING
d) FUSING
e) None of these

Answers:
1).c) 2).e) 3).c) 4).a) 5).c) 6).d) 7).e) 8).a) 9).c) 10).c)

Explanation:

Directions (Q. 1-5):


Hindi English Sanskrit French German

P 5 4 3 2 1

Q 3 5 1 4 2

R 1 3 2 5 4

S 2 1 4 3 5

T 4 2 5 1 3

www.bankingpdf.com
1).
Answer: c)
2).
Answer: e)
3).
Answer: c)
4).
Answer: a)
5).
Answer: c)

6). The reason for increase in temperature could be deforestation or increase in pollution or some other cause
People migrate to urban areas in search of jobs or better living conditions
Hence, the reasons for the given statements are different
Answer: d)

7). The reason for creating a separate lane in the metropolis for passenger vehicles with more than one occupant could
be to reduce the road traffic. Hence, statement I is implicit
To avoid traffic snarls, people may pool car system as they will be able to use separate lane. Hence, statement II is also
implicit
Answer: e)

www.bankingpdf.com
8). To avoid the thefts in the future, stricter security arrangements should be put in place . Hence, course of action I is
valid
Asking all the students to pay a hefty fine is not a valid step.
Answer: a)

9). First the given word is reversed and its letters are shifted in the below manner:
PRECISE
E S I C E R P

+1 +2 +3 0 -3 -2 -1

F U L C B P O

Similarly, for MONSOON


N O O S N O M

+1 +2 +3 0 -3 -2 -1

O Q R S K M L

Hence, it would be written as OQRSKML

www.bankingpdf.com
Answer: c)

10). 425967 = STRING


Answer: c)

420-430 Questions

Directions (Q. 1-3): Study the following information to answer the given questions:
In a certain code,
‘less money in bank’ is written as ‘zo li aa to’:
‘share in bank profit’ is written as ‘vo to je li’:
‘making less profit now’ is written as ‘su je zo ka’:
‘now the bank gains’ is written as ‘do li yo su’:
1). Which is the following does ‘vo’ stand for?
a) Profit
b) In
c) Share
d) Bank
e) In or profit

2). Which of the following is the code for ‘gains’?

www.bankingpdf.com
a) Su
b) Li
c) Yo
d) Do
e) Yo or do

3). ‘to ka li aa’ is a code for which of the following?


a) Share in bank
b) Now share less gains
c) The gains in bank
d) The gains in profit
e) Making money in bank

Directions (Q. 4-8): In each of the questions below are given four statements followed by four conclusions numbered I,II,
III and IV. You have to take the given statements to be true even if they seem to be at varience from commonly known
facts. Read all the conclusions and then decide which of the given conclusions logically follows from the given statements
disregarding commonly known facts.

4). Statements: Some tools are radios.


Some radios are ponds.

www.bankingpdf.com
Some ponds are mirror.
All mirror are chalk.
Conclusion:
I. some mirror are tools.
II. some chalks are ponds.
III. some ponds are tools.
IV. some chalks are radios.
a) None follows
b) Only I follows
c) Only II follows
d) Only III follows
e) Only IV follows

5). Statements: All chairs are sofas


All sofas are books
All books are nets
All nets are garden
Conclusion:
I some nets are sofas
II some garden are books
III some nets are chairs

www.bankingpdf.com
IV all books are garden
a) Only I, II and III follow
b) Only I, III and IV follow
c) Only II, III and IV follow
d) All I, II, III and IV follows
e) None of these

6). Statements: All letters are black.


All black are blue.
No blue is green.
Conclusions:
I. No letter is green.
II. some blue are black.
a) If only conclusion I follows.
b) If only conclusion II follows.
c) If either conclusion I or II follows.
d) If neither conclusion I nor II follows.
e) If both conclusions I and II follows

7). Statements: Some eyes are ears


Some ears are hands

www.bankingpdf.com
Conclusions:
I. No hand is an eye.
II. Some eyes are hands
a) If only conclusion I follows.
b) If only conclusion II follows.
c) If either conclusion I or II follows.
d) If neither conclusion I nor II follows.
e) If both conclusions I and II follows

8). Statements: Some books are pens.


Some pens are pencils.
Some pencils are buttons.
Conclusions:
I. some buttons are pens.
II. some pencils are books.
a) If only conclusion I follows.
b) If only conclusion II follows.
c) If either conclusion I or II follows.
d) If neither conclusion I nor II follows.
e) If both conclusions I and II follows

www.bankingpdf.com
Directions (Q. 9-10): Study the following information carefully to answer these questions.
(i) ‘P × Q’ means P is brother of Q
(ii) ‘P – Q’ means P is sister of Q
(iii) ‘P + Q’ means P is mother of Q
(iv) ‘P ÷ Q’ means P is father of Q
9). Which is the following means ‘M is daughter of R’?
a) R ÷ D × M
b) R + D × M
c) M – J × R + T
d) R + M - T
e) None of these

10). Which is the following means ‘K is maternal uncle of W?


a) K – J + W
b) B) K × J ÷ W
c) K × J + W
d) W + J × K
e) None of these

Answers:
1).c) 2).e) 3).e) 4).c) 5).d) 6).e) 7).c) 8).d) 9).d) 10).c)

www.bankingpdf.com
Explanation:

Directions (Q. 1 – 3)
Word Code

Less Zo

Money Aa

In To

Bank Li

Share Vo

Profit Je

Making Ka

Now Su

www.bankingpdf.com
The Do/yo

Gains Yo/do

1).
Answer: c)

2).
Answer: e)

3).
Answer: e)

4).

www.bankingpdf.com
Answer: c)

5).

Answer: d)

www.bankingpdf.com
6).

Answer: e)

7).

www.bankingpdf.com
Answer: c)

8).

Answer: d)

9).

www.bankingpdf.com
Answer: d)

10).

Answer: c)

www.bankingpdf.com
430-440 Questions

1). Starting from the point X, Anush walked 15 m towards west. He turned left and walked 20 m. He then turned left and
walked 15 m. After this he turned to his right and walked 12 m. How far and in which directions is now Anush from X?
a) 32 m, South
b) 47 m, East
c) 42 m, North
d) 27 m, South
e) 40 m, North-East

2). One evening before sunset Rekha and Hema were talking to each other face to face. If Hema’s shadow was exactly to
the right of Hema, which direction was Rekha facing?
a) North
b) South
c) East
d) Data is inadequate
e) West

3). A boy rode his bicycle Northward, then turned left and road 1 km and again turned left and road 2 km. He found
himself 1 km west of his starting point. How far did he ride northward initially?

www.bankingpdf.com
a) 1 km
b) 2 km
c) 3 km
d) 5 km
e) 4 km

Directions (Q .4-8: Study the following information to answer the given question:
A word and number arrangement machine when given an input line of words and numbers rearranges them following a
particular rule in each step. The following is an illustration of input and rearrangement.(All the numbers are two digits
numbers and are arranged as per some logic based on the value of the number.)

Input: win 56 32 93 bat for 46 him 28 11 give chance


Step 1 : 93 56 32 bat for 46 him 28 11 give chance win
Step II: 11 93 56 32 bat for 46 28 give chance win him
Step III: 56 11 93 32 bat for 46 28 chance win him give
Step IV : 28 56 11 93 32 bat 46 chance win him give for
Step V : 46 28 56 11 93 32 bat win him give for chance
Step VI : 32 46 28 56 11 93 win him give for chance bat
Step VI is the last Step of the arrangement of the above input as the intended arrangement is obtained.
As per the rules followed in the above steps, find out in each of the following questions the appropriate steps for the given
input,

www.bankingpdf.com
Input for the questions:
Input : 87 fun 89 at the 28 16 base 42 camp 74 song 35 yell 53 here 68 pie
(All the numbers given in the arrangement are two digit numbers.)

4).Which word/number would be at 7th position from the left in Step VI?
a) Base
b) At
c) 35
d) The
e) 53

5).Which step number would be the following output?


42 68 35 74 28 87 16 89 at 53 yell the song pie here fun camp base
a) There will be no such step
b) VIII
c) VII
d) VI
e) IV

6).Which step number would be the last step?

www.bankingpdf.com
a) X
b) VIII
c) VII
d) IX
e) VI

7).How many steps would be required for final arrangement after the following output?
87 16 89 fun at 28 base 42 camp 74 35 53 here 68 pie yell the song
a) There will be no such step
b) Four
c) Five
d) Six
e) Seven

8).Which of the following would be the Step V?


a) 74 28 87 16 89 fun at base 42 camp 53 35 68 yell the song pie here
b) 74 28 87 16 89 fun at base camp 42 53 35 68 yell the song pie here
c) 28 74 87 16 89 fun at base 42 camp 35 53 68 yell the song pie here
d) 74 28 87 16 89 fun at base 42 camp 35 53 68 yell the song pie here
e) None of these

www.bankingpdf.com
Directions (Q. 9-10): A coded statement is followed by two conclusions. Decode the statement and conclusion
The symbols have to be decoded as follows:
A (a) B means A is smaller than B.
A (b) B means A is greater or equal to B
A (c) B means A is greater than B.
A (d) B means A is equal to B.
A (e) B means A is shorter or equal to B.
a) If only conclusion I is true
b) If only conclusion II is true
c) If conclusion I or conclusion II is true
d) If neither conclusion I nor conclusion II is true
e) If both conclusions are true

9).Statement: N(a) P , P (c) R, R.(e) L


Inference: I. N(e) L
II.N (b) L

10). Statement: D(b)F, F (c) J, J (d) P


Inference: I. D (c) J

www.bankingpdf.com
II. F (c) P

Answers:
1).a) 2).b) 3).b) 4).b) 5).b) 6).d) 7).d) 8).d) 9).a) 10).e)

Explanation:

1).

Required distance = 20 + 12 = 32 m in south direction


Answer: a)

2).

www.bankingpdf.com
Answer: b)

3).

The boy rode 2 km. Northward


Answer: b)

Directions (Q. 4-8):


Input: 87 fun 89 at the 28 16 base 42 camp 74 song 35 yell 53 here 68 pie
Step I: 89 87 fun at the 28 16 base 42 camp 74 song 35 53 here 68 pie yell
Step II: 16 89 87 fun at 28 base 42 camp 74 song 35 53 here 68 pie yell the
Step III: 87 16 89 fun at 28 base 42 camp 74 35 53 here 68 pie yell the song
Step IV: 28 87 16 89 fun at base 42 camp 74 35 53 here 68 yell the song pie
Step V: 74 28 87 16 89 fun at base 42 camp 35 53 68 yell the song pie here
Step VI: 35 74 28 87 16 89 at base 42 camp 53 68 yell the song pie here fun
Step VII: 68 35 74 28 87 16 89 base at 42 53 yell the song pie here fun camp
Step VIII: 42 68 35 74 28 87 16 89 at 53 yell the song pie here fun camp base

www.bankingpdf.com
Step IX: 53 42 68 35 74 28 87 16 89 yell the song pie here fun camp base at
Step IX is the last step
4).
Answer: b)
5).
Answer: b)
6).
Answer: d)
7).
Answer: d)
8).
Answer: d)

9).
T≥P, P = L, J =>T≥P, P = L, J>L
Or, T > P, T = P = L, J > T = P = L.
Thus, I is true.
Answer: a)

10).
D ≥ F, F > J, J = P =>D > F, D = F > J, J = P

www.bankingpdf.com
Thus, both conditions I and II are true
Answer: e)

440-450 Questions

Direction (Q. 1-3): A coded statement is followed by two conclusions. Decode the statement and conclusion
The symbols have to be decoded as follows:
A (a) B means A is smaller than B.
A (b) B means A is greater or equal to B
A (c) B means A is greater than B.
A (d) B means A is equal to B.
A (e) B means A is shorter or equal to B.
a) If only conclusion I is true
b) If only conclusion II is true
c) If conclusion I or conclusion II is true
d) If neither conclusion I nor conclusion II is true
e) If both are true

1). Statement: M (d) T, T (b) S, S (e) K


Inference: I. M (e) K

www.bankingpdf.com
II. M (b) K

2). Statement: Q(a) R, R(b) T, T(e)U


Inference: I. Q (a) T
II. R (c) T

3). Statements: W (e) M, M (d) N, N (c) L


Inference: I. M (c) L
II. N (b) W

Directions (Q. 4-8): In these questions, relationship between different elements is given in the statements are followed by
two conclusions.
a) If only conclusion I follows.
b) If only conclusion II follows.
c) If either conclusion I or II follows.
d) If neither conclusion I nor II follows.
e) If both conclusion I and II follows
4). Statements:
N = P, P ≤ F, F ≥ L, L = K
Conclusions:

www.bankingpdf.com
I. F = K
II. F > K

5). Statements:
Z > T, T < M, M < J
Conclusions:
I. T < J
II. J < Z

6). Statements:
Q = Z, C ≥ G, G ≥ Q, Q ≥ R, J ≥ C
Conclusions:
I. G ≥ Z
II. C ≥ R

7). Statements: E < F ≤ G = H > S


Conclusions:
I. G > S
II. F ≤ H

www.bankingpdf.com
8). Statements: P ≤ Q < W = L
Conclusion:
I. L > P
II. Q ≤ L

9). In a certain code language, ‘kew xas huma deko, means ‘she is eating apples’: ‘kew tepo qua’ means ‘she sells toys’
and ‘sul lim deko’ means ‘ I like apples’,
Which of the following measn ‘she’ and ‘apples’ in that code language?
a) xas & deko
b) xas & kew
c) kew & deko
d) kew & xas
e) deko & tepo

10). In a certain code language, ‘851’ means ‘good sweet fruit’: ‘783’ means ‘good red rose’ and ‘341’ means ‘rose and
fruit’: 70 means ‘red garden’. How can ‘red rose garden, ’be written?
a) 470
b) 730
c) 753

www.bankingpdf.com
d) 580
e) 735

Answers:
1)c 2)e 3)c 4)c 5)a 6)e 7)e 8)a 9)c 10)b

Explanation:
1). M = T, T ≥ S, S ≤ K.
Or, M = T, M = T ≥ S, K ≥ S
Or, M = T = S, M = T > S, K ≥ S
Or, M = T = S = K, M = T > S, K > S.
Answer: c)

2). Q < R, R ≥ T, T ≤ U
Or, R > Q, R ≥ T, U ≥ T
Or, R = T > Q, R > T= U, U > T.
Thus, both conditions I and II are true
Answer: e)

3). W ≤M, M = N, N>L

www.bankingpdf.com
Or, M ≥ W, M = N, N >L
Or, M = N ≥ W, M = W =N, N > L
Answer: c)

4).
Answer: c)

5).
Answer: a)

6).
Answer: e)

7).
Answer: e)

8).
Answer: a)

www.bankingpdf.com
9).
Answer: c)

10).
Answer: b)

450-460 Questions

Direction (1 to 4): Read the information carefully and then answer the questions given below.
Shalu, Charu, Lata, Tom and Sandy help themselves to take some sweets from bowl. Four of them each take
a gulab jamun. Charu and Tom do not take a burfy as all the others do. Infact, Charu takes only one sweet, which is
a laddu. Laddu is not taken by others apart from Charu, only Shalu and Sandy do not take peda.

1). Who only had a peda and gulab jamun?


a) Charu
b) Sandy
c) Shalu

www.bankingpdf.com
d) Tom
e) None of these

2). Who takes three sweets?


a) Charu
b) Sandy
c) Shalu
d) Lata
e) None of these

3). Who are the two people taking the same number and same type of sweets?
a) Shalu and Lata
b) Sandy and Lata
c) Shalu and Sandy
d) Tom and Sandy
e) None of these

4). In total, how many sweets were taken by the group?


a) 11

www.bankingpdf.com
b) 12
c) 9
d) 10
e) 8

Directions (5 to 7):
Seven persons namely Prem, Tirkey, Madan, Jagan, Virendar, Raghubir and Waris are good friends and are
studying in MBA, engineering and medical courses. Three are doing MBA course, two are in engineering course and
another two are in medical course. Each of them has a very distinct and favorite colour choice ranging from blue, red,
yellow, white, black, pink and brown but not necessarily in the same order. None doing MBA likes either red or black.
Madan is doing engineering and he likes blue. Raghubir is doing medical and likes brown. Jagan is doing MBA and
likes yellow. Prem who does not like red is in the same discipline of Raghubir. Tirkey is in the same discipline of
Madan. Virender does not like pink.

5). Which of the following groups is doing MBA?


a) Jagan, Raghubir and Madan
b) Virendar, Waris and Tirkey
c) Jagan, Virendar and Waris
d) Jagan, Prem and Raghubir
e) None of these

www.bankingpdf.com
6). What is the colour combination choice of these who are in medical discipline?
a) Brown and pink
b) Black and white
c) Black and brown
d) Yellow and black
e) None of these

7). Which colour does Virendar like?


a) Yellow
b) Pink
c) White
d) Brown
e) None of these

Directions (8 to 11): Read the following information carefully and then answer the questions given below it.
i). P, Q, R, S, T and U are six members of a family, each of them engaged in a different profession – doctor, lawyer,
teacher, engineer, nurse and manager.
ii). Each of them remains at home on a different day of the week from Monday to Saturday.

www.bankingpdf.com
iii). The lawyer in the family remains at home on Thursday
iv). R remains at home on Tuesday
v). P, a doctor, does not remain at home either on Saturday or Wednesday
vi). S is neither the doctor nor the teacher and remains at home on Friday.
vii). Q is the engineer and T is the manager.

8). Which of the following combinations is not correct?


a) R-Teacher
b) Q-Engineer
c) T-Manager
d) All correct
e) None of these

9). Which of the following combinations is correct?


a) Lawyer-Tuesday
b) Teacher-Wednesday
c) Manager-Friday
d) Nurse-Friday
e) None of these

www.bankingpdf.com
10). Who is the nurse?
a) S
b) R
c) U
d) Data inadequate
e) None of these

11). Who among them remains at home on the following day on which R stays at home?
a) Q
b) Q or T
c) S
d) Cannot be determined
e) None of these

Answers:
1).d) 2).d) 3).c) 4).d) 5).c) 6).c) 7).c) 8).d) 9).d) 10).a) 11).b)

Solution:

www.bankingpdf.com
Direction (Questions 01-04) Following table can be drawn from the given information
Name Gulab Burfy Laddu Peda

Shalu ✓ ✓ ✗ ✗

Charu ✗ ✗ ✓ ✗

Lata ✓ ✓ ✗ ✓

Tom ✓ ✗ ✗ ✓

Sandy ✓ ✓ ✗ ✗

1). Tom took a peda and gulab jamun.


Answer : d)

2). Lata took gulab jamun, burfy and peda.


Answer : d)

www.bankingpdf.com
3). Shalu and Sandy took gulab jamun and burfy.
Answer : c)

4). In total 10 sweets were taken by the group.


Answer : d)

Direction (Questions 05-07) following table can be drawn from given information
Person Courses Colours

Prem Medical Black

Tirkey Engineering Red

Madan Engineering Blue

Jagan MBA Yellow

Virendar MBA White

www.bankingpdf.com
Raghubir Medical Brown

Waris MBA Pink

5). Jagan, Virendar and Waris are doing MBA.


Answer : c)

6). Medical persons choice of colours are black and brown.


Answer : c)

7). Virendar likes white colour.


Answer : c)

Direction (Questions 08-11) following table can be drawn from the given information.
Professions Days

Doct Law Teach Engg Nurse Mag Mon Tues Wed Thurs Fri sat

P ✓ ✗ ✗ ✗ ✗ ✗ ✓ ✗ ✗ ✗ ✗ ✗

www.bankingpdf.com
Q ✗ ✗ ✗ ✓ ✗ ✗ ✗ ✗ ✓ ✗ ✗ ✓

R ✗ ✗ ✓ ✗ ✗ ✗ ✗ ✓ ✗ ✗ ✗ ✗

S ✗ ✗ ✗ ✗ ✓ ✗ ✗ ✗ ✗ ✗ ✓ ✗

T ✗ ✗ ✗ ✗ ✗ ✓ ✗ ✗ ✓ ✗ ✗ ✓

U ✗ ✓ ✗ ✗ ✗ ✗ ✗ ✗ ✗ ✓ ✗ ✗

From the table, we find that Q, the engineer and T, the manager live at home either on Wednesday or Saturday.

8). It is concluded clearly from the above table that all the combinations given are correct.
Answer : d)

9). From the table it is clear that S is the nurse and stays at home on Friday.
Answer : a)

10). S is the nurse


Answer : a)

www.bankingpdf.com
11). R stays at home on Tuesday and from the table, any one of Q or T stays at home following the day on which R
stays at home i.e., on Wednesday
Answer : b)

460-470 Questions

Directions (Q. 1-5): Study the following information and answer the questions that follow.
Dharani, Dharitri, Dhatri, Dhitha, Dhithi, Dhriti and Dhuthi organise seminars in seven different schools, viz ABC, DEF,
GHI, JKL, MNO, PQR, STU, on different days of the week from Monday to Sunday, but not necessarily in the same order.
Dharitri organises seminar in school JKL on Wednesday. Dhithi does not organise seminar for school STU or E but
organises on the next day of Dhitha’s seminar, who organises seminar for school DEF. Dhuthi organises seminar for
school GHI on Friday. Dhriti organises seminar on Monday, but not for school MNO or A. Dhatri organises seminar for
school STU but not on Tuesday.

1). Who among the following organises seminar in school MNO and on which day?
a) Dhitha, Saturday
b) Dharani, Tuesday
c) Dhriti, Monday

www.bankingpdf.com
d) Dhatri, Thursday
e) None of these

2). Which of the following combination is true?


a) Dhitha – Tuesday - MNO
b) Dhithi – Wednesday - GHI
c) Dhriti – Monday - PQR
d) Dhuthi – Thursday - ABC
e) None of these

3). On which of the following days does Dhithi organise the seminar?
a) Tuesday
b) Sunday
c) Saturday
d) Cannot be determined
e) None of these

4). Dhatri organises seminar on which of the following days?


a) Saturday
b) Sunday

www.bankingpdf.com
c) Tuesday
d) Thursday
e) None of these

5). Who among the following organises seminar in Saturday?


a) Dhithi
b) Dharani
c) Dharitri
d) Dhitha
e) None of these

6). Statement: K < X ≤ V > U = Z < L < P = R


Conclusions:
I. K < Z
II. R > U
III. V > Z
IV. U > K

a) Only I and III follow


b) Only I and IV follow
c) Only II and III follow

www.bankingpdf.com
d) Only II, III and IV follow
e) None of these

7). Statement: A = Z ≥ D < V < M ≤ N


Conclusions:
I. M > Z
II. D < N
III. A > V
IV. D ≤ A

a) Only I and II follows


b) Only I, II and III follows
c) Only II and IV follow
d) Only II, III and IV follow
e) None of these

8). Statement: H ≥ P = R ≥ V < G ≥ E > S


Conclusions:
I. H ≥ V
II. R > E
III. G > P

www.bankingpdf.com
IV. G > S

a) Only I and III follow


b) Only I and IV follow
c) Only II and III follow
d) Only II, III and IV follow
e) None of these

9). Statement: J < Y ≤ S > U = W < L < Q = T


Conclusions:
I. Q > U
II. W < S
III. T > U
IV. S > J

a) Only I, II and III follows


b) Only II, III and IV follow
c) Only I, III and IV follows
d) All I, II, III and IV follows
e) None of these

www.bankingpdf.com
10). Statement: Q ≥ O = R ≥ N < F ≥ K > S
Conclusions:
I. F > O
II. S < R
III. Q > N
IV. K > N

a) None follows
b) Only III follows
c) Only IV follows
d) Only either III or IV follows
e) None of these

Answers:
1).b) 2).c) 3).b) 4).d) 5).d) 6).c) 7).c) 8).b) 9).d) 10).a)

Explanation:

Directions (Q. 1-5):

www.bankingpdf.com
www.bankingpdf.com
1).
Answer: b)
2).
Answer: c)
3).
Answer: b)
4).
Answer: d)
5).
Answer: d)

6). K < X ≤ V > U = Z < L < P = R


K < Z not follows
R > U follows
V > Z follows
U > K not follows
Answer: c)

7). A = Z ≥ D < V < M ≤ N


M > Z not follows

www.bankingpdf.com
D < N follows
A > V not follows
D ≤ A follows
Answer: c)

8). H ≥ P = R ≥ V < G ≥ E > S


H ≥ V follows
R > E not follows
G > P not follows
G > S follows
Answer: b)

9). J < Y ≤ S > U = W < L < Q = T


Q>U
W<S
T>U
S>J
All follows
Answer: d)

www.bankingpdf.com
10). Q ≥ O = R ≥ N < F ≥ K > S
F>O
S<R
Q>N
K>N
None follows
Answer: a)

470-480 Questions

Direction (Q. 1 – 5): Eight persons A, B, C, D, E, F, G, H who work in eight different companies namely Infosys, TCS,
Wipro, Accenture, IBM, Mind Tree, HCL and CTS. Each person is working in exactly one company and no two people are
working in the same company. They are sitting along two opposite sides of a rectangular table, 4 along each side. It is
further known that:
1 – C and D are sitting opposite to each and D works in CTS.
2 – A works in Infosys and he is sitting two places away from D.
3 – E doesn’t work in Wipro or HCL and F is sitting 2nd to the left of C.
4 – B works in Accenture and he is not sitting adjacent to D.
5 – The persons working in Mind Tree and HCL are sitting next to each other.
6 – H is sitting opposite to the person who works in Wipro and D is not seated at any corner.
7 – C doesn’t work in Mind Tree or HCL or TCS.

www.bankingpdf.com
1). What are the two pairs of people sitting diagonally opposite to each other?
a) A – B and F – G
b) A – C and E – F
c) A – C and D – F
d) A – B and E – F
e) can’t be determined

2). Who works in TCS?


a) H
b) F
c) G
d) E
e) None of these

3). Who works in HCL?


a) H
b) G
c) C

www.bankingpdf.com
d) F
e) Can’t be determined

4). The person who works in IBM is sitting opposite to whom?


a) D
b) E
c) H
d) G
e) Can’t be determined

5). Find the odd one out?


a) A – H
b) D – B
c) F – D
d) C – E
e) G – C

Direction (Q. 6 -10): Six persons Harish, Vinod, Raju, Chetan, Shruti and Shikha live in a four story building. The floors are
numbered G (ground), 1, 2, and 3 from bottom to top. Shruti and Shika are girls and rest are boys. These people belong

www.bankingpdf.com
to six different cities out of Bangalore, Kolkata, Delhi, Mumbai, Allahabad and Noida. Each person belongs to a different
city. Every floor has either one or two persons living on it. it is known that
1 – The number of persons living on the first and second floor together is equal to the number of people living on the
ground floor and top floor together.
2 – Vinod who belongs to Kolkata lives above Shruti and Shruti lives below the persons who belong to Bangalore and
Mumbai.
3 – Chetan Doesn’t live on the same with Shruti or Vinod.
4 – Raju does not live on the ground and Shikha and Harish do not live on the same floor.
5 – Harish lives on the 2nd floor and above Chetan who belongs to Delhi.
6 – Shruti and Shikha do not live on the same floor and Raju lives just below Shikha who belongs to Allahabad.

6). Which of the given options lists the persons who live alone on their respective floors?
a) Chetan, Shruti
b) Chetan, Either Shruti or Vinod
c) Raju, Chetan
d) Shruti, either Chetan or Vinod
e) Can’t be determined

7). Who belongs to Noida?


a) Harish

www.bankingpdf.com
b) Raju
c) Shruti
d) Vinod
e) Can’t be determined

8). Who lives on the top floor?


a) Shikha, Harish
b) Shikha, Vinod
c) Harish, Vinod
d) Shriti, Vinod
e) None of this

9). Who lives on the ground floor and the first floor respectively?
a) Shruti, Harish and Raju
b) Cheatan, Shruti
c) Vinod, Harish
d) Shruti, Chetan
e) None of this

www.bankingpdf.com
10). Who belongs to Mumbai?
a) Raju
b) Shikha
c) Harish
d) Vinod
e) Can’t be determined

Answers:
1)d 2)d 3)e 4)a 5)c 6)a 7)c 8)b 9)d 10)e

Explanation:

Directions (Q. 1-5):

www.bankingpdf.com
Directions (Q. 6-10):

www.bankingpdf.com
480-490 Questions

1). Among P, Q, R, S and T each having a different weight, S is heavier than P and T and Q is lighter than R. who among
them is the heaviest?
a) S
b) Q
c) R
d) Data inadequate
e) None of these

Directions (Q. 2-6): Refer to the following statements and answer the question:
Seven students Riya, Arun, Ruban, Sujin, Raina, Rakesh, and Vino take a series of test. No two students get similar
marks. Vino always scores more than Riya. Riya always scores more than Arun. Each time either Ruban scores the
highest and Raina gets the least, or alternatively Sujin scores the highest and Rakesh or Arun scores the least.

2). If Sujin is ranked sixth and Arun is ranked fifth, which of the following can be true?
a) Vino is ranked first or fourth
b) Ruban is ranked second or third
c) Raina is ranked fourth or fifth
d) Rakesh is ranked third or fourth

www.bankingpdf.com
e) None of these

3). If Ruban gets the highest, Vino should be ranked not lower than:
a) Second
b) Third
c) Fourth
d) Fifth
e) None of these

4). If Ruban is ranked scond and Arun is ranked first, which of the following must be true?
a) Sujin is ranked third
b) Raina is ranked third
c) Riya is ranked sixth
d) Both option 1 and 2
e) None of these

5). If Sujin is ranked second, which of the following can be true?


a) Rakesh gets more than Vino
b) Vino gets more than Sujin
c) Riya gets more than Ruban

www.bankingpdf.com
d) Riya gets more than Vino
e) None of these

6). If Vino is ranked fifth, which of the following must be true?


a) Sujin scores the highest
b) Ruban is ranked second
c) Raina is ranked third
d) Arun is ranked second
e) None of these

Directions (Q 7-11): Read the information given below and answer the next five questions that follow:
I. Six friends Shrey, Krish, Advik, Reyo, Amar and Ankit were engineering graduates. All six of them were placed in six
different companies and were posted in six different locations, namely Tisco-Jamshedpur, Telco-Pune, Wipro-Bangalore,
HCL-Noida, Mecon-Ranchi and Usha Martin-Kolkata.
Each of them has their personal e-mail id’s with different email providers i.e., Gamil, Indiatimes, Rediffmail, Yahoo,
Hotmail, Sancharnet, Though not necessarily in the same order.
II. The one who having e-mail account with Sancharnet works in Noida and the one having an e-mail account with
Indiatimes works for Tisco.
III. Amar does not stay in Bangalore and does not work for Mecon, the one who works for Mecon has an e-mail id with
Gmail.
IV. Shrey has an e-mail id with Rediffmail and works at Pune.

www.bankingpdf.com
V. Ankit does not work for Mecon and the one who works for Wipro does not have an e-mail account with Yahoo.
VI. Krish is posted in Kolkata, and does not have an account with Hotmail.
VII. Neither Ankit nor Reyo work in Noida
VIII. The one who is posted in Ranchi has an e-mail id which is not account of Rediffmail or Hotmail.
IX. Advik is posted in Jamshedpur.

7). The man who works in works in Wipro has an e-mail account wth?
a) Sancharnet
b) Yahoo
c) Rediffmail
d) Indiatimes
e) None of these

8). Which of the following e-mail-place person combination is correct?


a) Krish-Kolkata-rediffmail
b) Ankit-Banagalore-Indistimes
c) Amar-Noida-Yahoo
d) Reyo-Ranchi-Gmail
e) None of these

www.bankingpdf.com
9). Which of the following is true?
a) Amar is posted at Ranchi
b) Reyo is posted at Jamshedpur
c) Krish has an e-mail id with Yahoo
d) Shrey has an e-mail id with indiatimes
e) None of these

10). Which of the following sequences of location represent Ankit, Krish, Advik, Reyo, Shrey and Amar in the same order?
a) Bangalore, Noida, Pune, Jameshedpur, Ranchi, Kolkata
b) Bangalore, Kolokata, Jameshedpur, pune, noida, Ranchi
c) Kolkata, Bangalore, Jameshedpur, pune, noida, Ranchi
d) Kolkata, Jameshedpur, pune, Ranchi, Noida, Bangalore
e) None of these

11). People who have e-mail account with Indiatimes, Sancharnet and Yahoo work for which companies, in the same
sequence as the e-mail accounts mentioned?
a) Usha Martin, HCL, Wipro
b) Tisco, Wipro, Usha Martin
c) HCL, Tisco, Wipro
d) Tisco, HCL, Usha Martin
e) None of these

www.bankingpdf.com
Answers:
1)d 2)d 3)c 4)e 5)a 6)a 7)e 8)d 9)c 10)e 11)d

Solutions:
1). From the given data, S>P, S>T and R>Q
But there is no information about relationship between weights of R and S hence data is inadequate.
Answer: d)

2). Vino>Riya>Arun
Now given that Sujin is ranked sixth and Arun is ranked fifth.
Hence, the list can be:
Ruban
Vino/Rakesh
Vino/Riya/Rakesh
Riya/Rakesh
Arun
Sujin
Raina
So, Rakesh can be ranked second or third or fourth.

www.bankingpdf.com
Answer: d)

3). Vino>Riya>Arun
Given that Ruban gets the highest.
So, Ruban scores the highest and Raina gets the least
Hence, the list can be:
Ruban
Vino/Rakesh/Sujin
Vino/Riya/Rakesh/Sujin
Vino/Riya/Rakesh/Arun/Sujin
Riya/ Rakesh/ Arun/ Sujin
Rakesh/ Arun/ Sujin
Raina
So, Vino cannot be ranked lower than fourth.
Answer: c)

4). Arun is ranked first. This is not possible, as Arun always scores less than Riya and Riya scores less than Vino.
Answer: e)

5). If Sujin is ranked second, then Ruban scores the highest and Raina gets the least.

www.bankingpdf.com
Now, if you look at the option, Riya cannot get more than Ruban as Ruban is ranked 1 st and Riya always gets less than
Vino. So, option 3 and 4 are eliminated.
Also, Sujin is ranked 2nd Ruban is 1st, so Vino cannot score more than Sujin.
Hence, only option 1 is possible.
Answer: a)

6). Vino is ranked 5th. So, Riya has to be 6th and Arun 7th. As Arun is ranked last, Sujin has to be 1st.
Answer: a)

Directions (Q. 7-11):


Friends Companies Locations e-mail ID

Shrey Telco Pune Rediffmail

Krish Usha martin Kolkata Yahoo

Advik Tisco Jasmshedpur Indiatimes

Reyo Mecon Ranchi Gmail

Amar HCL Noida Sancharnet

www.bankingpdf.com
Ankit Wipro Bangalore Hotmail

7).
Answer: e)
8).
Answer: d)
9).
Answer: c)
10).
Answer: e)
11).
Answer: d)

www.bankingpdf.com
490-500 Questions

Directions [Ques 1-5]: Study the information below and answer the given below.
In a certain code ‘market has value’ is coded as ‘lo pe he’, ‘value for money’ is coded as ‘he zo kh’, ‘money in market’
is coded as ‘kh ma lo’ has little value is coded as ‘pe sy he’.
1). How will “in the market’ be coded?
a) Ma lo sy
b) Ma pe he
c) Lo he he
d) Ma lo ki
e) Lo kh sy
2). How will “little value for money” be coded?
a) Sy zo kh ma
b) He ma zo kh
c) Sy he pe lo
d) Sy lo zo pe
e) None of this
3). What is the code for “value”?
a) Pe
b) He
c) Lo

www.bankingpdf.com
d) Sy
e) None of this
4). What does the code “zo” stand for?
a) In
b) Market
c) Has
d) For
e) None of this
5). What is the code for money?
a) Kh
b) Ma
c) Pe
d) He
e) None of this
6). In each question below, there are three statements followed by four conclusions numbered I, II, III and IV. You
have to take the given statements to be true even if they seem to be at variance with commonly known facts and
then decide which of the given conclusions logically follow (s) from the given statements.
Statements:
All stones are rocks.
Some rocks are bricks.
Some bricks are cement.

www.bankingpdf.com
Conclusions:
I Some cements are rocks.
II Some bricks are stone.
III Some stones are cements.
a) Only III follow
b) I, II follow
c) Only either I and III follow
d) All follow
e) None of this
7). Statements:
All flats are buildings.
All buildings are bunglows.
All bunglows are apartments
Conclusions:
I some apartment are flats.
II all flats are bunglows.
III some bunglows are flats
a) None follows
b) Only I follow
c) Only II follows
d) Only III follows

www.bankingpdf.com
e) All follows
8). Statements;
Some spectacles are lenses.
Some lenses are frames
All frames are metals
Conclusions:
I some lenses are metals
II some metals are spectacles
III some frames are spectacles
a) None follows
b) Only I follows
c) Only III follows
d) Only II follows
e) All follows
Direcrion [questions 9-10]: Read the following information carefully and answer the questions which follow:
(i) A is mother of B
(ii) C is sister of A
(iii) D is father of C
(iv) B is son of E
9). Which of the following statements are required to establish that E is a male?
a) Only (iv)

www.bankingpdf.com
b) Both (ii) and (iv)
c) Both (i) and (iv)
d) Both (ii) and (iv)
e) Both (i) and (iii)
10). How is D related to E?
a) Father
b) Grandfather
c) Son in-law
d) Grandson
e) Father in-law

Answers:
1)d 2)e 3)b 4)d 5)a 6)e 7)e 8)b 9)c 10)e

Solutions:
Direction(1-5):

word Code

Value He

www.bankingpdf.com
Money Kh

For Zo

Has Pe

Market lo

Little Sy

In Ma

6. e)

7. e)

www.bankingpdf.com
8. b)

9. c) Consider option1:
From statement (iv), E could be male or female because if B is son of E can be mother or father of B.
Hence, we cannot conclude that E is male
Consider option 2:
Both (ii) and (iv)

www.bankingpdf.com
Using these statements also, we cannot conclude that E is male
Consider option 3:
A is the mother of B means A is definitely female and if B is son of E, then E should be father of B as A mother pf B.
Hence, we can conclude that E is male.

Option 3
10. e)
E is husband of D’s daughter. There, D is father – in – law of E

www.bankingpdf.com
500-510 Questions

1). In a certain code, TPIPPLE is written as SQHOOKD. How is DISPOSE written in the code?
a) CHRONRD
b) DSOESPI
c) ESJTPTF
d) ESOPSID
e) None of these

2). In a certain code MONKEY is written as YDJMNL. How is TIGER written in that code?
a) UJHFS
b) SDFHS
c) SHFDQ
d) QDFHS
e) None of these

3). In a certain code COMPUTER is written as RFUVQNPC. HOW IS MEDICINE written in same code?
a) EOJDJEFM
b) EOJDEJFM
c) MFEJDJOE

www.bankingpdf.com
d) MEEDJJOE
e) None of these

4). In a certain word code language, ‘bi nie pie’ means ‘some good jokes’; ‘nie bat lik’ means ‘some real stories’; and ‘pie
lik tol’ means ‘many good stories’. Which word in that code means ‘jokes’?
a) bi
b) nie
c) pie
d) kon
e) None of these

5). If tee see pee means ‘Drink fruit juice’; ‘see kee lee’ means ‘juice is sweet’ and ‘lee ree mee’ means ‘he is intelligent’,
which word in that language means ‘sweet’?
a) see
b) kee
c) lee
d) pee
e) tee

Directions (Q. 6-10): study the information below and answer questions based on it.

www.bankingpdf.com
A , B, C, D, E, F and G are brothers. Two brothers had an arguments and A said to B “you are as old as C was when I
was twice as old as D, and will be as old as E was when he was as old as C is now”. B said to A, “You may be older than
F but G is as old as I was when you were as old as G is, and D will be as old as F was when F will be as old as G is”.
6). Who is the eldest brother?
a) A
b) E
c) C
d) D
e) Cannot be determined

7). Who is the youngest brother?


a) B
b) D
c) F
d) A
e) Cannot be determined

8). Which two are probably twins?


a) D and G
b) E and C
c) A and B

www.bankingpdf.com
d) A and C
e) Cannot be determined

9). Which of the following is false?


a) G has 4 older brothers
b) A is older than G but younger than E
c) B has three older brothers
d) There is a pair of twins among the brothers
e) All are true

10). How many brothers are younger than G?


a) 0
b) 1
c) 2
d) 3
e) Cannot be determined

Answers:
1)a 2)d 3)a 4)a 5)b 6)b 7)b 8)c 9)c 10)c

www.bankingpdf.com
Explanation:
1). Each letter in. the word is moved one step backward to obtain the corresponding letter of code
Answer: a)

2). The letters of the word are written in a reverse older and then each letter is moved one step backward to obtain the
code.
Answer: d)

3). The letters of the word are written in a reverse order and each letter, except the first and the last one, is moved one
step forward, to obtain the code.
Answer: a)

4). In first and third statements, the common-code word is ‘fit’ and the common word is ‘good’. So ‘pie’ means ‘good’. In
the first and second statements, the common code word is ‘nie’ and the common word is ‘some’. So ‘nice’ means ‘some’
thus, in the first statement, ‘bi’ means ‘jokes’.
Answer: a)

www.bankingpdf.com
5). In the first and second statements, the common word is ‘juice’ and the common code word is ‘See’. So see means ‘
Juice’. Similarly the common word in word in the second and third statements is ‘is’ and code word is ‘lee’. So ‘lee’ means
is. Thus in the second statement is ‘is’ and code word is ‘lee’. So ‘lee’ means is. Thus is the second statement, the
remaining word is ‘sweet’ is coded as ‘kee’.
Answer: b)

Directions (Q. 6-10):


Combining all the statements
E>C>B=A>G>F>D
E is the eldest brother.
6).
Answer: b)
7).
Answer: b)
8).
Answer: c)
9).
Answer: c)
10).
Answer: c)

www.bankingpdf.com
510-520 Questions

Directions [question 1 -4]: Answer the questions on the basis of the information given below.
Six students of foreign languages, Abel, Banhi, Chetan, Daha, Edhas, and Farida, are seated together. They
do not all speak the same language, but enough of them speak the same languages that they can translate for each
other. Abel and Daha speak only English, French, and Spanish. Banhi speaks only English, French; and Swedish.
Chetan speaks only German and Spanish. Edhas speaks only Spanish. Farida speaks only Swedish.

1). Which language does majority of the students speak?


a) English
b) French
c) German
d) Spanish
e) Swedish

2). Which of the following pair of students could talk to each other without a translator?
a) Abel and Farida
b) Banhi and Chetan
c) Banhi and Edhas
d) Banhi and Farida
e) Edhas and Farida

www.bankingpdf.com
3). Who could act as a translator for a conversation between Banhi and Chetan
I. Abel II. Daha III. Edhas IV. Farida
a) I only
b) I and II
c) I, II, and III
d) II, III, and IV
e) None

4). If Chetan and Farida wish to talk to each other, what is the fewest number of translators they would need?
a) 0
b) 1
c) 2
d) 3

e) 4

Directions (Q. Nos. 5-9) Read the following information and choose the right alternative in the questions that follow.
During the cultural week of an institute six competitions were conducted. The cultural week was inaugurated in the
morning of 19th October, Wednesday and continued till 26th October. In the span of 8 days six competitions namely
debate, folk dance, fash-p, street play, rock band and group song, were organized along with various other cultural
programmes. The information available from the institute is

www.bankingpdf.com
I. Only one competition was held in a day.
II. Rock band competition was not conducted on the closing day.
III. Fash-p was conducted on the day prior to debate competition.
IV. Group song competition was conducted neither on Wednesday nor on Saturday.
V. None of the competition was conducted on Thursday and Sunday.
VI. Street play competition was held on Monday.
VII. There was gap of two days between debate competition and group song competition.

5). The cultural week started with which competition?


a) Fash-p competition
b) Debate competition
c) Street play competition
d) Rock band competition
e) None of these

6). How many days gap is there between Rock band competition and group song competition?
a) Two
b) Three
c) Four
d) Five

www.bankingpdf.com
e) None of these

7). Which pair of competition was conducted on Wednesday?


a) Rock band competition and debate competition
b) Debate competition and fash-p competition
c) Rock band competition and folk dance competition
d) All of the above
e) None of the above

8). Which competition exactly precedes the street play competition?


a) Rock band competition
b) Group song competition
c) Debate competition
d) Fash-p competition
e) None of these

9). Fash-p competition follows which competition?


a) Debate competition
b) Street play competition

www.bankingpdf.com
c) Rock band competition
d) All of the above

e) None of the above


10). Among five friends P, Q, R, S and T, each scored different marks in the examination. P scored more than Q but
less than R. S scored more than only T. who amongst the following scored the second highest marks?
a) P
b) Q
c) R
d) S
e) T

Answers: 1).d 2).d 3).b 4).c 5).d 6).d 7).c 8).c 9).c 10).a

Solutions (Q. Nos. 5-9)


Form V, no competition was held on 20th and 23rd.
From VI, street play was held on 24th.
From III and VII, debate must have been held on 22nd
Fash-p on 21st and group song on 25th.
From II, rock band was held on 19th.

www.bankingpdf.com
Now, following table can be drawn from the given information

5). The cultural week started with ‘rock band’ competition.


Answer: d)

6). Gap of 5 days is there between rock band competition and group song competition.
Answer: d)

7). Rock band and folk dance was conducted on Wednesday.


Answer: c)

8). Debate precedes street play.


Answer: c)

9). Fash-p follows rock band.

www.bankingpdf.com
Answer: c)

10). P scored more than Q but less than R


R>P>Q
S Scored more than only T
S>T
Since, S scored more than only T, T’s score will be lowest

Hence, R > P > Q > S > T

520-530 Questions

1). In a row of children facing North, Deepa if shifted to her right by four places becomes eighteenth from the right end of
the row. Sharma, who is fifteenth from the left end of the row, is fifth to the left of Deepa. How many children are there in
the row?
a) 42
b) 38
c) 41
d) 39
e) None of these

www.bankingpdf.com
2). If '+' means '÷', 'x' means '–', '–' means 'x' and '÷' means '+', then what is the value of : 38 + 19 – 16 × 17 ÷ 3 = ?
a) 16
b) 19
c) 17
d) 12
e) None of these

3). Looking at a woman sitting next to him, Raghu said, ‘she is the sister of the husband of my wife’. How is the woman
related to Raghu?
a) Niece
b) Daughter
c) Sister
d) Wife
e) None of these

4). In a certain code language 'DEAR' is written as '%#$?’,'RED' is written as '?#%', 'MORE' is written as '@*?#', how is
'DREAM' written in that code?
a) # ? % $ @
b) # % ? $ @

www.bankingpdf.com
c) % ? # $ @
d) % ? # $ @
e) None of these

5). In a class among the passed students Anjali is 22nd from the top and Manoj, who is 5 ranks below Anjali is 34th from
the bottom. All the students from the class appeared for an examination. If the ratio of the students, who passed in the
examination to those who failed, is 4:1 for the class, how many students were there in the class?
a) 90
b) 60
c) 75
d) Data Inadequate
e) None of The Above

Directions (Q.6-10): Study the given information carefully to answer the given question.
A show case fixed against the wall of a showroom has six compartments. Each compartment contains a chain of a
different fruits viz Apple, Orange, Mango, Grapes, Banana and Pears, but not necessarily in the same order. The Apple
chain has been kept neither on the top nor on the bottom. Only one chain has been kept between the Banana chain and
the Mango chain. There are two chains between the Pears chain and the Apple chain. The Grapes chain is just above the
Banana chain. The Orange chain is below the Apple chain.

www.bankingpdf.com
6). What is the position of the Banana chain with respect to Pears chain?
a) Four places above
b) Four places below
c) Two places below
d) Either option a or c
e) None of these

7). How many chains are there between the Pears chain and the Orange chain?
a) None
b) Three
c) Four
d) Either option a or c
e) None of these

8). What is the position of the orange chain when counted from the bottom?
a) First
b) Second
c) Third
d) Fourth
e) Cannot be determined

www.bankingpdf.com
9). How many rearrangements are possible based on the given information?
a) One
b) Two
c) Three
d) Four
e) None of these

10). How many chairs are there between the Apple chain and the Orange chain?
a) None
b) One
c) Two
d) Cannot be determined
e) None of these

Answers:
1). c) 2). e) 3). c) 4). d) 5). c) 6. d) 7. d) 8. e) 9. b) 10. b)

Explanation:
1).

www.bankingpdf.com
Answer: c)

2). 38 + 19 – 16 × 17 ÷ 3 = ?
38 ÷ 19 × 16 - 17 + 3 = 18
Answer: e)

3).
Answer: c)

4). DEAR = %#$?


RED = ?#%
MORE = @*?#
DREAM = % ? # $ @
Answer: d)

5). From among those who passed Anjali is 22nd from the top and Manoj is 22 + 5 = 27th from the top and 34th from the
bottom. Therefore, total number of students who passed the examination = 27 + 34 – 1 = 60
Therefore, total number of students in the class = 60/4 × 5 = 75
Answer: c)

www.bankingpdf.com
Directions (Q. 6-10):
There are two possible arrangements
Possibility
Possibility III
Grapes
Pears
Banana
Grapes
Banana
Apple
Mango
Apple
Orange
Mango
Orange
Pears

6).
Answer: d)
7).
Answer: d)
8).
Answer: e)
9).
Answer: b)
10).

www.bankingpdf.com
Answer: b)

530-540 Questions

Directions (Q. 1-5): study the following information carefully and answer the given questions.
Seven flights namely IndiGo, GoAir, Air Asia India, Kingfisher, Vistara, Spice Jet, Jet Lite are scheduled to fly to London.
There is only one flight to London on each-of the seven days of the week, starting from Monday and Ending on Sunday.
Air Asia India flies on Wednesday. Jet Lite flies the day next to GoAir. GoAir does not fly on Monday or Friday. Two
airlines fly between the days GoAir and Vistara fly. Vistara does not fly on Sunday. Kingfisher flies a day before Spice Jet.

1). On which of the following days does IndiGo fly?


a) Friday
b) Sunday
c) Tuesday
d) Thursday
e) None of these

2). How many flights fly between Spice Jet and Air Asia India?

www.bankingpdf.com
a) None
b) One
c) Two
d) Three
e) Five

3). Which of the following flights on Friday?


a) Air India
b) Kingfisher
c) Vistara
d) Spice Jet
e) IndiGo

4). If Air Asia India postpones its flight to Sunday owing to some technical reasons and all the flights scheduled for
Thursday to Sunday are now made to take off a day ahead of the schedule, which of the following flights would now fly on
Friday?
a) Spice Jet
b) IndiGo
c) GoAir
d) Air India
e) Kingfisher

www.bankingpdf.com
5). If Vistara is related to GoAir and Air Asia India is related to Kingfisher in a certain way based upon the given flight
schedule, then IndiGo will be related t owhich of the following based upon the same relationship?
a) Spice Jet
b) Quatas
c) Air Asia India
d) Air India
e) None of these

Directions (Q. 6-10): In each question below is given a group of letters followed by four combinations of digits/symbols
numbered (1), (2), (3) and (4). You have to find out which of the combinations correctly represents the group of letters
based on the following coding system and the conditions that follow and mark the number of that combinations as your
answer. If none of the combinations correctly represents the group of letters, mark (e) i.e. ‘none of these’ as your answer.
Letters M A E K J R D W P F Q I U B H

Digit 9 1 2 3 % ✮ 4 5 8 8 6 $ 9 7 ©

Symbol

Code

Conditions:

www.bankingpdf.com
(i) If the first and the second letters are consonants, both are to be coded as the code for the second letter.
(ii) If the first and the third letters are vowels, both are to be coded as the code for the first letter.
(iii) If the first letter is a vowel and the last letter is a consonant, both are to be coded as #.

6). JREMQI
a) ✮✮296$
b) %✮296$
c) %2✮96$
d) %✮296$
e) None of these

7). UBHMED
a) @7©924
b) #7©92#
c) 77©924
d) @@©924
e) None of these

8). AKEJPI
a) 123%8$

www.bankingpdf.com
b) 132%8$
c) 131%8$
d) 113%8$
e) None of these

9). FURIJK
a) 598$%3
b) 99✮$%3
c) 3%$✮@5
d) 89✮$%3
e) None of these

10). QMIAWE
a) 99$152
b) 69$152
c) #9$15#
d) 698156
e) None of these

Answers:

www.bankingpdf.com
1).c 2).d 3).a 4).e 5).d 6).a 7).b 8).c 9).d 10).a

Explanation:
Directions (Q. 1-5):
Day Flight

Monday Vistara

Tuesday IndiGo

Wednesday Air Asia India

Thursday GoAir

Friday Jet Lite

Saturday Kingfisher

Sunday SpiceJet

1).
Answer: c)
2).
Answer: d)

www.bankingpdf.com
3).
Answer: a)
4).
Answer: e)
5).
Answer: d)

6). J  %
R✮
E-2
M9
Q 6
I$
Since first and second letters are consonants (condition i), both of them will be coded as ✮
Hence, code is ‘✮✮296$’
Answer: a)

7). U 9
B7
H©
M9

www.bankingpdf.com
E2
D4
Since first letter is vowel and last letter is consonant (condition iii), both of them will coded as #
Hence, code is ‘#7©92#’
Answer: b)

8). A 1
K3
E2
J%
P8
I $
Since, first and third letters are vowels (condition ii), both of them will be coded as ‘1’
Code is ‘131%8$’
Answer: c)

9). F 8
U9
R✮
I$
J%

www.bankingpdf.com
K3
No conditions applied
Hence, code is ‘89✮$%3’
Answer: d)

10). Q- 6
M9
I $
A1
W 5
E 2
Since, first two letters are consonants (condition i), both of them will be coded 9
Hence, code is ‘99$152’
Answer: a)

www.bankingpdf.com
540-550 Questions

1). In making decisions about important questions, it is desirable to be able to distinguish between ’strong’ arguments and
‘weak’ arguments. ‘Strong’ arguments are those which are both important and directly related to the question. ‘Weak’
arguments are those which are of minor importance and also may not be directly related to the question or may be related
to a trivial aspect of the question.
Statement: Should the govt. deregulate the retail prices of petrol, diesel and cooking gas and allow the prices to be driven
by market conditions?
Arguments: I. yes, this will largely help the oil companies to sell their products at competitive price.
II. No, the general public cannot afford market driven prices of these products.
III. Yes, govt. needs to stop subsidizing these products and channelize the money for developmental projects.
a) Only I and II are strong
b) Only I and III are strong
c) Only II and III are strong
d) All I, II and III are strong
e) None of these

2). In each question below a statement is given followed by two courses of action numbered I and II, A course of action is
a practicable and feasible step or administrative decision to be taken for follow-up, improvement, or further action in
regard to the problem, policy, etc. on the basis of the information given in the statement, you have to assume everything
in the statement to be true, and decide which of the suggested courses of action logically follow(s) for pursuing.

www.bankingpdf.com
Statement: Despite all the efforts by the government, thirty cases of polio were reported in State X as the parents of those
children remained uninformed of this drive against polio.
Courses of action:
I. Strict action should be taken against the parents of children affected from polio.
II. Strict action should be taken against the doctors administering polio drops in the state.
a) If only course of action I follows.
b) If only course of action II follows.
c) If either course of action I or course of action follows
d) If neither course of action I nor course of action II follows.
e) If both courses of action I and II follow.

3). Below in each question are given two statements I and II. These statements may be either independent causes or may
be effects of independent causes or a common cause. One of these statements may be the effect of the other statement.
Read both the statements and decide which of the following answer choice correctly depicts the relationship between
these two statements
(a) The association of management colleges conducted a combined admission exam for all the institutes this year.
(b) As the dates for entrance exam for many management colleges clashed last year, many candidates complained that
they could not appear for a number of entrance exams.
a) If statement(a) is the cause and statement (b) is its effect.
b) If statement(b) is the cause and statement (a) is its effect.
c) If both the statements (a) and (b) are independent causes.

www.bankingpdf.com
d) If both the statements (a) and (b) are effects of independent causes.
e) If both the statements (a) and (b) are effects of some common cause.

Directions (Q. 4-8): In each of the question given below, there are two statements followed by two conclusions numbered
1 and 2.you have to take the given statements to be true even if they seem to be at variance with commonly known facts
and then decide which of the given conclusions logically follow(s) from the given statements.
a) Only (1)conclusion follows
b) Only (2) conclusion follows
c) Either(1) or (2) follows
d) Neither (1) or (2) follows
e) Both (1) and (2) follow

4). Statements: (a)All mangoes have juices


(b) No juice has colour
(c) Some colours have liquid.
Conclusions:
I.Some mangoes have colour.
II.At least some juices have liquid.

5). Statements: (a)Some boys are cricketers.

www.bankingpdf.com
(b)All cricketers are players.
(c) Some players are winners.
Conclusions:
I. Some winners are cricketers.
II. Some players are boys.

6). Statements: (a)Some pencil are pins


(b)all pins are iron.
(c) Some iron is element.
Conclusion:
I.Some element is pins.
II.No iron is pencil.
7). Statements: (a)All fans are AC.
(b)Some AC are cooler.
(c) No wind-mill is AC
Conclusion:
I. No fan is wind-mill.
II. Some coolers are wind-mills.
8). Statements: (a)All apples are Samsung.
(b)No Samsung is Sony.
(c) Some apples are Nokia.
Conclusion:

www.bankingpdf.com
I. Some Samsung are Nokia.
II. No apple is Sony.

Directions (Q. 9-10): A family consist of six members P,Q,R,X,Y and Z.Q is the son of R but R is not mother of Q.P and R
are a married couple, Y is the brother of R. X is the daughter of P.Z is the brother of P.
9).Who is the brother-in-law of R?
a) P
b) Z
c) Y
d) X
e) Indeterminable

10).Who is the father of Q?


a) R
b) P
c) Z
d) None of these
e) Indeterminable

Answers:
1).c 2).d 3).b 4).d 5).b 6).d 7).a 8).e 9).b 10).a

www.bankingpdf.com
Explanation:
1).The prices of petrol, diesel and cooking gas will influence prices of lot of other basic things. Hence, government
subsidize these commodities. Therefore, argument I is not strong.
Argument II is strong as the general public cannot afford market driven prices of these products
Stopping subsidy and channelizing the money for developmental projects is also strong argument.
Answer: c)

2).In the given statement, it is clearly given that parents of those children were uninformed about the drive against polio.
Hence, neither parents nor doctors can be held responsible for this. Instead, the agency or company which had taken up
the responsibility of the marketing of this drive should be held responsible.
Answer: d)

3).Many students were not able to appear for a number of entrance exams as dates of the entrance exams had clashed
last year, so the association of management colleges conducted a combined admission exam for all the institutes this
year.
Hence, statement B is cause and statement A is effect
Answer: b)

www.bankingpdf.com
4).

Conclusion I :This conclusion cannot be drawn because there is no intersection between mango and colour.
Conclusion II : From the Venn diagram, we can see that there is a possibility that no liquid is on juice. Thus, statement II is
not valid always.
Hence, neither conclusion I not II follows.
Answer: d)

5).

Conclusion I: From the Venn diagram we can see that I is not a definite conclusion as there could be a possibility that no
winner is cricketer. Therefore, conclusion I is a possibility but not a definite conclusion.

www.bankingpdf.com
Conclusion II: From the Venn diagram we can see that some players that are cricketers are definitely boys. Hence , only
conclusion II follows.
Answer: b)

6).

Conclusion I :From the Venn diagram we can see that I is not a definite conclusion as there could be a possibility that no
Element is pin. Thus, conclusion I is a possibility but not a definite conclusion.
Conclusion II : From the Venn diagram we can see that some iron(that is pin) has to be pencil. Thus, II is not a definite
conclusion. Thus, conclusion II doesn’t follow. Hence, neither conclusion I nor II follow.
Answer: d)

7).

www.bankingpdf.com
conclusion I : From the Venn diagram we can see that since no Wind-mill is AC, There is no set where there are both
wind-mill and AC. But as all the fans are AC, there cannot be a set which has fan and wind-mill as this set will have to
contain AC. Thus, conclusion I follows.
Conclusion II : From the Venn diagram we can see that, some cooler may or may not be wind-mill. Thus, statement II is
not a definite conclusion. Hence, only conclusion I follows.
Answer: a)

8).

Conclusion I: From the Venn the diagram we can see that some Samsung are Nokia. Thus, conclusion I definitely follows.
Conclusion II : From the Venn diagram we can see that no Apple is Sony. Thus, conclusion II definitely follows. Therefore,
both conclusion I and II follow.
Answer: e)

www.bankingpdf.com
Directions (Q. 9-10): Q is the son of R is not the mother.So, R is the father Q.P is married to R. So, P is the wife of R and
the mother of Q. X is the daughter of P and hence of R and so she is the sister of Q. Y is the brother of R and Z is the
brother of Q.
R is the husband of P and Z is the brother of P. So, Z is the brother-in-law of R.
9).
Answer: b)

10).
Answer: a)

www.bankingpdf.com
550-560 Questions

Directions (Q. 1-5): In the following questions the Symbols @, ©, $ , % and # are used with the following meanings as
illustrated below.
'A $ B' means 'A is not smaller than B'.
'A # B' means 'A is not greater then B'.
'A @ B' means 'A is neither smaller than nor equal to B'.
'A © B' means 'A is neither smaller than nor greater than B'.
'A % B' means 'A is neither greater than nor equal to B'.

1). Statements: K # N, N $ T, T % J
Conclusions:
I. J @ N
II. K @ T
III.T @ K

a) Only I and II follows


b) Only II and III follow
c) Only I and III follow
d) None follows
e) None of these

2). Statements: H % J, J © N, N @ R

www.bankingpdf.com
Conclusions:
I. R % J
II. H @ J
III.N @ H

a) Only II follows
b) b) Only I and III follow
c) Only I follow
d) d) Only III follows
e) None of these

3). Statements : B © K, K # L, L @ P
Conclusions:
I. P @ B
II. K # P
III.L $ B

a) Only II follows
b) Only I and II follows
c) c) Only III follows
d) d) Only II and III follow
e) None of these

www.bankingpdf.com
4). Statements: M @ J, J $ T, T © N
Conclusions:
I. N # J
II. T % M
III.M @ N

a) Only I and II follows


b) b) Only II and III follow
c) Only I and III follow
d) None follows
e) All follows

5). Statements: D © K, K # F, F @ P
Conclusions:
I. P @ D
II. K # P
III.F $ D

a) Only II follows
b) Only I and II follows
c) Only III follows
d) Only II and III follow
e) None of these

www.bankingpdf.com
Directions (Q. 6-10): Study the following information and answer the questions given below:

Eight persons Asgar, Ashis, Ashok, Ashraf, Ashu, Arya, Akash and Akhil are playing a game of musical chairs. The chairs
placed around a circular table face away from the table) each person knows a different language – Telugu, Urdu, Marathi,
Nepali, Kashmiri, Tamil, Bengali and Kannada – but not necessarily in the same order. When all of them are sitting in the
chairs during the game, the following conditions were fulfilled.
Ashok is a Bengali speaker and sits third to the left of Ashu.
The Tamil and the Nepali speaker are immediate neighbours of each other.
Neither the Nepali nor the Tamil speaker is an immediate neighbour of either Ashu or Ashok.
The Urdu and the Kannada speaker are immediate neighbours of each other.
Neither Ashu nor Ashraf speaks Kannada)
The one who speaks Nepali sits to the left of Ashis, who speaks Marathi.
Asgar speaks Urdu and is not an immediate neighbour of the Bengali speaker.
Only Arya sits between the Nepali and the Telugu speaker.
Akash sits third to the left of the person who speaks Kannada)

6). Ashu speaks which of the following languages?


a) Tamil
b) Kashmiri
c) Nepali
d) Telugu
e) None of these

www.bankingpdf.com
7). Which of the following pairs represents the immediate neighbours of the Telugu speaker?
a) Nepali and Kashmiri
b) Kannada and Nepali
c) Urdu and Kashmiri
d) Marathi and Tamil
e) None of these

8). Who among the following speaks Kannada?


a) Akash
b) Akhil
c) Arya
d) Ashu
e) None of these

9). Who among the following sits exactly between Ashis and Akhil?
a) Urdu speaker
b) Tamil speaker
c) Nepali speaker
d) Telugu speaker
e) Bengali speaker

www.bankingpdf.com
10). How many persons sit between the Bengali speaker and Ashraf when counted in clockwise direction from the Bengali
speaker?
a) None
b) One
c) Two
d) Three
e) Four

Answers:
1)e 2)b 3)c 4)e 5)e 6)b 7)e 8)b 9)e 10)d

Explanation:
Directions (Q. 1-5):
'A $ B' means 'A ≥ B'.
'A % B' means 'A ≤ B'.
'A @ B' means 'A > B'.
'A * B' means 'A = B'.
'A # B' means 'A < B'.

1). K = N , N = T , T > J
J>N

www.bankingpdf.com
K>T
T>K
None follows
Answer: e)

2). H < J , J = N , N > R


R < J follows
H > J not follows
N > H follows
Answer: b)

3). B = K , K = L , L > P
P > B not follows
K = P not follows
L = B follows.
Answer: e)

4). M > J , J = T , T = N
N=J
T<M

www.bankingpdf.com
M>N
All follows.
Answer: e)

5). D = K , K = F , F > P
P > D not follows
K = P not follows
F = D follows
Answer: e)

Directions (Q. 6-10):

www.bankingpdf.com
560-570 Questions

Directions (Q. 1-5): Study the following information to answer the given questions.
Four men A, B, C and D and their respective wives P, Q, R and S went for a dinner and are sitting around a
circular table. No two men or women sit adjacent to each other. A is the only person sitting adjacent to his wife and
sits to her left. S is a photographer and P wants her to sit exactly opposite to her. B, R and S are schoolmates and sit
on consecutive seats.

www.bankingpdf.com
1). How many such arrangements are possible?
a) 2
b) 4
c) 1
d) 3
e) More than 5

2). Who sits second to the right of A?


a) D
b) C
c) B
d) B or D
e) C or D

3). Which of the following is definitely false?


a) Q sits directly opposite to R
b) S sits adjacent to C
c) R sits to the left of A
d) D sits third to the right of S
e) None of these

www.bankingpdf.com
4). C and his wife had to attend another function so they left early. The others were still seated at their original
places. What is the position of S with respect to A now?
a) Cannot be determined
b) Third to the left
c) Third to the right
d) Second to the right
e) Second to the left

5). C and his wife had to attend another function so they left early. The others were still seated at their original
places. If the photographer takes a snap of the remaining five such that they remain seated at their places, which
three of them appear in the middle?
a) APQ
b) PDQ
c) APD
d) PQB
e) Cannot be determined

Directions (Q. 6-10): Five Scientists involved in an international Research Project hold a Quarterly Review Meeting in
Singapore. There are certain limitations on their language skills. Expert S1 knows only Japanese and Hindi; S2 is
good at Japanese and English; S3 is good at English and Hindi: S4 knows French and Japanese quite well; and S5,
an Indian, knows Hindi, English and French.

www.bankingpdf.com
6). Besides S5, which of the following can converse with S4 without an interpreter?
a) Only S1
b) Only S2
c) Only S3
d) Both S1 and S2
e) None of these

7). Which of the following cannot converse without an interpreter?


a) S2 and S5
b) S1 and S2
c) S1 and S3
d) S3 and S4
e) None of these

8). Choose the language that is least commonly used at the meeting.
a) English
b) French
c) Japanese
d) Hindi
e) None of these

www.bankingpdf.com
9). Which of the following can act as an interpreter when S3 and S4 wish to discuss?
a) Only S1
b) Only S2
c) Only S5
d) Only S4
e) None of these

10). Suppose a sixth Expert S6 joins into chair the session so that maximum number of the earlier five is able to
understand him, he should be fluent in:
a) English & French
b) Japanese & Hindi
c) English & Hindi
d) All of the above
e) None of these

Answers with Detailed Explanation:


Let us draw a circle with 8 points and fix A and P on it. Also S sits directly opposite to P. B, R and S sit on
consecutive seats. Since no two men or women sit next to each other, B must sit between R and S. no two couples
except AP sit adjacent to each other, so we have two possible cases of arrangement as shown in the figure.

www.bankingpdf.com
1). Two such arrangements are possible.

www.bankingpdf.com
Answer : a

2). In either case, D sits second to the right of A.


Answer : a

3). In both the cases, D sits third to the left of S.


Answer : d

4). In either case, H was sitting third to the left of A.


In case 1, C moves from his place and hence S is now second to the left of A.
In case 2, R moves from her place and hence S is now second to the left of A.
Answer : e

5). After C and R leave, the people around S, the photographer, are BDPAQ or QDPAB (taken clockwise). So in
either case D, P and A appear in the middle.
Answer : c

6). The data can be shown as in the table:


Japanese Hindi French English

www.bankingpdf.com
S1 Yes Yes

S2 Yes Yes

S3 Yes Yes

S4 Yes Yes

S5 Yes Yes Yes

S4 knows Japanese and French. So, anyone who knows either of these languages can converse with S4.
So, S5, S1 and S2 can converse with S4 without any interpreter.
Answer : d

7). S2 and S5-Can speak in English


S1 and S2 –Can speak in Japanese
S1 and S3 –Can speak in Hindi
S3 and S4 –Don’t have a common language, so need and interpreter.
Answer : d

8). The language that is least common is French.


Answer : b

www.bankingpdf.com
9). When S3 and S4 wish to discuss, S1, S2 and S5 can all be interpreters.
So, none of these because the options say, only S1, only S2 and only S5.
Answer : e

10). S6 joins the chair and maximum no. of people understand him.
English and French –S2, S3, S4 and S5.
Japanese and Hindi – S1, S2, S3, S4 and S5
English and Hindi – S1, S2, S3, and S5
So, Japanese and Hindi has maximum audience.
Answer : b

www.bankingpdf.com
570-580 Questions

1).Two statements (A) and (B) are given. These statements may be either independent causes or may be effects of
independent causes or a common cause. One of these statements may be ‘the effect of the other statement. Read both
the statements and decide which of the following answer choice correctly depicts relationship between these two
statements.
(A)The road traffic between the two towns in the state has been disrupted since last week.
(B)The rail traffic between the two towns in the state has been disrupted since last week.
a) If statement (A) is the cause and statement (B) is its effect.
b) If statement (B) is the cause and statement (A) is its effect.
c) If both the statements (A) and (B) are independent causes.
d) If both the statements (A) and (B) are effects of independent causes.
e) If both the statements (A) and (B) are effects of some common cause.
2).Each question given below consists of a statement, followed by two arguments I and II. You have to decide which of
the arguments is a ‘strong’ argument and which is a ‘weak’ argument.
Statement: Should one close relative of a retiring government employee be given a job in government in India?
Arguments:
I. Yes, where else will the relative get a job like this?
II. No, it will close doors of government service to competent and needy youth.
a) If only argument I is strong.
b) If only argument II is strong.
c) If either I or II strong.

www.bankingpdf.com
d) If neither I nor II strong.
e) If both the arguments are strong
3). Statement: Should purchase of gold by individuals be restricted in India to improve its foreign exchange position?
Arguments:
I. Yes, interference on customers’ right and freedom is desirable.
II. No, business interest has to be guarded first.
a) If only argument I is strong.
b) If only argument II is strong.
c) If either I or II strong.
d) If neither I nor II strong.
e) If both the arguments are strong.
4). In the question below, a statement followed by three courses of action numbered I,II and III. A course of action is a
step or administrative decision to be taken for improvement, follow up or further action in regard to the problem, policy,
etc. On the basis of the information given in the statement, you have to assume everything in the statement to be true,
then decide which of the suggested courses of action logically follow(s) for pursuing.
Statement: Large number of students of the local school fell ill after having their midday meal provided by the school.
Course of action: I. The govt. should immediately suspend the school Principal.
II. The school authority should initiate an enquiry to find out the reason for student’s sickness.
III. The govt. should instruct all the schools in the state to suspend their
midday meal scheme till further order.
a) Only I follows
b) Only II follows

www.bankingpdf.com
c) Only III follows
d) Only II and III follow
e) None of these
Directions (Q. 5-8): Answer the questions on the basis of information given below.
Five business leaders, Bill Gates, Donald Trump, Jack Welch, Laxmi Mittal, and Warren Buffet, were being judged for the
award of Business leader of the year. The selection committee ranked them on five different parameters viz. empathy,
vision, focus, creativity ,and intuition. The five leaders were given points on these five parameters. The topper gets 5 and
the least gets 1 point. There are no ties. The person with highest total gets the award. Bill Gates gets the award by
scoring 24. Warren Buffet gets 5 points in creativity and 3 points in intuition. Jack Welch got consistent score in 4 of the
parameters. Their final standings were in alphabetical order of their names.
5).What is the overall score of Jack Welch?
a) 12
b) 13
c) 14
d) Cannot be determined
e) None of these
6).What is the rank of Donald Trump in creativity?
a) 1st
b) 2nd
c) 5th
d) Cannot be determined
e) None of these

www.bankingpdf.com
7).what are the points scored by Jack Welch in intuition?
a) 1
b) 2
c) 4
d) Cannot be determined
e) None of these
8).If Laxmi Mittal scored his highest points in Empathy then what is the score of Donald Trump In vision?
a) 1
b) 2
c) 3
d) 4
e) None of these
9).In the question below, a statement followed by three courses of action numbered I,II and III.A course of action is a step
or administrative decision to be taken for improvement, follow up or further action in regard to the problem, policy, etc. On
the basis of the information given in the statement, you have to assume everything in the statement to be true, hen
decide which of the suggested courses of action logically follow(s) for pursuing.
Statement: Many teachers of the local school have tendered their resignation to the Principal to protest against the
management’s decision of not implementing revised pay scales.
Course of action: I. The school management should accept the resignations and appoint new teachers.
II. The school management should persuade the teacher to continue with an assurance of considering the pay hike issue.
III. The school management should approach the govt. for guidelines.
a) None follows

www.bankingpdf.com
b) Only either I or II follows
c) Only III follows
d) Only I follows
e) Only II follow
10). Which of the following expressions is not necessarily true given the expression ‘P ≤ Q = R > S ≥ T’ is definitely true?
a) Q ≥ P
b) S < R
c) P ≥ S
d) R > T
e) None of these
Explanation:
1). Bad weather or protests might be the reason for disruption of road traffic and rail traffic between two towns. Hence
given statements are effects of some common cause.
Answer: e)
2). I is weak because we can’t vitiate the system for one individual. II is strong. As the seats will be filled by close relatives
of government employees, competent and other needy youths won’t get entry for govt. services.
Answer: b)
3). I is weak because such interference cannot be desirable in a democracy, II is weak because it gives priority to
business interest on foreign exchange position, which would not be in the interest of the country.
Answer: d)
4). Enquiry should be initiated immediately to identify the reason for students falling ill, so that corrective measures can be
taken to avoid such incidents in future. Hence statement II is a valid action.

www.bankingpdf.com
Statement III is not valid and is extreme step to stop the program in all schools as it is unlikely that the same source can
be the provider of food for all schools In a region. Unless that is known, III is an extreme step and not a valid course of
action.
Answer: b)

Directions(5-8):

5).
Answer: b)
6).
Answer: c)

www.bankingpdf.com
7).
Answer: a)
8).
Answer: d)
9).Accepting all the resignations does not ensure that issue will be solved. Newly appointed teachers might also expect
higher salaries. Hence the statement is not a valid course of action.
Approaching government for guidelines does not make sense as government already has certain guidelines in place for
educational institutes.
By providing an assurance, the school management can then discuss the issue with the stakeholders and arrive at a
decision. In the interim, the school operations are also not affected.
Answer: e)
10). Relationship between P and S cannot be determined.
Answer: c)

www.bankingpdf.com
580-590 Questions

Directions (Q. 1-5): Study the following information carefully to answer the given questions.
A word and number arrangement machine when given an input line of words and numbers rearranges them following a
particular rule in each step. The following is an illustration of the input and its rearrangement.
Input: song 72 excel owl 28 11 power 81 40 cut
Step I: excel song 72 owl 28 power 81 40 cut 11
Step II: excel owl song 72 power 81 40 cut 11 28
Step III: excel owl cut song 72 power 81 11 28 40
Step IV: excel owl cut power song 81 11 28 40 72
Step V: excel owl cut power song 11 28 40 72 81
Step V is the last step of the above input. As per the rules followed in the above steps, find out in each of the following
questions, the appropriate step for the given input below and answer the questions based on it.

Input: collection 53 vitamin 49 33 article 45 application 23 ice 69 encourage best

1). What will be the position of ‘69’ in Step IV?


a) 9th from the left
b) Fifth from the right
c) Extreme left
d) Extreme right

www.bankingpdf.com
e) None of these

2). Which step would be the following output? ‘application article encourage collection 53 vitamin 49 ice 69 best 23 33 45’
a) IV
b) V
c) VI
d) III
e) VII

3). How many steps will be required to complete the arrangement of the above input?
a) Four
b) Five
c) Six
d) Seven
e) None of these

4). Which of the following steps would be the last step but one?
a) V
b) VI
c) VII
d) IV
e) None of these

www.bankingpdf.com
5). Which word/number would be at the 9th position from the right end in Step V?
a) collection
b) vitamin
c) best
d) 53
e) 49

Directions (Q. 6–10): Read the following information carefully and answer the questions which follow.
Eight persons – Balan, Balbir, Baldev, Balwant, Bansi, Barun, Bashir and Bimal – are sitting around a circular table facing
away from the centre, Each of them likes a movie of a different actor, viz Rajnikanth, Amitabh Bachchan, Mohan Lal,
Mammooty, Kamal Haasan, Amir Khan, Chiranjeevi and Mahesh Babu, but not necessarily in the same order. Balan, who
likes Mammooty, sits third to the right of Bansi. The one who likes Mahesh Babu sits second to the left of Balwant, who
likes the movie of Rajnikanth. Baldev, who likes Amitabh Bachchan, sits between the persons who like Amir Khan and
Mahesh Babu. Balwant sits second to the left of Balbir, who cannot sit adjacent to Bansi. Bashir likes Chiranjeevi and sits
second to the right of Bimal, who likes Mohan Lal.

6). What is the position of Balwant with respect to the person who likes the movie of Amitabh Bachchan?
a) Third to the left
b) Third to the right
c) Second to the right
d) Fifth to the right
e) Fourth to the left

www.bankingpdf.com
7). Which of the following combinations is definitely true?
a) Balan – Mahesh Babu
b) Bimal – Rajnikanth
c) Bansi – Chiranjeevi
d) Barun –Mammooty
e) Balbir – Kamal Haasan

8). How many persons are sitting between Bansi and the person who likes the movie acted by Mammooty (if counted from
Bansi in clockwise direction)?
a) One
b) Two
c) Three
d) None
e) More than three

9). Who likes the movie of Amir Khan?


a) Balbir
b) Balwant
c) Bansi
d) Barun
e) Bimal

www.bankingpdf.com
10). Balwant is related to ‘Mahesh Babu’ and Bashir is related to Mohan Lal. In the same way, Barun is related to whom
among the following?
a) Rajnikanth
b) Amir Khan
c) Chiranjeevi
d) Amitabh Bachchan
e) Mammooty

Answers:

1)e 2)d 3)c 4)a 5)c 6)b 7)e 8)b 9)c 10)b

590-600 Questions

1). Statements: All cells are bins.


All bins are petals.
No petal is root.
All roots are leafs.
Conclusions:

www.bankingpdf.com
I. No cell is bin.
II. All leafs is root.
III. All cells are petals.
IV. All leafs are roots.

a) Only I and II follow


b) Only III follow
c) Only I, II and III follow
d) All follows
e) None of these

2). Statements: All poles are fans.


All fans are stands.
Some stands are lines.
Some lines are boxes.
Conclusions:
I. Some boxes are poles.
II. Some fans are boxes.
III. Some lines are poles.
IV. Some lines are fans.

a) None follows
b) Only I and II follow

www.bankingpdf.com
c) Only II and IV follow
d) Only III and IV follow
e) Only I, II and IV follow

3). Statements: Some scales are weights.


All weights are metals.
Some metals are rings.
All rings are bands.
Conclusions:
I. Some bands are scales.
II. Some weights are bands.
III. Some rings are scales.
IV. Some metals are scales.

a) Only I and III follow


b) Only I and II follow
c) Only II and III follow
d) Only II and IV follow
e) None of these

4). Statements: Some tools are seeds.


Some seeds are cycles.
Some cycles are tubes.

www.bankingpdf.com
Some tubes are rains.
Conclusions:
I. Some tubes are seeds.
II. Some cycles are tools.
III. No seed is tube.
IV. Some rains are cycles.

a) Only I follows
b) Only either I or III follows
c) Only I and II follow
d) Only either I or III and IV follow
e) None of these

5). Statements: All stones are pearls.


Some pearls are shells.
Some shells are boxes.
No box is container.
Conclusions:
I. Some stones are shells.
II. No pearl is container.
III. No shell is container.
IV. Some pearls are containers.

www.bankingpdf.com
a) Only II follows
b) Only II & III follow
c) Only either II or IV follows
d) Only III follows
e) None follows

Directions (Q. Nos. 6-10) Study the following information carefully and answer the questions given below it.
Digits in the numbers are to be coded as follows
Digits 5 3 6 4 9 2 1 8 7

Codes Z P C H K V O R L

Conditions
(i) If the first as well as the last digits are even, both are to the coded by the code for the first digit.
(ii) If the first as well as the last digits are odd, both are to be coded by the code for the last digit.

6). 397416
a) PKLHOP
b) CKLHOC
c) PVLHOC
d) PKLHOC

www.bankingpdf.com
e) None of these

7). 562183
a) PCVORP
b) ZCVORZ
c) PCVORZ
d) ZCVORP
e) None of these

8). 734192
a) ZPHOKV
b) LPCOKV
c) LPHOVK
d) LPHOKV
e) None of these

9). 627851
a) PULRZO
b) CVLZRO
c) CVLRZO
d) CKLRZO
e) None of these

www.bankingpdf.com
10). 812354
a) RLVPZH
b) HOVPZH
c) ROVPZR
d) HOVPRZ
e) None of these

Answers:
1)b 2)a 3)e 4)b 5)c 6)d 7)a 8)d 9)c 10)c

Explanation:

1).

www.bankingpdf.com
Answer: b)

2).

Answer: a)

3).

www.bankingpdf.com
Answer: e)

4).

Answer: b)

5).

www.bankingpdf.com
Answer: c)

6).

(no condition follows)


Answer: d)
7).

www.bankingpdf.com
(condition number (ii) follows)
Answer: a)

8).

(no condition follows)


Answer: d)

9).

(no condition follows)


Answer: c)

10).

www.bankingpdf.com
(condition number (i) follows)
Answer: c)

600-610 Questions

Directions (Q.1-5): In the following questions the symbols +,*,?,@ and $ are used with the following meanings:
A + B means A is neither smaller nor greater than B.
A × B means A is neither equal to nor smaller than B.
A ? B means A is neither greater than nor equal to B.
A @ B means A is either greater than or equal to B.
A $ B means A is not equal to B.
Now in each of the following questions assuming the given statements to be true, find which of the two conclusions I
and II given below them is/are definitely true.
a) If only conclusion I is true;
b) If only conclusion II is true;
c) If either I or II is true;
d) If neither I nor II is true; and
e) If both I and II are true.

www.bankingpdf.com
1). Statements : S$T,T× U,S + U
Conclusions : I.T × S II. S ? U

2). Statements: P + Q, Q $ R,R ? P


Conclusions : I.R $ P II.Q+ R

3).Statements : U @ V,V × Q,Q $ P


Conclusions: I.U? Q II. U ? P

4).Statements : A × B,B@L,L + N
Conclusions : I.N + B II. A × L

5).Statements : H @ J, J ? K,K × M
Conclusions : I.H @ M II. M × J

6).Which of the following expressions is not necessarily true given the expression ‘E ≤ F = G> H ≥ I ’is definitely true?
a) F ≥ E
b) H < G

www.bankingpdf.com
c) E ≥ H
d) G > I
e) None of these

7).Which of the following should be placed in blank spaces respectively (in the same order from left to right) in order
to complete the given expression in such a manner that makes the expression L < N definitely true--------- < ------
- ≥ ----------- > ---------
a) L,M,N,O
b) N,M,O,L
c) M,O,N,L
d) L,O,N,M
e) None of these

8).which of the following symbols should replace the question mark in the given expression in order to make the
expressions ‘A > B’ and ‘C ≤ D’ definitely true?
A > D =E? B ≥ C < F
a) >
b) <
c) ≤
d) Either < or ≤

www.bankingpdf.com
e) =

9).In which of the following expressions will the expression ‘K > N ‘ can be true?
a) K < L ≥ M=N
b) K ≤ M ≤ N < L
c) L = N > M ≥ K
d) N > L ≥ M < K
e) N < L ≤ M < K

10). If the expressions ‘A ≤ D > E > B’,’A > C ‘ and ‘B > F’ are true, which of the following conclusions will be
definitely false?
a) F < D
b) D > C
c) E < F
d) B < E
e) None of the above

Answers:
1)e 2)a 3)d 4)b 5)d 6)c 7)c 8)e 9)e 10)c

www.bankingpdf.com
Explanation:
1). S ! =T….(i), T> U …(ii),S= U….(iii) from (ii) and (iii),we get T > U = S. Hence T > S .Hence both I and II are true.
Answer: e)

2).P= Q….(i) Q is not equal to R….(ii),R < P…..(iii),conclusion I is true. II contradicts statement (ii),hence it is not
true.
Answer: a)

3). U ≥ V…(i), V > Q….(ii), Q is not equal to P….(iii) from (i) and (ii),we get U ≥ V > Q. Hence, U > Q….(A) Hence I is
not true. from (iii),two possible relationships between P and Q are;
Case I : When P > Q
Now, using(A),we get U>Q<P. Hence, no conclusion.
Case II : When Q > P
Using (A),we get U > Q > P. Hence,U>P.
Hence II is not true.
Answer: d)

www.bankingpdf.com
4).A > B….(i) B ≥ L….(ii), L = N…..(iii) from (ii) & (iii),we get B ≥ L =N Hence, B≥ N or N≤ B. Hence I may be true but
not necessarily so. From (i) and (ii),we get A > F ≥ L. Hence,A > L Hence II is true.
Answer: b)

5). H ≥ J….(i),J < K….(ii),K > M…(iii)


From (i) and (iii),we get J < K > M. Hence, no relationship between J and M can be established. Hence II can’t be
established.
Again, combining all we can’t conclude the relationship between H and M.
Hence I is not true.
Answer: d)

6).option 1 is true
Option 2 is true
Option 3 is false since relationship between P and S cannot be determined
Option 4 is also true
Answer: c)

7). Consider option 1:


L < M ≥ N > O : relationship between L and N cannot be determined

www.bankingpdf.com
Consider option 2:
N < M ≥ O >L :relationship between L and N cannot be determined
Consider option 3:
M < O ≥ N > L :since N > L,expression L < N is definitely true.
Answer: c)

8).consider expression,A > D =E = B ≥ C < F


Then it can concluded that both the expression ‘A > B’ and ‘C ≤ D’ are definitely true.
Hence,
Answer: e)

9).consider option 1 :Relationship between K and N cannot be determined


Consider option 2 :’K > N’ is definitely false
Consider option 3 :’K > N’ is definitely false
Consider option 4 :Relationship between K and N cannot be determined
Consider option 5 :’K > N’ is definitely true
Hence,
Answer: e)

www.bankingpdf.com
10).Combining the given expressions,
‘C < A ≤ D > E > B > F’
Considering the above expression
Option 1 is true
Option 2 is true
Option 3 is false
Option 4 is true
Answer: c)

610-620 Questions

Direction ( Q. 1-5): A pattern of random words is given as input. Certain rules are followed for rearranging this message to
produce sequential outputs. You are therefore required to figure out the pattern followed for rearrangement and then
answer the given questions.
Note: Treat each word as individual elements of the input pattern.
Input: protect you we at every step of life.
Step-1: you protect we at every step of life.
Step-2: you we protect at every step of life.
Step- 3: you we step protect at every of life.

www.bankingpdf.com
Step-4: you we step protect of at every life.
Step-5: you we step protect of life at every.
Step-6: you we step protect of life every at.
For the following input:

1). Input: kithne log the re bola sambha


Which of the following step would be: the sambha re kithne log bola
a) Step-2
b) Step-3
c) Step-4
d) Step-5
e) Step-1

2). Which of the following will be step – 4 for the giving input?
Input: all men have four heads
a) Men heads four have all
b) Men heads four all have
c) Men heads have four all
d) Men have all four heads
e) None of these

www.bankingpdf.com
3). For the following input, namely
Input: chandni raath diya mirza mere saath
Which of the following will be step-5?
a) Saath mere raath mirza diya chandni
b) Saath raath mirza mere diya chandni
c) Saath raath mirza mere chandni diya
d) Mera raath diya mirza saath chandni
e) None of these

4). Step-3 of a certain input is as follows,


shashi balijeet prashant dheeraj payal chandar anish
Which of the following will be the original input?
a) Prashant payal dheeraj baljeet chandar Shashi anish
b) Dheeraj Shashi baljeet payal prashant chandar anish
c) Dheeraj Shashi prashanth payal chandar anish baljeet
d) Dheeraj Shashi baljeet prashanth payal chandar anish
e) Cannot be determined

5). Which of the following will be step-3 for the given input?

www.bankingpdf.com
Input: giant walrus is found in polar regions
a) Walrus polar regions giant is found in
b) Regions polar walrus giant in is found
c) Polar regions giant walrus is in found
d) Walrus regions polar giant is found in
e) None of these

Directions (Q. 6-10): In each of the question below are given four statements followed by four conclusions numbered I, II,
III. And IV.
You have to take the given statements to be true even if they seem to be at variance from commonly known facts. Read
all the conclusions and then decide which of the given conclusions logically follows from the given statements
disregarding commonly known facts.

6). Statements : All computers are machines.


All machines are printers.
Some printers are keys.
No key is board.
Conclusions;
I. Some keys are machines.
II. Some boards are printers.
III. All printers are machines.
IV. Some boards are computers.

www.bankingpdf.com
a) None follows
b) Only I & II follows
c) Only III follows
d) Only either II or IV follows
e) None of these

7). Statements : Some coins are metals.


Some metals are papers.
Some papers are clothes.
Some clothes are dress.
Conclusion:
I. Some dress are papers
II. Some clothes are coins.
III. Some dress are metals.
IV. No cloths is coin.
a) Only I follows
b) Only either I or III follows
c) Only either II or IV follows
d) Only II & IV follows
e) None of these

www.bankingpdf.com
8). Statements : All figures are toys.
Some toys are fames.
All frames are kites.
No kite is doll.
Conclusions :
I. Some dolls are toys.
II. Some kites are toys.
III. No doll is toy
IV. Some frames are figures.
a) None follows
b) Only II follows
c) Only either I or III follows
d) Only either I or III and II follow
e) Only either I or III and II and IV follows

9). Statements : All shops are markets.


All goods are markets.
All bottles are shops.
Some markets are huts.
Conclusion:
I. Some huts are bottles.

www.bankingpdf.com
II. Some goods are bottles
III. All bottles are markets.
IV. Some shops are huts.
a) Only I and III follow
b) Only III follows
c) Only II, III and IV follows
d) Only I, III and IV follow
e) None of these

10). Statements: Some books are papers.


Some papers are roads.
Some roads are lanes.
Some lanes are streets.
Conclusions:
I. Some streets are roads.
II. Some lanes are books
III. Some streets are books
IV. Some roads are books.
a) None follows
b) Only I and II follows
c) Only III follows

www.bankingpdf.com
d) Only IV follows
e) Only II follows

Answers:

1)b 2)c 3)b 4)e 5)d 6)a 7)c 8)d 9)b 10)a

Explanation:
1). The word are arranging themselves in reverse alphabetical order from Z to A. At a time only one word is changing in
every step. The same logic is to be applied for all the questions in sets
Step I: the kithne log re bola sambha
Step II: the sambha kithne log re bola
Step III: the sambha re kithne log bola
Answer: b)

2). The words are arranging themselves in reverse alphabetical order from Z to A. At a time only one word is changing in
every step. The same logic is to be applied for all the questions in sets
Step I: men all have four heads
Step II: men heads all have four
Step III: men heads have all four

www.bankingpdf.com
Step IV: men heads have four all
Answer: c)

3). The words are arranging themselves in reverse alphabetical order from Z to A. At a time only one word is changing in
every step. The same logic is to be applied for all the questions in sets
Step I: saath chandni raath diya mirza mere
Step II: saath raath chandni diya mirza mere
Step III: saath raath mirza chandni diya mere
Step IV: saath raath mirza mere chandni diya
Step V: saath raath mirza mera diya chandni
Answer: b)

4). The words are arranging themselves in reverse alphabetical order from Z to A. At a time only one word is changing in
every step. The same logic is to be applied for all the questions in sets
Answer: e)

5). The words are arranging themselves in reverse alphabetical order from Z to A. At a time only one word is changing in
every step. The same logic is to be applied for all the questions in sets
Step I: walrus giant is found in polar regions
Step II: walrus regions giant is found in polar

www.bankingpdf.com
Step III: walrus regions polar giant is found in
Answer: d)

6).

Answer: a)

7). There are two cases possible.

www.bankingpdf.com
Answer: c)

8). There are two possible cases:

www.bankingpdf.com
In either of these cases, some kites are toys.
Answer: d)

9).

www.bankingpdf.com
Answer: b)

10).

Answer: a)

www.bankingpdf.com
620-630 Questions

Directions (Q.1-2): These questions are based on the following information:


‘X $ Y’ means ‘X is father of Y’.
‘X # Y’ means ‘X is son of Y’.
‘X @ Y’ means ‘X is sister of Y’.
‘X % Y’ means ‘X is wife of Y’.

1). M#K$T@ N represents what relationship of M with N?


a) Sister
b) Cousin
c) Brother
d) Paternal Uncle
e) None of these

2). Which of the following expressions represents the relationship “V is mother of D”?
a) A%B$R@D
b) A%B@R$D
c) D#T@J$A

www.bankingpdf.com
d) A@F$D%M
e) None of these

3). Riyaz walks five km west, then turns 1800 in clockwise direction and then walks 2km, and then he again turns 45 0 in
anti-clockwise direction and walks √18 km. what is the shortest distance between starting point and destination?
a) 5 km
b) 2.45km
c) 3.5km
d) 3km
e) 3.2km

4). A Man rode his bicycle Northward, then turned left and road 1 km and again turned left and rode 2 km. He found
himself 1 km west of his starting point. How far did he ride northward initially?
a) 1 km
b) 2 km
c) 3 km
d) 5 km
e) 4 km

www.bankingpdf.com
5). How many such pairs of letters are there in word FOREIGN each of which has as many letters between them in the
word as in the English alphabet?
a) None
b) One
c) Two
d) Three
e) More than three

6). If ‘P÷ Q’ means ‘subtract Q from P’; ‘P + Q’ means ‘multiply P by Q, ‘P – Q ’ means ‘add Q to P’ and ‘ P × Q’ means ‘
divide P by Q’ then 5-24×4+2 ÷ 3 = ?
a) 14
b) 15
c) 5
d) 19
e) None of these

7). In a certain code CERTAIN is coded as XVIGZRM. How is MUNDANE coded in that language?
a) NFMWZMV
b) VMZWMFN
c) NFMWZMX
d) NFMXZMV

www.bankingpdf.com
e) VMWZFMN

8). In a certain code CHAIR is written as EGCHT. How is AUDIT written in that code?
a) CTFHV
b) CSFHV
c) BTFHV
d) CTEHV
e) BFTVH

9). In a certain code CALANDER is written as CLANAEDR. How is CIRCULAR written in that code?
a) ICCRLURA
b) CRIUCALR
c) CRIUCLARA
d) CRUICLAR
e) None of these

10). In a certain code, a number 13479 is written as AQFJL and 2568 is written as DMPN. How is 396824 written in that
code?
a) QLPNMJ
b) QLPNMF

www.bankingpdf.com
c) QLPMNF
d) QLPNDF
e) QPLMJD

Answers:
1)c 2)a 3)d 4)b 5)c 6)a 7)a 8)a 9)b 10)d

Explanation:
1). M is son of K who is father of T and T is sister of N.
Thus K is father of M, T and N. or M is brother of N
Answer: c)

2). A is wife of B, who is father of R and R is sister of D.


Hence A is mother of D.
Answer: a)

3).The figure shows the movement by Riyaz.

www.bankingpdf.com
Since BD=AB√2 and angle ABD is 45 degrees, AD must be perpendicular to AB.
Triangle ABD is right isosceles triangle.
Hence AD=3km
Answer: d)

4).

The Man road 2 km. Northward.


Answer: b)

www.bankingpdf.com
5). There are two pairs : EG,RN.
Answer: c)

6). 5-24 ×4+2÷3 = 5 + 24 ÷ 4 × 2 – 3(in the coded language)


=5 + 6 × 2 – 3 = 5 + 12 – 3 = 14
Answer: a)

7). A→Z, B → Y, C→X,……,Z →A.

Answer: a)

8). 1st ,3rd and 5th letters move +2 steps.


2 nd and 4 th letters move -1 step.

www.bankingpdf.com
Answer: a)

9). 2nd letter ↔ 3rd letter, 4th letter ↔ 5th letter


6th letter ↔ 7th letter

Answer: b)

10).

Answer: d)

www.bankingpdf.com
630 – 640 Questions

Directions (Q. 1 - 5): Study the following information carefully and answer the given questions.
P, Q, R, S, T, U, V and W are eight members of a family. They belong to three different generations.
There are three married couples. All of them are sitting around a circular table, facing the centre but not necessarily
in the same order.
S and U are married couple, S is the wife of U and sits second to the left of her husband.
R and P are offspring of S ( R and P are not immediate neighbours of S).
There is only one person sitting between R and his niece V, but that person is not V’s father.
T, a bachelor, sits third to the right of his uncle P, but neither to the opposite nor to the immediate left of his father.
V is not an immediate neighbour of her aunt Q. No three females are sitting together.
R and his sister-in-law are immediate neighbour.

1). Who among the following is V’s father?


a) U
b) P
c) W
d) Q
e) Can’t be determined

2). How many females are there in the family?

www.bankingpdf.com
a) Two
b) Three
c) Five
d) Can’t be determined
e) None of these

3). What is the position of S with respect to her grand-daughter?


a) Second to the left
b) Fourth to the right
c) Third to the right
d) Fourth to the left
e) Immediate left

4). Who among the following is the aunt of T?


a) Q
b) S
c) V
d) W
e) Can’t be determined

5). Who among the following is sitting opposite his wife?


a) U

www.bankingpdf.com
b) R
c) P
d) T
e) None of these

Direction (6-10): In the following questions, the symbol ×, ∂, %, ©, @, are used with the following illustrations.
P % Q means P is not smaller than Q
P © Q means P is neither smaller than nor equal to Q
P x Q means P is neither greater than nor equal to Q
P ∂ Q means P is not greater than Q
P @ Q means P is neither greater than nor smaller than Q

6). Statement – R ∂ K ; K x M ; M @ J
Conclusions:
I. J©K
II. M © R
III. R x J

a) Only I & II follows


b) Only II & III follows
c) Only I & III follows
d) All I, II & III follows

www.bankingpdf.com
e) None of these

7). Statements – Z@M; M©K; KxF


Conclusions:
I. F©Z
II. K x Z
III. F © M

a) None follows
b) Only I follow
c) Only II follows
d) d) Only III follows
e) Only II & III follows

8). Statements – V % H ; H @ F ; F ∂ E
Conclusions:
I. F@V
II. F x V
III. E % H

a) Only either I or II follows


b) Only III follows
c) Only I & II follows

www.bankingpdf.com
d) All I, II & III follows
e) Only either I or II and III

9). Statements – W © T ; T ∂ N ; N % D
Conclusions:
I. DxT
II. W © N
III. D @ T

a) None follows
b) Only I follow
c) Only II follows
d) d) Only III follows
e) Only I & II follows

10). Statements – Y@G; G©K; KxR


Conclusions :
I. R©Y
II. K x Y
III. R © G

a) None follows

www.bankingpdf.com
b) Only I follow
c) Only II follows
d) d) Only III follows
e) Only II & III
Answers:

1)b 2)e 3)c 4)d 5)b 6)d 7)c 8)e 9)a 10)c

Explanation:
Directions (Q. 1-5):

www.bankingpdf.com
Directions (Q. 6-10):
P % Q means P is not smaller than Q ----------------- [P≥ Q]
P © Q means P is neither smaller than nor equal to Q --- [P > Q]
P x Q means P is neither greater than nor equal to Q --------- [P < Q]

www.bankingpdf.com
P ∂ Q means P is not greater than Q ---------- [P ≤ Q]
P @ Q means P is neither greater than nor smaller than Q------- [P=Q]

6). R ≤ K < M = J
J>K
M> R
R<J
All follows
Answer: d)

7). Z = M > K < F


F > Z not follows
K< Z follows
F> M not follows
Answer: c)

8). V ≥ H = F ≤ E
F=V
F< V
E≥H

www.bankingpdf.com
Either I and II and III follows
Answer: e)

9). W > T , N ≥ T , N ≥ D
D<T
W> N
D=T
None follows
Answer: a)

10). Y = G > K < R


R > Y not follows
K< Y follows
R> G not follows
Answer: c)

www.bankingpdf.com
640 – 650 Questions

Directions (Q. 1 - 5): Study the following information carefully and answer the given questions.
Mr and Mrs Sukumar have one son and two daughters and all are married. Mr Sukumar invited his entire family for
dinner. All eight persons are sitting around a rectangular table, three on each longer side and one on each smaller
side. Mr Sukumar sits on the smaller side of the table. Sanjay and Divya represent one of the couples. Priya sits on
her son’s immediate right. Gokul sits between Mamta and Divya. Nisha is not the wife of Mohan. Each man sits
between two women. Mamta is daughter-in-law of Mr Sukumar, who is not the neighbour of his daughters.

1). Who among the following is husband of Mamta?


a) Gokul
b) Mohan
c) Sanjay
d) Either Mohan or Gokul
e) None of these

2). What is Priya’s position with respect to her husband?


a) Immediate right
b) Second to the left
c) Immediate left
d) Third to the right
e) None of these

www.bankingpdf.com
3). Who sits opposite to Mr Sukumar?
a) Mohan
b) Gokul
c) Nisha
d) Sanjay
e) None of these

4). How is Priya related to Gokul?


a) Wife
b) Sister
c) Mother-in-law
d) Sister-in-law
e) None of these

5). If all the males interchange their positions with the persons on their immediate right and all the females
interchange their positions with the persons on their immediate left, then who among the following is third to the right
of Mohan?
a) Nisha
b) Divya
c) Mamta
d) Gokul

www.bankingpdf.com
e) None of these

Directions (Q. 6-10): Each of the questions below consists of a question and two statements numbered I and II given
below it. You have to decide whether the data provided in the statements are sufficient to answer the question. Read
both the statements and give answer
a) if the data in statement I alone are sufficient to answer the question, while the data in statement II alone are not
sufficient to answer the question.
b) if the data in statement II alone are sufficient to answer the question, while the data in statement I alone are not
sufficient to answer the question.
c) if the data either in statement I alone or in statement II alone are sufficient to answer the question.
d) if the data in both statements I and II together are not sufficient to answer the question.
e) if the data in both statements I and II together are necessary to answer the question.

6). In which month is my birthday?


I. I will celebrate my birthday in a month which begins on a Friday and ends on a Friday too.
II. I will celebrate my birthday in the current month. The result of adding the date of the last Monday of last month and
the date of the first Thursday of next month is 38.

7). Deepa and Subha put 40 matches on the table and make themselves aware of the game for which the matches
are to be used. As per the rules, the winner is the one who takes the last match. Who will win the game?
I. Each player in turn takes 1, 3 or 5 matches.

www.bankingpdf.com
II. Subha chooses to go first and takes 3 matches.

8). Is it 5 o’clock?
I. I’m looking at my watch. From this moment on, the hour hand will take exactly twice as long as the minute hand to
reach the number six.
II. I’m looking at my watch. From this moment on, the hour hand will take exactly three times longer than the minute
hand to reach the number six.

9). What was the score in each match of a three-team tournament in which the three teams (who participated) A, B
and C got 3 points, 2 points and 1 point respectively? 2 pointsare awarded for a win and 1 for a draw.
I. Each team played once against each of the two other teams.
II. Each team scored one goal.

10). Two cars start travelling from different points and in opposite directions in a circuit race at a constant speed.
How much faster is one car going than the other?
I. The distances from A to B, B to C, and C to A are the same.
II. The cars cross for the first time at point A, the second time at point B, the third time at point C, and the fourth time
again at point A.

Answers:
1)b 2)c 3)d 4)c 5)b 6)a 7)e 8)c 9)e 10)b

www.bankingpdf.com
Explanation:

Directions (Q. 1-5):

www.bankingpdf.com
6). From I: If a month starts and ends with the same day of the week, it must have a complete number of weeks plus
one more day. The only possible month is a 29-day February.
From II: In order to add up to 38, it can only be the highest possible number for the last Monday of a month (31) and
the highest for the first Thursday of a month (7). Therefore, both last month and the current must have 31 days. The
only two 31-day months in a row in the same calendar year are July and August. If both the months are from the
same calendar year, August is our answer. But the statement does not say that the both the months are from the
same calendar year. Note that December and January are also the months in a row which have 31 days each.
Answer: a)

7). From I and II together: Every player takes an odd number of matches per play. After the first player goes, there
will always be an odd number of matches left. After the second player goes, there will always be an even number of
matches left. Therefore, the second player, ie Deepa, is the winner.
Answer: e)

8). From I: Our answer is ‘Yes’ because from here, the minute hand will take 30 minutes to reach 6, and the hour
hand will take an entire hour.
From II: Our answer is ‘No’. There are two possible times in this situation: approx 5 : 15 (the minute hand takes 15
minutes to reach 6 and the hour hand takes 45) and approx 3 : 45 (the minute hand takes 45 minutes to reach 6 and
the hour hand takes 2 hours and 15 minutes, which is 135 minutes.)
Answer: c)

www.bankingpdf.com
9). From I and II together: The team A could win 3 points only by winning one match and tying another. Since they
only scored one goal, the results must be 1-0 and 0-0. The team C tied one and lost the other match. The scores
must have been 1-1 and 0-1. Their tied game must have been against the team B. So team A beat team C (1-0) and
team A tied with team B (0-0).
Answer: e)

10). From II: The first crossing took place at point A. Consider A as a new starting point. Do the same for every
crossing point. Since they drove at consistent speeds, the distances from A to B, B to C, and C to A are the same.
After point A, one car must have driven twice the distance as the other to reach B at the same time. Therefore, one
goes twice as fast as the other.

Answer: b)

www.bankingpdf.com
650 – 660 Questions

Directions (Q. 1-5): Each of the questions below consists of a question and two statements numbered I and II given
below it. You have to decide whether the data provided in the statements are sufficient to answer the question.

1). How is A related to B?


I. B has two sisters C and O.
II. C’s father and A are brothers.
a) If the data in Statement I alone are sufficient to answer the question, while the data in Statement II alone are not
sufficient to answer the question.
b) If the data in Statement II alone are sufficient to answer the question, while the data in Statement I alone are not
sufficient to answer the question.
c) If the data either in Statement I alone or in Statement II alone are sufficient to answer the question.
d) If the data in both the Statements I and II are not sufficient to answer the question.
e) If the data in both the Statements I and II together are necessary to answer the question.

2). How is ‘go’ written in a certain code language?


I. ”go home now” is written as “ja ho na” in that code language.
II. “come home later” is written as “pa ta ho” in that code language.
a) If the data in Statement I alone are sufficient to answer the question, while the data in Statement II alone are not
sufficient to answer the question.

www.bankingpdf.com
b) If the data in Statement II alone are sufficient to answer the question, while the data in Statement I alone are not
sufficient to answer the question.
c) If the data either in Statement I alone or in Statement II alone are sufficient to answer the question.
d) If the data in both the Statements I and II are not sufficient to answer the question.
e) If the data in both the Statements I and II together are necessary to answer the question.

3).How many children does Mr. (Y) have?


I. He has only one son and twice as many daughters.
II. One of his daughters is elder than his son.
a) If the data in Statement I alone are sufficient to answer the question, while the data in Statement II alone are not
sufficient to answer the question.
b) If the data in Statement II alone are sufficient to answer the question, while the data in Statement I alone are not
sufficient to answer the question.
c) If the data in Statement I alone or in Statement II alone are sufficient to answer the question.
d) If the data in both the Statements I and II are not sufficient to answer the question.
e) If the data in both the Statements I and II together are necessary to answer the question.

4). Is it 5 0’clock?
I.One hour ago Reshma left for home.
II.One hour later the hour and minute hands of clock will make a straight line.

www.bankingpdf.com
a) If the data in Statement I alone are sufficient to answer the question, while the data in Statement II alone are not
sufficient to answer the question.
b) If the data in Statement II alone are sufficient to answer the question, while the data in Statement I alone are not
sufficient to answer the question.
c) If the data either in Statement I alone or in Statement II alone are sufficient to answer the question.
d) If the data in both the Statements I and II are not sufficient to answer the question.
e) If the data in both the Statements I and II together are necessary to answer the question.

5).Who is tallest among W,X,Y and Z?


I.W is taller than Z, who is taller than X.
II.Y is shorter than X.
a) If the data in Statement I alone are sufficient to answer the question, while the data in Statement II alone are not
sufficient to answer the question.
b) If the data in Statement II alone are sufficient to answer the question, while the data in Statement I alone are not
sufficient to answer the question.
c) If the data either in Statement I alone or in Statement II alone are sufficient to answer the question.
d) If the data in both the Statements I and II are not sufficient to answer the question.
e) If the data in both the Statements I and II together are necessary to answer the question.

www.bankingpdf.com
6). In each question below is given a statement followed by two assumptions numbered I and II An assumption is
something supposed or taken for granted. You have to consider the statement and the following assumptions and
decide which of the assumptions is implicit in the statement.
Statements: Many organizations have switched over to on-line mode of examinations.
Assumptions: I.Candidates from all parts of the country may be well versed using computers.
II.On-line mode of examinations helps in recruiting more capable personnel.
a) If only assumption I is implicit
b) If only assumption II is implicit
c) If either assumption I or II is implicit,
d) If neither assumption I nor II is implicit.
e) If both assumption I and II are implicit.

7).In each question below is given a statement followed by two conclusions numbered I and II. You have to assume
everything in the statement to be true, then consider the two conclusions together and decide which of them logically
follows beyond a reasonable doubt from the information given in statement.
Statement : It is high time we talked in terms of practical solutions to deal with problems, rather than populist slogans
like free goods for poor, which is a direct invitation to corruption in india.
Conclusions :
I. Most of the government officials in india are corrupt.
II. Corruption in india can be curbed now or never.

www.bankingpdf.com
If only conclusion I follows
If only conclusion II follows
If neither I nor II follows
If both I and II follow
If either I or II follows

Directions (Q. 8-10): Rahul started in north direction from his house and travelled 3 m, then he covered 5m in south-
east direction to reach Amul’s house. Shreyan is brother of Amul and lives in the same home with him
8). If Shreyan has to go to Church, which is 4m away from Amul’s house in the north direction. Then what is the
shortest distance between Church and Rahul’s home?
a) 2m
b) 4m
c) 5m
d) 7m
e) 10m

9). Shreyan can reach a general store by walking 5m in the north east direction. Then how much Rahul has to travel
to reach to general store; if it is known that he can travel only in horizontal and vertical direction
a) 9m
b) 11m

www.bankingpdf.com
c) 15m
d) 17m
e) 21m

10). If after reaching Shreyan’s house, Rahul moves 5 m in the south direction, then in which direction he is now, with
respect to the location of his house.
a) South
b) North
c) North-west
d) South-east
e) South-west

Answers:
1)e 2)d 3)a 4)d 5)e 6)a 7)b 8)c 9)b 10)d

Explanation:
1).From l,C and O are B’s sisters.
From II,C’s father and M brothers
Thus from I and II, A is uncle of B.
Answer: e)

www.bankingpdf.com
2).From I and II,home –ho
Go can be ‘ja’ or ‘na’
Answer: d)

3). From I,Mr Y has 3 children


From II, the number of children cannot be found out.
Hence only statement I is sufficient to answer the question.
Answer: a)

4).From I,the time cannot be determined.


Statement II tells that one hour later, the hour and minute hands of clock will make a straight line, which occurs 11
times in 12 hours. So, still the time cannot be determined.
Answer: d)

5).From statement l,W>Z>X


From statement II,Y<X
Hence, W>Z>X>Y
Answer: e)

www.bankingpdf.com
6).if organizations have switched to on-line mode of examination,then it is assumed that candidates may be well
versed with computers as companies would not like to lose good candidates if they believed someone might not be
well verses with computers. Hence assumption I is implicit
Answer: a)

7).conclusion 1 can’t be concluded as there is no mention of government officials in the main statement.Whereas,
conclusion 2 follows the main statement as it addresses the curbing off corruption which is the main theme of the
passage.
Answer: b)

Directions (Q. 8-10):

www.bankingpdf.com
8).
Answer: c)
9).
Answer: b)
10).
Answer: d)

660 – 670 Questions

1). Statements:
Some belts are papers
Some papers are desks
Some desks are chairs

Conclusions:
(i) Some belts are desks
(ii) Some papers are chairs
(iii) Some belts are chairs

a) All follow
b) None follows
c) Only i and ii follows

www.bankingpdf.com
d) Only ii and iii follows
e) None of these

2). Statements:
Some pots are buckets
All buckets are tubs
All tubs are drums

Conclusions:
(i) Some drums are pots
(ii) All tubs are buckets
(iii) Some drums are buckets

a) All follow
b) None follows
c) Only i and ii follows
d) Only ii and iii follows
e) None of these

3). Statements:
All pins are bags
All chalks are bags
All needles are bags

www.bankingpdf.com
Conclusions:
(i) Some needles are pins
(ii) Some chalks are needles
(iii) No needle is pin

a) Only i and iii follows


b) Either i or iii follows
c) None follows
d) Only ii follows
e) None of these

4). Statements:
Some buses are trucks
Some trucks are boats
No boat is pilot

Conclusions:
(i) Some pilot are buses
(ii) Some boats are buses
(iii) Some pilot are trucks

a) None follows

www.bankingpdf.com
b) All follow
c) Only ii and iii follow
d) Only iii follow
e) None of these

5). Statements:
All flowers are trees
All trees are jungles
No jungle is hill

Conclusions:
(i) No flower is hill
(ii) No tree is hill
(iii) Some jungles are flowers

a) None follows
b) All follow
c) Only ii and iii follow
d) Only iii follow
e) None of these

Directions (Q. 6 - 10): Study the following information carefully and answer the given questions.

www.bankingpdf.com
There are eight balls of different colours, viz White, Orange, Yellow, Green, Black, Red, Brown and Grey. They are
kept on the corners of two square tables placed side by side. If we consider the two squares together as a rectangle,
then each ball on one long side of the rectangle is facing a ball each on the other long side.
Red and Grey balls are kept on the two corners that meet. Brown ball is kept in front of Red Ball. Yellow ball is
opposite to Orange ball and on the immediate left of Brown Ball. Black ball is kept diagonally opposite Grey ball on
one square table. White ball is not adjacent to Brown ball.

6). If the positions of White ball and Green ball are interchanged then which ball will place second to the left of Green
ball?
a) Brown
b) White
c) Grey
d) Yellow
e) None of these

7). Which ball places diagonally opposite to Yellow ball on the same table?
a) Green
b) Red
c) Black
d) White
e) None of these

www.bankingpdf.com
8). Which ball places adjacent to Black ball?
a) Yellow, Green
b) Orange, White
c) White, Green
d) Green, Red
e) None of these

9). Four of the following five are alike in a certain way and hence form a group. Which is the one that does not belong
to that group?
a) Red, Yellow
b) Orange, Brown
c) Red, Black
d) White, Green
e) Grey, Black

10). Which of the following balls is in front of the White ball?


a) Black
b) Grey
c) Green
d) Red
e) None of these
Answers:

www.bankingpdf.com
1)b 2)c 3)b 4)a 5)d 6)b 7)b 8)c 9)c 10)a

Explanation:

1). b)

2). c)

3). b)

www.bankingpdf.com
4). a)

5). d)

Direction (06 to 10):

www.bankingpdf.com
6). b)
7). b)
8). c)
9). c)
10). a)

www.bankingpdf.com
670 – 680 Questions

1). Rubesh started in north direction from his house and travelled 3m, then he covered 5m in south-east direction to
reach Akil’s house. Nikil is brother of Akil and lives in the same home with him. In which direction is Rubesh’s house
from his final position?
a) North-west
b) North
c) South
d) South-east
e) North-east

2). In making decision about important questions, it is desirable to be able to distinguish between “strong” arguments
and “weak” arguments. “Strong” arguments must be both important and directly related to the questions. “Weak”
arguments may not be directly related to the question and may be of minor important or may be related to the trivial
aspects of the questions. Each questions below is followed by two arguments numbered I and II. You have to decide
which of the arguments is “strong” arguments and which is “weak” argument.
Should all the Universities switch over to online admissions at all levels all over the country with immediately effect?
I. No, all the students may not have easy access to internet.
II. Yes, this will save the students and parents from all the hassles of visiting various colleges and standing in queue.
a) If only argument I is strong

www.bankingpdf.com
b) If only argument II is strong
c) If either argument I or II is strong
d) If neither argument I nor II is strong
e) If both arguments I and II are strong

3). In the question below, a statement followed by two assumptions numbered I and II. An assumption is something
supposed or taken for granted. You have to consider the statement and the following assumptions and decide which
of the assumptions is implicit in the statement.
Statement: The government announced a heavy compensation package for all the victims of the terrorist attacks.
Assumptions:
I. Such incidents of terror may not occur in near futures.
II. Compensation may mitigate the anger among the citizens against the current government
a) If only assumption I is implicit
b) If only assumption II is implicit
c) If either assumption I or II is implicit,
d) If neither assumption I nor II is implicit
e) If both assumptions I and II are implicit

Direction (04 to 08): In the questions given below, certain symbols are used with the following meanings:
S $ T means S is neither equal to nor smaller than T

www.bankingpdf.com
S © T means S is not smaller than T
S * T means S is neither greater nor smaller than T
S # T means S is neither greater than nor equal to T
S @ T means S is not greater than T
Now, in each of the following questions, assuming the given statements to be true, find which of the two conclusions I
and II given below them is/are definitely true.
a) If only conclusion I is true
b) If only conclusion II is true
c) If either I or II is true
d) If neither I nor II is true
e) If both I and II are true
4). Statements: A@B, B © C, C#D
Conclusion: I. C#A, II. A#D

5). Statements: I#J,J@K, K*L


Conclusion: I. L$J, II. J*L

6). Statements: S$T, T#U, U©V


Conclusion: I. S$V, II. T$V

www.bankingpdf.com
7). Statements: R©P, P$M, M@D
Conclusion: I. D$R, II. M#R

8). Statements: F#K, K©D, N@D


Conclusion: I. N*K, II. N#K

Direction (09 to 10): Study the following information to answer the given questions.
In a certain code, “her idea has merit” is written as “fo la bu na”, “merit list has been displayed” is written as “jo ke la
si na”, “her name displayed there” is written as “ya si bu zo” and “name in merit list” is written as “na ya go ke”.

9). What does “ke” stand for?


a) Been
b) Has
c) Merit
d) Name
e) List

10). What is the code for ‘idea’?

www.bankingpdf.com
a) Fo
b) La
c) Bu
d) Na
e) Either bu or na

Answers:
1)a 2)e 3)d 4)d 5)c 6)d 7)b 8)c 9)e 10)a

Solutions:
1).

Answer: a)

www.bankingpdf.com
2). Majority of the population lives in small towns and villages where internet penetration is very low. Hence all the
students may not have easy access to internet
Introducing online admissions will save students and parents from all the hassles of visiting colleges and standing in
queue as they can select their seats without travelling to other places.
Hence both arguments are strong
Answer: e)

3). Announcing a heavy compensation package does not ensure that terrorist attacks will be stopped. Hence
assumption I is also not valid
And compensation package will not mitigate or lessen the anger among the citizens. Hence assumption II is also not
valid
Answer: d)

4). A ≤ B…(i); B ≥ C…(ii); C < D…(iii)


With these equations no relation can be established between C and A, and A and D.
Answer: d)

5). I< J… (i); J ≤ K … (ii); K = L… (iii)

www.bankingpdf.com
Combining all the equations, we get K = L ≥ J > I hence, L≥J, ie L> J or L = J hence, either conclusion I or II is true.
Answer: c)

6). S >T … (i); T< U …(ii); U ≥ V … (iii)


No relation can be established between S and V or between T and V. hence, conclusion I (S > V) are not true
Answer: d)

7). R ≥ P …(i); P > M… (ii); M ≤ D … (iii) Combining (i) & (ii), we get R ≥ P > M …(iv). From conclusion (iv), we get
R > M. hence, conclusion II (M < R) is true. But we can’t get any specific relation between D and R. therefore
conclusion I is not true
Answer: b)

8). F < K …..(i); K ≥ D …. (ii); N ≤ D ….(iii) combining the equations (ii) and (iii), we get
K ≥ D ≥ N Hence K>N, i.e. K > N. Hence, either conclusion I or II is true.
Answer: c)

Directions (Q. 9-10):


Using the given statements, the codes are
Herbu

www.bankingpdf.com
Beenjo
Ideafo
Displayedsi
Hasla
Nameya
Meritna
Therezo
Listke
Ingo
9).
Answer: e)
10).
Answer: a)

www.bankingpdf.com
680 – 690 Questions

Direction (Q. 1-5): Study the given information and answer the following questions:
When a word and number arrangement machine is given an input line of words and numbers, it arranges them
following a particular rule. The following is an illustration of an input and rearrangement:

Input: 45 near change 33 48 drop track fold 62 88 98 grey order 21


Step I: 45 near change 33 48 drop fold 62 88 98 grey order track 21
Step II: 45 near change 48 drop fold 62 88 98 grey track 21 order 33
Step III: change 48 drop fold 62 88 98 grey track 21 order 33 near 45
Step IV: change drop fold 62 88 98 track 21 order 33 near 45 grey 48
Step V: change drop 88 98 track 21 order 33 near 45 grey 48 fold 62
Step VI: change 98 track 21 order 33 near 45 grey 48 fold 62 drop 88
Step VII: track 21 order 33 near 45 grey 48 fold 62 drop 88 change 98
Step VII is the last step of the above arrangement as the intended arrangement is obtained. As per the rules followed
in the given steps, find out the appropriate steps for the given input.
Input: 35 quite hard catch 50 65 98 solve great 71 82 11 orange menu

1). Which of the following is Step III of the given input?


a) quite 82 hard 50 great 35 catch 11 65 98 solve 71 orange menu
b) hard catch 65 98 great 71 82 menu solve 11 quite 35 orange 50
c) hard catch 98 great 71 82 solve 11 quite 35 orange 50 menu 65

www.bankingpdf.com
d) hard catch 98 65 great 71 82 menu solve 11 quite 35 orange 50
e) None of these

2). How many steps will be required to complete the given input?
a) Five
b) Six
c) Seven
d) Eight
e) None of these

3). What is the position of ‘menu’ from the left end in the final step?
a) Sixth
b) Seventh
c) Eighth
d) Fifth
e) Ninth

4). Which element is fourth to the right of ‘great’ in Step V?


a) catch

www.bankingpdf.com
b) 11
c) quite
d) 98
e) 35

5). Which of the following is the fourth element from the left end of Step VI?
a) catch
b) menu
c) 71
d) orange
e) None of these

Directions (Q. 6-10): Read the following passage carefully and answer the questions given below it.

A group of seven friends, P, Q, R, S, T, U and V, work as Clerk, Probationary Officer, Manager, Senior Manager,
Chief Manager, Assistant General Manager and General Manager, for Banks Indian Bank, Bank of India, Allahabad
Bank, Syndicate Bank, Dena Bank, Punjab National Bank and Vijaya Bank, but not necessarily in the same order. R
works for Allahabad Bank and is neither a General Manager nor a Chief Manager. T is a Manager and works for
Punjab National Bank. P works as Assistant General Manager and does not work for Indian Bank or Dena Bank. The
one who is a Probationary Officer works for Bank of India, The one who works for Indian Bank works as a Senior

www.bankingpdf.com
Manager, U works for Dena Bank. V works for Syndicate Bank as a General Manager. S is not a Probationary
Officer.

6). Who amongst the following works as a Probationary Officer?


a) R
b) Q
c) U
d) S
e) None of these

7). What is the profession of R?


a) Senior Manager
b) Probationary Officer
c) Clerk
d) Cannot be determined
e) None of these

8). For which bank does Q work?


a) Bank of India
b) Vijaya Bank

www.bankingpdf.com
c) Indian Bank
d) Either Bank of India or Vijaya Bank
e) None of these

9). What is the profession of the person who works for Vijaya Bank?
a) Chief Manager
b) Probationary Officer
c) Senior Manager
d) Assistant General Manager
e) None of these

10). Which of the following combinations of person, profession and bank is correct?
a) P- Assistant General Manager – Bank of India
b) S - Chief Manager - Indian Bank
c) U -Probationary Officer - Dena Bank
d) Q -Probationary Officer – Vijaya Bank
e) None of these

Answers:

www.bankingpdf.com
1)b 2)c 3)b 4)c 5)e [11] 6)b 7)c 8)a 9)d 10)e

Explanation:

Directions (1-5):
Words are arranged in reverse alphabetical order and the numbers are arranged in ascending order. One word and
one number is arranged in each step. The arrangement is done from right to left.
Input: 35 quite hard catch 50 65 98 solve great 71 82 11 orange menu
Step I. 35 quite hard catch 50 65 98 great 71 82 orange menu solve 11
Step II. hard catch 50 65 98 great 71 82 orange menu solve 11 quite 35
Step III. hard catch 65 98 great 71 82 menu solve 11 quite 35 orange 50
Step IV. hard catch 98 great 71 82 solve 11 quite 35 orange 50 menu 65
Step V. catch 98 great 82 solve 11 quite 35 orange 50 menu 65 hard 71
Step VI. catch 98 solve 11 quite 35 orange 50 menu 65 hard 71 great 82
Step VII. solve 11 quite 35 orange 50 menu 65 hard 71 great 82 catch 98

Directions (Q. 6-10):


Friend Bank Occupation

P Vijaya Bank Assistant General Manager

www.bankingpdf.com
Q Bank of India Probationary Officer

R Allahabad Bank Clerk

S Indian Bank Senior Manager

T Punjab National Bank Manager

U Dena Bank Chief Manager

V Syndicate Bank General Manager

www.bankingpdf.com
690 – 700 Questions

Directions (Q. 1-3): Answer the questions on the basis of information given below.
There are nine people Alex (B), Bill (R), Clive (G), Dan (R), Ed (B), Fred (G), Greg (B), Hugh (R), Ian (G) who are on the
bank of Thames and want to cross it with the help of a row boat. The boat is such that only one person can row and it can
accommodate additional two people. The B, G, R in the bracket means that color of the shirt worn by the person is blue,
green or red respectively. Some additional information is given as follows:
 Only the persons wearing Green shirts can row.
 No two persons wearing red shirt or 3 persons wearing green shirt or 2 persons wearing green and one wearing red can
go together.
 Alex cannot be in the boat with Fred.
 Dan cannot be in the boat with Greg.
 Clive rows the boat only the third ferry and is not in the boat otherwise. (Crossing the river from origin to destination is
called as ferry. The return trip is not addressed as ferry.)
 From the information given above, try to find the answers of the following questions:
1). If Alex goes in first ferry and Hugh in the third ferry then minimum no.of times the river needs to be crossed?
a) 8
b) 7
c) 9
d) 11
e) None of these

www.bankingpdf.com
2). If Greg and Bill decide to go together then which of the following statements will be necessarily true?
a) Clive and Hugh were in the third ferry
b) River was crossed at least 9 times before everybody reached the other bank.
c) In any case Fred rowed the boat more number of times than Ian
d) Ed and Dan were never together in the boat if they completed the task in 5 ferries.
e) None of these

3). If Ian does not go with Alex and Ed goes with Bill then, who among the following will be definitely there in the third ferry
if the task is completed in minimum ferries?
a) Alex and Dan
b) Alex and Hugh
c) Clive and Greg
d) Ed and Clive
e) None of these

Directions (Q. 4-6): In each of the questions below are given three statements followed by two conclusions numbered I,
and II. You have to take the given statements to be true even if they seem to be variance from commonly known facts.
Read all the conclusions and then decide which of the given conclusions logically follows from the given statements
disregarding commonly known facts.

www.bankingpdf.com
a) If only conclusion I follow
b) If only conclusion II follow
c) If either conclusion I or II follows
d) If neither conclusion I nor II follows
e) If both conclusion I and II follows
4). Statements : some bags are purses
All purses are folders
No folder is a box.
Conclusion:
I. No box is a bag.
II. some folders are bags.

5). Statements : some bottles are cans


Some cans are tanks
All tanks are reservoirs.
Conclusions:
I. Some tanks are cans.
II. Some reservoirs are cans.

6). Statements : some jams are pickles.


Some jams are sweets.

www.bankingpdf.com
No sweet is flower.
Conclusions:
I. Some sweets are jams.
II. No pickles are jam.

7). Statements : some doors are walls.


Some walls are bricks.
All bricks are stones.
Conclusions:
I. Some stones are walls.
II. Some stones are doors.

Directions (Q. 8-10): Study the following information to answer the given questions.
In a certain code ‘her idea has merit’ is written as ‘fo la bu na’, ‘merit list has been displayed’ is written sa ‘jo ke la si na’, ‘
her name displayed there’ is written as ’ya sib u zo’ and ‘name in merit list’ is written as ‘na ya go ke’.

8). Which of the following represents ‘name has been displayed’?


a) ya la ke si
b) jo si ya la
c) si jo ken a

www.bankingpdf.com
d) buy a ke la
e) ya si jo zo

9). What does ‘zo’ stand for?


a) There
b) Displayed
c) Name
d) Her
e) Cannot be determined

10). Which of the following may represent ‘her name is there’?


a) zo ya go wo
b) bu ya zo si
c) zo ya bu ke
d) ya zo wo bu
e) wo go zo ya

Answers:
1)c 2)b 3)a 4)b 5)e 6)a 7)a 8)b 9)a 10)d

www.bankingpdf.com
Explanation:
Directions (Q. 1-3): On the basis of the given conditions we can infer that to reduce the number of ferries, the best
possible combination in the boat is 1R + 1B + 1G.now Alex (B) is going in first ferry and Clive(G) is going in third ferry. As
Alex cannot go with Fred(0),so he must go with lan(G) along with one person wearing Red shirt who can be Bill or Dan
because Hugh is in the third ferry. Also, Hugh is going in the third ferry with Clive who cannot return so there must be one
person on the opposite bank wearing Green shirt for which Fred and lan have to go together in the second ferry along with
a person with Blue shirt. That person can be Greg or Ed.

1). So the possible arrangement is,


Alex(B) + Bill(R) + lan(G)…………(1st Ferry, lan comes back)
Greg(B) +Fred(G)+ lan(G)…….. .. . . ..(2nd Ferry,lan or fred comes back)
Ed (B) + Hugh (R) +Clive (G)………(3rd ferry,Fred or lan comes back)
Dan (R) + lan / Fred(G)…………………(4th ferry,lan/fred comes back)
Ian (G) + Fred (G)………………………………(5th ferry)
Hence the river is crossed 9 times.
Answer: c)

2). First statement may not be true if the task is not completed in minimum number of ferries. Second statement is
possible because in five ferries everybody can cross the river.
The third statement is not necessarily true because either of Fred and lan can row the boat more number of times than the
other. For the fourth statement, as explained above the best possible combination to complete the task in four ferries is

www.bankingpdf.com
1R + 1B + 1G,now we consider this statement. Ed and Dan can be together if the task is completed in 5 ferries. This is
also not necessarily true.
Answer: b)

3). If they follow the conditions given in the question, then Alex will have to go in the third ferry with Clive and Dan.
Answer: a)

4).Taking the possible cases of the statements, we see that only conclusion II follows.

Answer: b)

5).Clearly, both the conclusions follow.

www.bankingpdf.com
Answer: e)

6).only conclusion I follows

Answer: a)

7).Some stones are walls.

www.bankingpdf.com
Answer: a)

Directions (Q. 8-10): Using the given statements, the codes are
Her → bu
been → jo
idea → fo
displayed → si
has → la
name → ya
merit → na
there → zo
list → ke
in → go

www.bankingpdf.com
8).
Answer: b)

9).
Answer: a)

10).
Code for ‘is’ not known. So out of the given options only ‘ya zo wo bu’ may be the code for ‘her name is there’
Answer: d)

www.bankingpdf.com
700-710 Questions :

Directions (Q. 1 - 5): Study the following information carefully and answer the given questions.
Eight persons Nimai, Nimish, Ninad, Nipun, Nirad, Niraj, Niral and Nirav are sitting around a circular table) Each of them
has a mobile of a different company, viz Nokia, Samsung, HTC, Moto, Micromax, Xiaomi, LeEco and Lenovo but not
necessarily in the same order. Four of them are not facing the centre)
The one who has Micromax is on the immediate left of Nipun, who does not have LeEco. Nipun is third to the right of
Nirav. Niral is fourth to the left of Niraj. Neither Niral nor Niraj is an immediate neighbour of Nipun. Ninad has HTC and sits
exactly between Nimai and Niraj. Ninad is facing the centre and is to the right of both Nimai and Niraj. Nirad is third to the
left of Nimai, who has Lenovo.
Nimai sits third to the left of the one who has Micromax, The one who has Samsung sits second to the left of the one who
has Lenovo. The one who has Nokia sits second to the right of Nipun. The one who has Xiaomi sits second to the right of
the person who has HTC) Nimish does not like Micromax. Nimai faces the centre)

1). Who among the following has the mobile of Moto?


a) Nipun
b) Niraj
c) Nirav
d) Ninad
e) None of these

2). Who among the following sits third to the right of the one who has the mobile of LeEco?

www.bankingpdf.com
a) Nirad
b) Nipun
c) Niraj
d) Nimai
e) None of these

3). Who among the following sits third to the left of Nirav?
a) Ninad
b) Nimish
c) Niraj
d) Nipun
e) None of these

4). Four of the following five are alike in a certain way and so form a group. Which is the one that does not belong to that
group?
a) Niraj, Nimish
b) Ninad, Nirad
c) Nimai, Nipun
d) Nipun, Nirad
e) Ninad, Nirav

5). Who among the following sits opposite to Nirad?


a) The one who has Lenovo

www.bankingpdf.com
b) The one who has Xiaomi
c) The one who has HTC
d) The one who has Moto
e) None of these

6). Statement: B > D < T ≥ V = M ≥ X > Z


Conclusions:
I. T > X
II. T = X
III. T ≥ Z
IV.M > T

a) None follows
b) Only III follows
c) Only IV follows
d) Only either I or II follows
e) None of these

7). Statement: A ≥ R > S = Y < W < V


Conclusions:
I. A > Y
II. W > R

www.bankingpdf.com
III. V > S
IV. Y ≤ A

a) Only I and III follows


b) Only I and IV follows
c) Only II and III follows
d) Only I, II and IV follows
e) None of these

8). Statements: O ≤ U < L, P ≥ I < C = L


Conclusions:
I. U > I
II. C > O
III. L > I
IV. P > O

a) Only I and II follows


b) Only I, II and III follows
c) Only II and III follows
d) Only II, III and IV follows
e) None of these

www.bankingpdf.com
9). Statements: A > F ≥ G, D < H = G
Conclusions:
I. A ≥ H
II. F ≥ D
III. D ≥ G
IV. A = D

a) None follows
b) Only I follows
c) Only II follows
d) Only III follows
e) Only IV follows

10). Statement: R ≥ U > F = E ≥ X > Z


Conclusions:
I. R > E
II. U > X
III. F > Z
IV. R > Z

a) Only I, II and III follows


b) Only II, III and IV follows
c) Only I, III and IV follows

www.bankingpdf.com
d) All I, II, III and IV follows
e) None of these

Answers:
1)a 2)b 3)c 4)d 5)c 6)d 7)a 8)c 9)a 10)d

Explanation:

Directions (Q. 1-5):

www.bankingpdf.com
6). B > D < T ≥ V = M ≥ X > Z
T>X
T=X
T≥Z
M>T
Either I or II follows.
Answer: d)

www.bankingpdf.com
7). A ≥ R > S = Y < W < V
A > Y follows
W > R not follows
V > S follows
Y ≤ A not follows
Answer: a)

8). O ≤ U < L, P ≥ I < C = L


U > I not follows
C > O follows
L > I follows
P > O not follows
Answer: c)

9). A > F ≥ G, D < H = G


A≥H
F≥D
D≥G
A=D

www.bankingpdf.com
None follows
Answer: a)

10). R ≥ U > F = E ≥ X > Z


R>E
U>X
F>Z
R>Z
All follows

Answer: d)

710-720 Questions :

Directions (Q. 1-5): In making decisions about important questions, it is desirable to be able to distinguish between “
strong” arguments and “weak” arguments insofar as they relate to the question. “Strong” arguments are those which
are both important and directly related to the question. “Weak” arguments are those which are of minor importance
and also may not be directly related to the question or may be related to a trivial aspect of the question.

www.bankingpdf.com
Each question below is followed by two arguments numbered I and II. You have to decide which of the arguments is
a “strong” argument and which is a “weak” argument.
Give answer
a) if only argument I is “strong”.
b) if only argument II is “strong”.
c) if either I or II is “strong”.
d) if neither I nor II is “strong”.
e) if both I and II are “strong”.

1). Statement: Should Non-Resident Indians (NRIs) be entitled to dual citizenship?


Arguments:
I. Yes. This will bolster the intimacy of the NRIs with the Indian soil.
II. No. The move will not be fruitful for all NRIs because there is lack of provision for dual citizenship in many
countries.

2). Statement: Should power theft be made a cognisable offence?


Arguments:
I. Yes. People do not use power honestly.
II. Yes. This will be a wise decision and will reduce antisocial activities in society.

3). Statement: Should reputed private driver training schools be allowed to issue driving licences?
Arguments:

www.bankingpdf.com
I. Yes. This will enable more and more people to get driving licences.
II. No. This will increase the congestion of the vehicles on the road and will result in traffic jams too.

4). Statement: Should Antipiracy Bill be brought in the Parliament?


Arguments:
I. Yes. This will help the film industry to fight against piracy as a result of which the industry will be financially strong.
II. No. We have already provisions such as Copyright laws etc to fight against piracy.

5). Statement: Should a trouble-torn state like Jammu & Kashmir release separatist leaders without any condition?
Arguments:
I. Yes. Release of separatist leaders will be useful as a goodwill gesture to make the situation peaceful in the state.
II. No. Unconditional release of the separatist leaders will aggravate the existing situation because these leaders
have not shown any flexibility in their views.

Directions (Q. 6-10): Study the following information carefully and answer the given questions.
There are seven friends, namely Poorna, Pankaj, Pramila, Priya, Prabhas, Prithvi and Punita. They work in the same
organisation but in different departments, ie Sales, Production and Administration. There are at least two persons in
the same department. Moreover, each of these seven friends has a different salary as well. No two of them have the
same salary. In addition to this, some information is given as follows.
Prithvi works in Production department and earns more than only Punita.

www.bankingpdf.com
Poorna does not work with Pankaj and Punita. Pankaj stands 4th from the top when arranged in descending order of
their salaries.
Prabhas is neither from Sales not from the department where Priya works. Punita and Pramila are in the same
department.
Priya works only with Pankaj but not in Sales department and she earns less than only Pramila and Prabhas.
At most three people can work in the same department.

6). Who among the following work in the Production department?


a) Prithvi, Prabhas and Poorna
b) Prithvi, Priya and Pankaj
c) Pramila, Punita and Prithvi
d) Prithvi, Poorna and Pramila
e) None of these

7). Which of the following is definitely true about Pramila?


a) She works in the Production department.
b) She earns the most among all the friends.
c) She works with Punita in the Administration department.
d) All are true
e) None of these

8). Among the following persons, Poorna works with whom?

www.bankingpdf.com
a) Priya
b) Pramila
c) Prabhas
d) Cannot be determined
e) None of these

9). Which of the following statements is/are false?


a) Punita earns more than only Poorna.
b) Prabhas works in the Administration department.
c) The Sales department has the maximum number of persons working.
d) All are false
e) None of these

10). Who amongst them earns the most?


a) Pramila
b) Prabhas
c) Punita
d) Cannot be determined
e) None of these

Answers:

www.bankingpdf.com
1)a 2)d 3)d 4)a 5)c 6)a 7)e 8)c 9)d 10)d

Explanation:
1). I is strong because intimacy of NRIs with Indian soil will be helpful for Indian economy. II is a weak argument
because a move can’t be restricted only because the move is not giving benefit to all of a kind.
Answer: a)

2). Answer: d)

3). I is weak because it is not desirable. Moreover, is falsely assumes that govt authorities can’t cope with the
demand of driving licences. II is weak because it is not logical. It wrongly assumes that such a move will lead to a
spurt in the number of vehicles.
Answer: d)

4). Only I is strong. II is not strong because we do not know whether the existing provisions are sufficient or not.
Answer: a)

www.bankingpdf.com
5). I is strong because it will encourage non-violence and will make the environment conducive for peace. II is also
strong because effort from only one side will not necessarily usher in desirable result. But the state can’t be both
more and less peaceful at the same time. Hence either I or II follows.
Answer: c)

Direction (Q. 6-10):


Department Persons

Production Prithvi, Poorna and Prabhas

Sales Punita, Pramila

Administration Priya, Pankaj

Salary in descending order


1 = Pramila/Prabhas
2 = Pramila/Prabhas
3 = Priya
4 = Pankaj
5 = Poorna
6 = Prithvi

www.bankingpdf.com
7 = Punita

6).
Answer: a)
7).
Answer: e)
8).
Answer: c)
9).
Answer: d)
10).
Answer: d)

www.bankingpdf.com
720-730 Questions :

1). Which of the following expressions will be definitely true if the given expressions ‘A ≥D = E < B’, ‘F ≤ C < E’ is
definitely true?
a) A ≥ B
b) F < D
c) B ≥ E
d) C > E
e) None of these

2). Which of the following should be placed in blank spaces respectively (in the same order from left to right) in order
to complete the given expression in such a manner that makes the expression R > S definitely true ? _____________
≥________ > ______≤ ____
a) P, R, Q, S
b) S, Q, P, R
c) R, Q, S, P
d) P, S, Q, R
e) None of these

www.bankingpdf.com
3). Which of the following symbols should replace the question mark in the given expression in order to make the
expressions ‘N > P’ and ‘L ≤ N’ definitely true? P<L = O? N ≥M
a) >
b) ≥
c) ≤
d) Either < or ≤
e) =

4). In which of the following expressions will the expression ‘A ≤ D’ can be true?
a) B=A ≥ C ≤ D
b) D > B = C ≤ A
c) A > C ≥ D < B
d) C > D = B ≥ A
e) None of these

5). If the expressions ‘L ≤P >R > O > S’ is true, which of the following conclusions will be definitely false?
a) O<P
b) P>S
c) R<S
d) P≥L

www.bankingpdf.com
e) None of these

Directions [6-9]: Read the following information carefully to answer these questions.
A. In a family of six members P, Q, R, S, T and U each one has a different hobby.
B. The varieties in hobbies are Cooking, Driving, Singing, Dancing, Mimicry and Politics.
C. There are three married couples.
D. No female has Dancing or Mimicry as her hobby.
E. S is the brother of P.
F. Q is the daughter-in-law of T, who has cooking as her hobby
G. The singer is married to the dancer.
H. U does not sing.
I. P, who is the son of R, has Driving as his hobby.

6).Who among the following has Mimicry as hobby?


a) Q
b) R
c) S
d) U
e) Data inadequate

www.bankingpdf.com
7). How is Q related to P?
a) Daughter- in-law
b) Sister-in-law
c) Sister
d) Mother
e) Data inadequate

8). What is the hobby of U?


a) Singing
b) Dancing
c) Politics
d) Mimicry
e) Data inadequate

9). Which of the following hobby pair for a husband-wife combination is definitely true?
a) Driving- Singing
b) Singing – Mimicry
c) Mimicry –Politics

www.bankingpdf.com
d) Politics – Driving
e) Mimicry – Driving

10). Pointing to a photograph, Shyam said, “His mother’s only child is my father’s only son”. Whose photograph is
Shyam pointing to?
a) Brother’s
b) His own
c) Cousin’s
d) Father’s
e) Nephew

Answer:
1)b 2)c 3)c 4)d 5)c 6)b 7)b 8)c 9)d 10)b

Solutions:
1). From both the expressions: Since D = E, F≤ C <D
Hence, F< D
Answer: b)

www.bankingpdf.com
2). Consider option1:
Relationship between R and S cannot be determined
Consider option2:
Relationship between R and S cannot be determined
Consider option 3:
Since R ≥ Q and Q > S, expression R > S is definitely true
Answer: c)

3). Consider expression, P<L = O ≤ N ≥ M


Then it can concluded that both the expressions ‘ N > P’ and “ L ≤ N’ are definitely true.
Answer: c)

4). Consider option1: Relationship between A and D cannot be determined


Consider option2: Relationship between A and D cannot be determined
Consider option3: ‘A ≤ D’ is definitely false
Consider option4: ‘A ≤ D’ is definitely true
Answer: d)

5). Option 1 is true

www.bankingpdf.com
Option 2 is true
Option 3 is false
Option 4 is true
Answer: c)

Directions (Q. 6-9):

10). Shyam is pointing to himself


Shyam’s gender is not given. So, if we assume Shyam as male, then his father’s only son will be himself
If we assume Shyam as female, then his father’s only son will be her brother. But according to the first statement, his
mother have only child. Hence ,this assumption is invalid.
Answer: b)

www.bankingpdf.com
730-740 Questions :

Directions (Q. 1-5): Study the following information to answer the given questions.
Twelve people are sitting in two parallel rows containing six people each such that they are equidistant from each
other.
In row 1, Abhinav, Abhiraam, Abhiraj, Abhirath, Abhirup and Abhishek are sitting facing South. In row 2, Abhati,
Abhidha, Abhidhya, Abhijna, Abhilasa and Abhinithi are sitting facing North.
Therefore, in the given seating arrangement, each member sitting in a row faces another member of the other row.
Three persons sit between Abhiraj and Abhirath. Either Abhiraj or Abhirath sits at an extreme end of the line. The one
who faces Abhirath sits third to the left of Abhidhya. Abhijna faces the one who sits third to the left of Abhinav and he
cannot sit adjacent to Abhidhya. The immediate neighbour of Abhidha faces the immediate neighbour of Abhinav.
Only one person sits between Abhati and Abhilasa, who is facing the one sitting on the immediate right of Abhirup.
Neither Abhirup nor Abhishek faces Abhidhya. Abhinithi and Abhidha cannot sit adjacent to each other.

1). Who among the following faces Abhiraam?


a) Abhati
b) Abhidha
c) Abhilasa
d) Abhidhya
e) Abhijna

www.bankingpdf.com
2). Who among the following sit at the extreme ends of the rows?
a) Abhirath, Abhidha
b) Abhilasa, Abhishek
c) Abhirup, Abhijna
d) Abhiraam, Abhilasa
e) Abhishek, Abhidha

3). If Abhirup is related to Abhati in the same way as Abhiraj is related to Abhijna, which of the following is Abhinav
related to, following the same pattern?
a) Abhinithi
b) Abhidhya
c) Abhidha
d) Abhilasa
e) Cannot be determined

4). How many persons are sitting between Abhirup and Abhishek?
a) None
b) One

www.bankingpdf.com
c) Two
d) Three
e) Four

5). Which of the following is true regarding Abhati?


a) Abhidha is an immediate neighbour of Abhati.
b) Abhati faces the immediate neighbour of Abhiraam.
c) Abhati sits second from the left end of the row.
d) Abhati sits at one of the extreme ends.
e) Abhati faces the one who is second from the right end of the row.

Directions (Q. 6-10): Below is given a passage followed by several possible inferences which can be drawn from the
facts stated in the passage. You have to examine each inference separately in the context of the passage and decide
upon its degree of truth or falsity. Mark answer
a) if the inference is “definitely true”, ie it properly follows from the statement of facts given.
b) if the inference is “probably true” though not “definitely true” in the light of the facts given.
c) if the “data are inadequate”, ie from the facts given you cannot say whether the inference is likely to be true or
false.
d) if the inference is “probably false” though not “definitely false” in the light of the facts given.

www.bankingpdf.com
e) if the inference is “definitely false”, ie it cannot possibly be drawn from the facts given or it contradicts the given
facts.

Pharmaceutical science involves the synthesis and mass manufacture of drugs from natural and synthetic sources
for the treatment and prevention of diseases. Doctors rely on pharmaceutical companies for the development of new
and more effective drugs to combat diseases. Today, pharmaceutical sector is a highly competitive sector with lots of
companies competing for supremacy and market control. Pharmacists are concerned with production, productive
processes, quality control and testing of drugs. According to Indian government statistics, today the market
comprises over 25000 manufacturing units; one-fifth of them involved in making bulk drugs. The fact that the industry
has been showing nearly 10 per cent growth, indicates the potential it has. With liberalisation in practice, there are
many job opportunities opening up, especially in multinational pharmaceutical companies.

6). Below 5000 manufacture units are involved in making bulk drugs.
7). An interest in life science is an essential qualification to specialise in the field of pharmaceuticals.
8). There is a good range of employment opportunities for specialisers.
9). Indigenous companies are accumulating huge profits as compared to multinational companies.
10). All those who want to become a doctor but fail, end up in pharmacy.

Answers:
1)d 2)b 3)b 4)d 5)b 6)e 7)b 8)a 9)d 10)c

www.bankingpdf.com
Explanation:

Directions (Q.1-5):

6). According to the passage, more than 5000 manufacturing units are involved in making bulk drugs.
Answer: e)

7). Service provided through pharmaceuticals and its nature of job make it probably true.

www.bankingpdf.com
Answer: b)

8). Passage says that pharmacists are concerned with different types of work such as production, productive
processes, quality control and testing of drugs etc. And the last sentence of the passage says that there are many
job opportunities opening up.
Answer: a)

9). Job prospects are better for an individual in MNCs. It is likely MNCs are in a stronger position than indigenous
companies. Hence, the inference drawn seems false.
Answer: d)

10).

Answer: c)

www.bankingpdf.com
740-750 Questions :

Directions (Q. 1-7): Study the following information carefully and answer the given questions.
A group of seven friends A, B, C, D, E, F and Y has four males and three females. Each of them has a different
colour of watch, viz Violet, Pink, Red, White, Green, Blue and Black, and each of them has passed out from a
different college – TVS, AKT, SKC, JKS, VIT, RJEC and SRM but not necessarily in the same order. None of the
females has Blue or Violet. C has Red and she has passed out from VIT. A has passed out from RJEC. B does not
have White. E has Black and has passed out from AKT. F has Violet and has not studied in TVS. G has Blue and has
studied in SKC. The one who has White has passed out from SRM. The one who has Pink has studied in TVS. None
of the female members has studied either in RJEC or in AKT.

1). Which of the following watches does D have?


a) Blue
b) White
c) Pink
d) Red
e) None of these

2). Which of the following combinations is true?


a) C-Red-VIT-Female

www.bankingpdf.com
b) A-Green-AKT-Male
c) G-Pink-TVS-Female
d) E-Black-AKT-Female
e) None of these

3). Which of the following groups has only female members?


a) FDB
b) ECG
c) CDB
d) DAB
e) None of these

4). The one who has Violet has passed out from which of the following colleges?
a) AKT
b) VIT
c) RJEC
d) JKS
e) Cannot be determined

www.bankingpdf.com
5). From which of the following colleges did E pass out?
a) SKC
b) AKT
c) JKS
d) Cannot be determined
e) None of these

6). Who possesses Pink?


a) B
b) C
c) D
d) E
e) F

7). Which of the following statements is true?


a) D has passed out from TVS college.
b) C is a male and possesses Red.
c) F possesses Violet and has passed out from JKS.
d) G is a female and possesses Blue.
e) All are true

www.bankingpdf.com
Directions (Q. 8-9): Read the following information carefully and answer the questions which follow:
‘A × B’ means A is son of B.
‘A + B’ means A is father of B.
‘A > B’ means A is daughter of B.
‘A < B’ means A is wife of B.

8). Which of the following pairs of persons represent first cousins with regard to the relations given in the
expressions, if it is provided that A is the sister of J,
‘L > V < J + P’ and ‘S × A < D + F < E + K’?
a) LP
b) SP
c) SK
d) SF
e) Cannot be determined

9). What will come in place of question mark (?), if it is provided that M is grandmother of F in the expression ‘F × R <
S ? M’?
a) >

www.bankingpdf.com
b) <
c) +
d) ×
e) Cannot be determined

10). In a class of 42 children, Rohit’s rank is sixteenth from the top. Dinesh is seven ranks below Rohit. What is
Dinesh’s rank from the bottom?
a) 22nd
b) 20th
c) 19th
d) 23rd
e) 25th

Answers:
1)b 2)a 3)c 4)d 5)b 6)a 7)c 8)b 9)e 10)b

Explanation:

Directions (Q. 1-7):

www.bankingpdf.com
8).

www.bankingpdf.com
9).

www.bankingpdf.com
10). Dinesh’s rank from the top = (16 + 7) = 23rd

Now Dinesh’s rank from the bottoms = (42 –23 + 1) = 20th

www.bankingpdf.com
750-760 Questions :

Directions (1-5): In these questions, certain symbols have been used to indicate relationships between elements as
follows:

a) A % B means A is either smaller than or equal to B.


b) A - B means A is greater than B.
c) A # B means A is neither greater than nor smaller than B.
d) A $ B means A is smaller than B.
e) A @ B means A is either greater than or equal to B.

1). Statements: U # F , F $ W , W - K
Conclusions:
I. K$U
II. K#U
III. W@U

a) Only I and II follows


b) Only II and III follows
c) Only I and III follows
d) None follows
e) All follows

www.bankingpdf.com
2). Statements: H @ D , D - B , B $ W
Conclusions:
I. B$H
II. W$H
III. D$W

a) Only II follows
b) Only I and III follows
c) Only I follow
d) Only III follow
e) None of these

3). Statements: V $ F , F % P , P - J
Conclusions:
I. J$F
II. P–V
III. V-J

a) None follows
b) Only I follow
c) Only II follow
d) Only III follow

www.bankingpdf.com
e) Only II & III follows

4). Statements: Q $ T , T % G , Q - N
Conclusions:
I. Q$G
II. N$T
III. N–G

a) Either I or II only follows


b) I and II only follows
c) Only I follow
d) II and III only follows
e) None of these

5). Statements: A $ T , T % C , C @ F
Conclusions:
I. A#T
II. A@F
III. C – A.

a) Only II follows
b) Only I and II follows
c) Only III follows

www.bankingpdf.com
d) Only II and III follows
e) None of these

Directions (Q. 6–10): Read the information given below and answer the questions that follow.
Six persons, viz Sagar, Sagun, Sahaj, Sahas, Sahib and Sahil plan to go to six different cities, viz Mumbai, Surat,
Indore, Chennai, Jaipur and Agra. They wish to go in six different months, viz January, March, May, June, August
and December. Two of them want to travel by bus and two by rail and others are planning to go either by car or by
flight. Following information is also given.
Sahib goes to Mumbai but neither by air nor by bus.
Sagar prefers bus journey but travels neither in August nor in May.
Neither Sahil nor Sagun goes either to Jaipur or to Indore.
Chennai is visited in December.
Persons going to Surat and Jaipur go by bus and car respectively.
Mr Sahaj goes to Agra in March by air but
Sahil does not travel in December.

6). Who among the following visits Jaipur?


a) Sagun
b) Sagar
c) Sahil
d) Sahas
e) None of these

www.bankingpdf.com
7). Chennai is visited by
a) Sagar
b) Sahil
c) Sagun
d) Sahas
e) Cannot be determined

8). Sahas visited by


a) Bus
b) Rail
c) Cannot be determined
d) Car
e) None of these

9). If Sahib travelled in June, Sagar must have travelled in


a) January
b) December
c) Cannot be determined
d) August
e) None of these

10). Which of the following places did Sagar visit?

www.bankingpdf.com
a) Agra
b) Indore
c) Surat
d) Mumbai
e) Cannot be determined

Answers:
1)d 2)c 3)c 4)b 5)c 6)d 7)c 8)d 9)a 10)b

EXPLANATIONS:

1). U = F , F < W , W > K


K<U
K=U
W≥U
None follows
Answer: d)

2). H ≥ D , D > B , B < W


B < H follows
W < H not follows
D < W not follows
Answer: c)

www.bankingpdf.com
3). V < F , F ≤ P , P > J
J < F not follows
P > V follows
V > J not follows
Answer: c)

4). Q < T , T < G , Q > N


Q < G follows
N < T follows
N > G not follows
Answer: b)

5). A < T , T ≤ C , C ≥ F
A = T not follows
A ≥ F not follows
C > A follows
Answer: c)

Directions (Q. 6-10):

www.bankingpdf.com
760-770 Questions :

Directions (Q. 1-5): Study the following information and answer the questions given below
Ten people are sitting in two parallel rows containing five people each in such a way that there is an equal distance
between adjacent persons. In a row 1 – Amar, Jesmar, Kumar, Omar and Samar are seated and all of them are
facing north and in row 2 – Abinaya, Mariya, Ramya, Priya and Sofiya are seated and all are facing south (but not
necessarily in the same order). Each person also likes different fruits namely Apple, Orange, Banana, Grapes,
Papaya, Mango, Pears, Cherry, Pineapple and Strawberry (but not necessarily in the same order)

www.bankingpdf.com
Samar sits exactly in the centre of the row and faces the one who likes Grapes. Only one person sits between Samar
and the one who likes Papaya. Priya faces the immediate neighbour of the one who likes Papaya. Only one person
sits between Abinaya and the one who likes Grapes. Abinaya is not an immediate neighbour of Priya. Abinaya faces
one of the immediate neighbours of Kumar. Omar is neither an immediate neighbour of Kumar nor faces Priya.
Mariya is not an immediate neighbour of Priya and faces the one who likes Orange. Omar does not face Ramya.
Omar faces one of the immediate neighbours of the one who likes Mango. Only one person sits between the one
who likes Mango and the one who likes Apple. The one who likes Pineapple and the one who likes Cherry face each
other. Only two people sit between the one who likes Cherry and the one who likes Pears. Amar does not like Pears.
The one who likes Strawberry sits fourth to the right of the one who likes Pineapple.

1). Who amongst the following is an immediate neighbour of the one who likes Pineapple?
a) The one who likes Mango
b) The one who likes Papaya
c) The one who likes Apple
d) Jesmar
e) Sofiya

2). Which of the following fruits does Priya like?


a) Papaya
b) Apple
c) Mango
d) Grapes

www.bankingpdf.com
e) Pears

3). Which of the following is definitely true as per the given arrangement?
a) Kumar faces Ramya
b) Mariya is an immediate neighbour of Abinaya
c) The one who likes Cherry is an immediate neighbour of Abinaya
d) Mariya likes Pears
e) The one who likes Strawberry faces Samar

4). Which of the following pairs represent the people sitting at the extreme ends of the two rows?
a) Priya, Kumar
b) Mariya, Kumar
c) Ramya, Jesmar
d) Priya, Jesmar
e) Abinaya, Amar

5). Who amongst the following likes Banana?


a) Samar
b) Abinaya
c) Priya
d) Mariya
e) Amar

www.bankingpdf.com
Directions (Q. Nos. 6-10) Read the information/ statement given in each question carefully and answer the questions.

6). In which of the following expressions will the expression ‘A < B’ be definitely true?
a) T < A ≥ Y = B
b) A > T ≥ Y = B
c) B = Y ≥ T > A
d) A ≥ T > Y < B
e) None of these

7). Which of the following expressions will be true if the expression ‘C ≥ F > V ≥ W’ is definitely true?
a) W ≤ C
b) C = V
c) C < W
d) F ≥ W
e) None is true

8). Which of the following expressions will be true if the expression ‘E ≥ C < O = S’ is definitely true?
a) S < C
b) E ≥ O
c) E < S
d) O = E
e) None is true

www.bankingpdf.com
9). Which of the following expressions may not be true if the expression ‘H ≥ R = U ≤ G’ is definitely true?
a) U ≤ H
b) G ≥ H
c) R ≤ G
d) Only (2) and (3)
e) All are true

10). In which of the following expressions does the expression ‘P > S’ hold true?
a) P = Q < R ≤ S
b) S ≥ Q > R > P
c) Q = S > R ≥ P
d) P ≥ R >Q = S

e) S ≤ Q > P > R

Answers:

1)c 2)c 3)b 4)e 5)a 6)c 7)e 8)e 9)b 10)d

www.bankingpdf.com
770-780 Questions :

Directions (Q. 1-5): In the following questions, certain symbols are used with the following meanings:
a) A # B means A is not greater than B.
b) A $ B means A is neither smaller than nor equal to B.
c) A ? B means A is neither greater than nor smaller than B.
d) A * B means A is neither greater than nor equal to B.
e) A @ B means A is not smaller than B.

1). Statements: P $ Q, R @ S, P * R
Conclusions:
I. Q*R
II. P#S
III. P$S

a) None follows
b) Only I follow
c) Only II follows
d) Only III follows
e) Only II & III follows

2). Statements: U $ V, W * X, U @ X
Conclusions:

www.bankingpdf.com
I. V@X
II. V*X
III. U$W

a) Only I follow
b) I and II only follows
c) Only III follows
d) II and III only follows
e) None of these

3). Statements: K # T, D $ F, T * F
Conclusions:
I. K*D
II. D$T
III. T$D

a) Only I and II follows


b) Only II and III follows
c) Only I and III follows
d) None follows
e) All follows

4). Statements: M$N, G@H, N?H

www.bankingpdf.com
Conclusions:
I. M@H
II. M$G
III. G@N

a) Only II follows
b) Only I and III follows
c) Only III follows
d) Only I follow
e) None of these

5). Statements: G@M, N#L, G*L


Conclusions:
I. G@N
II. L$M
III. N$G

a) Only I and II follows


b) Only II follows
c) Only I and III follows
d) None follows
e) None of these

www.bankingpdf.com
Directions (Q. 6-11): Study the following information and answer the questions given below
Seven persons Ishan, Ishana, Ishit, Ishir, Ishat, Ishrat and Ishwar belong to seven different states, viz Assam,
Karnataka, Maharashtra, Haryana, Manipur, Odisha and Sikkim but not necessarily in the same order. Each of them
uses a different sim, viz Airtel, BSNL, Reliance, Aircel, Docomo, Vodafone and Idea but not necessarily in the same
order. Ishana uses neither Airtel nor Idea. Ishir belongs to Maharashtra. Ishit belongs to Karnataka and uses Aircel.
Ishat and Ishrat do not hail from Haryana. The one who hails from Haryana uses BSNL. Neither Docomo nor Airtel
used by Ishir. Ishana hails from Manipur. The one who uses Idea is either from Sikkim or from Manipur. Ishrat does
not like Airtel or Docomo. Ishan uses Reliance, Ishwar is not from Odisha.

6). Who among the following uses Idea?


a) Ishat
b) Ishrat
c) Ishwar
d) Cannot be determined
e) None of these

7). Ishir uses which of the following sims?


a) Vodafone
b) Reliance
c) BSNL
d) Docomo

www.bankingpdf.com
e) None of these

8). The one who uses Vodafone is from which of the following states?
a) Manipur
b) Assam
c) Haryana
d) Maharashtra
e) None of these

9). If the person who uses Airtel is from Odisha, then the following person who uses Reliance is from which of the
following States?
a) Haryana
b) Maharashtra
c) Assam
d) Cannot be determined
e) None of these

10). Which of the following Statements is/are true?


a) The one who is from Odisha uses Reliance
b) Ishir hails from Maharashtra and uses Vodafone
c) The one who is from Manipur uses Airtel
d) All are true

www.bankingpdf.com
e) None is true

11). Which of the following combinations is false?


a) Ishana – Manipur - Docomo
b) Ishrat – Sikkim - Idea
c) Ishwar – Haryana - BSNL
d) All are false
e) All are true

Answers:
1)b 2)c 3)a 4)c 5)b 6)b 7)a 8)d 9)c 10)b 11)e

Explanation:

1). P > Q ..... (i) R ≥ S ..... (ii), P < R ..... (iii)


R > Q follows
P ≤ S not follows
P > S not follows

2). U >V .... (i), W < X ..... (ii) , U ≥ X ...... (iii)


V ≥ X not follows
V < X not follows

www.bankingpdf.com
U > W follows

3). K ≤ T ... (i), D > F ...(ii), T < F ...(iii)


D > T follows
D > K follows
T > D not follows

4). M>N, G≥H, N=H


M ≥ H not follows
M > G not follows
G ≥ N follows

5). G≥ M..(i), N ≤ L...(ii) G < L .... (iii)


G ≥ N not follows
L > M follows
N > G not follows

Directions (Q. 6-11):


Person State Sim

Ishan Assam / Odisha Reliance

www.bankingpdf.com
Ishana Manipur Docomo

Ishit Karnataka Aircel

Ishir Maharashtra Vodafone

Ishat Assam / Odisha Airtel

Ishrat Sikkim Idea

Ishwar Haryana BSNL

www.bankingpdf.com
780-790 Questions :

Directions (Q. 1-5): Study the following information carefully and answer the given questions.
Ten sportsmen Sushil Kumar, Gagang Narang, Abhinav Bindra, Vijendar Singh, Sandeep Tomar, Jitu Rai, Vikas
Gowda, Arun Kumar, Shiva Thapa and Vijay Kumar are travelling by ten different airlines to ten different destinations.
The airlines are Air India, Air Asia, Vistara, Jet Airways, Go Air, Spicejet, Indigo, Truejet, Tajair and Easy Air. The ten
destinations are China, Japan, Netherland, Spain, Germany, Ireland, Australia, France, Argentina and Poland, but
not necessarily in the same order.
Gagang Narang is travelling to Japan
Sandeep Tomar is travelling by Go Air and going to Germany
The person travelling by Easy Air is going to Poland
Vikas Gowda is travelling by Indigo
Shiva Thapais travelling to Argentina
Arun Kumar is travelling to France
Jitu Rai is travelling to Ireland by Spicejet
Vijendar Singh is travelling by Jet Airways to Spain
Neither Sushil Kumar nor Gagang Narang is travelling by Tajair
Shiva Thapa is not travelling by Air India or Air Asia
Sushil Kumar travels neither by Air India nor by Truejet
The person who travels to Netherland is using Vistara
Sushil Kumar is travelling to China

www.bankingpdf.com
Abhinav Bindra is travelling by Vistara and Arun Kumar by Truejet.

1). Shiva Thapa is travelling by which airline?


a) Truejet
b) Go Air
c) Air India
d) Other than given options
e) Vistara

2). Vikas Gowda is travelling to which destination?


a) Australia
b) Argentina
c) Poland
d) Japan
e) Germany

3). Which of the following pairs is correct?


a) Vijay Kumar– Easy Air - Germany
b) Arun Kumar – Truejet - Netherland
c) Abhinav Bindra – Jet Airways - Netherland
d) Jitu Rai – Spicejet - Ireland
e) Vikas Gowda – Go Air – Australia

www.bankingpdf.com
4). Gagang Narang is travelling by which Airline?
a) Air India
b) Air Asia
c) Truejet
d) Tajair
e) Either Air India or Air Asia

5). Sushil Kumar is travelling by which airline and to which destination?


a) Spicejet - Ireland
b) Jet Airways - Spain
c) Air Asia - Netherland
d) Air Asia - China
e) Vistara – Netherland

Directions (Q. 6-10): The following questions are based on the five three-letter words given below
DOG NID CUP HOT PUT
(Note: The words formed after performing the given operations may or may not be meaningful English words)

6). If the positions of the first and the third letters of all the words are interchanged, how many words will form
meaningful English words?
a) None

www.bankingpdf.com
b) One
c) Two
d) Three
e) None of these

7). If the first letter of all the words is changed to the next letter of the English alphabetical series, how many words
will have more than one vowel?
a) One
b) Two
c) Three
d) Four
e) None

8). If the positions of the first and the second letters of all the words are interchanged, how many words will form
meaningful English words?
a) None
b) One
c) Two
d) Three
e) None of these

www.bankingpdf.com
9). If all the letters in each of the words are arranged alphabetically (within the word) how many words will remain
unchanged?
a) None
b) One
c) Two
d) Three
e) None of these

10). How many letters are there in English alphabet between the second letter of the second word and the third letter
of the fourth word?
a) Seven
b) Eight
c) Nine
d) Ten
e) More than ten

Answers:
1)d 2)a 3)d 4)a 5)d 6)c 7)c 8)a 9)b 10)d

Explanation:

www.bankingpdf.com
Directions (Q. 1-5):

6). Given words: DOG, NID, CUP, HOT, PUT


Now, GOD, DIN, PUC, TOH, TUP
There are only two meaningful words – GOD and DIN.

7). After changing EOG OID DUP IOT QUT


Thus there are three words having more than one vowel. These words are EOG, OID and IOT.

8). After interchanging: ODG IND UCP OHT UPT

www.bankingpdf.com
9). DGO DIN CPU HOT PTU
There is only one such word – HOT.

10). The second letter of the second word is I. The third letter of the fourth word is T.
IJKLMNOPQRST
There are ten letters between I and T.

790-800 Questions :

Directions (Q. 1-3): Study the following information carefully and answer the given questions.
A and Y are brothers of K. Y is son of P and S. P is Daughter of X. M is father in law of S. Q is son of X.

1). If J is Brother of X, then how is J related to Q?


a) Uncle
b) Nephew
c) Cannot be determined
d) Brother -in-law
e) Son-in-law
2). How is Y related to M?
a) Nephew
b) Father
c) Brother-in-law

www.bankingpdf.com
d) Grandson
e) Brother
3). How is K related to Q?
a) Niece
b) Daughter
c) Nephew
d) Son-in-law
e) Cannot be determined
Directions (Q. 4-5): Study the following information carefully and answer the given questions.
P + Q means P is sister of Q
P @ Q means P is father of Q
P ÷ Q means P is son of Q
P % Q means P is mother of Q

4). What will come in place of question mark (?) if A is grandson of B is to be true in the given expression? B % D @
A?E
a) +
b) @
c) ÷
d) %
e) None of these

www.bankingpdf.com
5). In which of the following pairs is the first person daughter of the second person with regard to the relation given in
the expression?
F@G÷I+J+H
a) I, G
b) I, H
c) G, J
d) J, H
e) None of these
Directions (Q. 6-10): Study the following information carefully and answer the given questions.
A, B, C, D, E, F, G and H are sitting around a circular area of equal distances between each other, but not necessarily
in the same order. Some of the people facing the centre while some face outside (ie in a direction opposite the centre).
Note: Facing the same direction means if one faces the centre then the other also faces the centre and vice versa.
Facing opposite direction means if one person faces the centre then the other faces outside and vice versa.
G sits second to the right of D. D faces the centre. H sits second to the left of G. H is an immediate neighbour of both
C and E. F sits second to the left of E. A sits second to the left of F. H sits second to the left of B. A sits second to the
left of C. H faces the same direction as E. B sits third to the right of A.

6. Who sits exactly between G and H when counted from the left of G?
a) A
b) E
c) D

www.bankingpdf.com
d) C
e) B
7. Who sits second to the right of B?
a) F
b) D
c) H
d) G
e) C
8. Four of the following five are alike in certain way based on the given seating arrangement and so form a group.
Which is the one that does not belong to that group?
a) F
b) G
c) E
d) C
e) A
9. Which of the following is true regarding H as per the given seating arrangement?
a) H faces the centre
b) Only two people sit between B and H
c) Only three people sit between H and D
d) G sits second to the left of H
e) Only one person sits between H and F

www.bankingpdf.com
10. What is A’s position with respect to H?
a) Second to the left
b) Fifth to the right
c) Third to the right
d) Third to the left

e) Fourth to the right

Answer:
1).a 2).d 3).e 4).c 5).e 6).d 7).b 8).d 9).c 10).c

Directions(1-3):

www.bankingpdf.com
Directions(6-10):

www.bankingpdf.com
800-810 Questions :

Directions (Q. 1-5): In each of the question below are given four statements followed by three conclusions numbered
I, II and III. You have to take the given statements to be true even if they seem to be at variance from commonly
known facts. Read all the conclusions and then decide which of the given conclusion logically follows from the given
statements disregarding commonly known facts.

1). Statements: Some villages are town.


Some towns are huts.
All huts are rivers.
Some rivers are tents.

www.bankingpdf.com
Conclusion:
I. Some tents are towns.
II. Some rivers are towns.
III. Some huts are villages.
a) None follows
b) Only I follows
c) Only II follows
d) Only III follows
e) only II and III follows

2). Statements: All hotels are buses.


Some buses are cars.
All cars are trains.
Some trains are clouds.
Conclusions:
I. some trains are buses.
II. some trains are hotels.
III. some clouds are cars.
a) None follows
b) Only I follows

www.bankingpdf.com
c) Only II follows
d) Only III follows
e) Only I and III follows

3). Statements: All flowers are books.


All books are carpets.
Some carpets are keys.
Some keys are locks.
Conclusions:
I. some keys are books.
II. some keys are flowers.
III. some locks are books.
a) Only I follows
b) Only II follows
c) Only III follows
d) Only I and II follow
e) None follow

4). Statements: All boxes are cups.


All chairs are cups.

www.bankingpdf.com
All cups are mirror.
All tables are mirror.
Conclusions:
I Some tables are chairs.
II some mirrors are boxes.
III some mirrors are chairs.
a) Only I and II follows
b) Only I and III follows
c) Only II follows
d) Only III follows
e) Only II and III follow

5). Statements: Some pins are needles.


All needles are ropes.
Some ropes are buckets.
All buckets are trees.
Conclusions:
I Some buckets are pins.
II some ropes are pins.
III No bucket is pin.

www.bankingpdf.com
a) Only either I or III and II follow
b) Only either I or III follows
c) Only II follows
d) Only either I or II and III follow
e) None of these

Directions (Q. 6-10): In each of these questions a group of letters is given followed by some combination of digits and
symbols numbered (a), (b) and (c). Letters are to be coded by digit/symbol as per the scheme and conditions given
below. Serial number of the combination of digit/symbol that represents the group of letters is your answer. If none of
the combinations is correct, your answer is (e).
L F J T A H I M R E K D U B P

3 7 # $ 4 9 1 8 2 @ 6 % © * 5

Conditions: (i) if the first letter is a consonant and the last letter is a vowel their codes are to be interchanged.
(ii) if the first as well as the last letter is a vowel both are to be coded as £.
(iii) if the first as well as the last letter is a consonant and in between there are two or more vowels all the vowels are
to be coded as ¥
(Iv) if the first letter is a vowel and the last letter is a consonant, both are to be coded as the coded for consonant.
6). EFHKLB
a) @7963*

www.bankingpdf.com
b) *7963*
c) @7963@
d) *7963@
e) None of these

7). FTAMIR
a) 7$4812
b) 7$1842
c) 7$¥8¥2
d) 7#$8$2
e) None of these

8). DKPRTB
a) *652$%
b) E2$%
c) £652$£
d) *652*
e) None of these

www.bankingpdf.com
9). UPJTRA
a) 45#$2©
b) ©5#$24
c) ©5#$2©
d) $5#2$$
e) None of these

10). HLEKBI
a) 13@6*9
b) 93@6*1
c) 13@6*1
d) 936@*1
e) None of these

Answer:
1)c 2)b 3)e 4)e 5)a 6)b 7)c 8)e 9)e 10)a

Solution:
1).

www.bankingpdf.com
Answer: c)
2).

Answer: b)
3).

www.bankingpdf.com
Answer: e)
4).

www.bankingpdf.com
Answer: e)
5).

Answer: a)

6). E F H K L B : * 7 9 6 3 *
Condition (iv) applied here. Hence, option (b).
Answer: b)

7). F T A M I R : 7 $ ¥ 8 ¥ 2
Condition (iii) applied here. Hence, option (c).
Answer: c)

www.bankingpdf.com
8). D K P R T B : % 6 5 2 $ *
No condition is applicable.
Answer: e)

9). U P J T R A : £ 5 # $ 2 £
Condition (ii) applicable. Hence, option (e).
Answer: e)

10). H L E K B I : 1 3 @ 6 * 9
Condition (i) is applicable. Hence, option (a).
Answer: a)

www.bankingpdf.com
810-820 Questions :

Directions (01 to 05): Study the following information carefully and answer the given questions.

A, B, C, D, E, F, G, and H are sitting around a circular table facing the centre. Each one of them has a different
profession viz. Driver, Dentists, Lawyer, Professor, Farmer, Accountant, Shop Keeper and Physician.
The Physician sits third to right of G. C is an immediate neighbor of G. Lawyer sits second to right of C. B sits third of
H. H is neither Physician nor an Lawyer. Only one person sits between C and the Accountant. A and F are immediate
neighbors of each other. Neither A nor F is a Physician. Driver sits second to right of A. Two people sit between D
and the Dentists. D is not a Physician. Professor is not an immediate neighbor of the Physician. Farmer sits second
to left of A.

1). What is the position of F with respect to the Physician?


a) Immediate right
b) Third to the left
c) Second to the right
d) Fourth to the left
e) Second to the left

www.bankingpdf.com
2). What is the profession of G?
a) Shop Keeper
b) Professor
c) Farmer
d) Accountant
e) Lawyer

3). Four of the following five are alike in a certain way based on the given seating arrangement and thus form a
group. Which is the one that does not belong to that group?
a) E-Physician
b) G-Driver
c) A-Lawyer
d) H-Shop Keeper
e) D-Professor

4). Who sit(s) exactly between the Accountant and the Dentists?
a) G and H
b) The Physician
c) The Driver
d) C and B

www.bankingpdf.com
e) The Lawyer and the Shop Keeper

5). Which of the following is true with respect to the given seating arrangement?
a) Driver sits second to the right of the Shop Keeper
b) D is an immediate neighbor of G
c) G is a Farmer
d) The Farmer and the Accountant are immediate neighbors of each other
e) Professor sits exactly between Lawyer and the Shop Keeper

Direction (06 to 10): Study the following information carefully and answer the given questions:
A word and number arrangement machine when given an input line of words and numbers rearranges them following
a particular rule in each step. The following is an illustration of input and rearrangement.
Input: world 32 73 verb 26 new desk 19
Step I: 73 world 32 verb 26 new desk 19
Step II: 73 desk world 32 verb 26 new 19
Step III: 73 desk 32 world verb 26 new 19
Step IV: 73 desk 32 new world verb 26 19
Step V: 73 desk 32 new 26 world verb 19
Step VI: 73 desk 32 new 26 verb world 19

www.bankingpdf.com
And Step VII is the last step of the above input. As per the rules followed in the above steps, find out in each of the
following questions the appropriate step for the given input.

6). Step II of an input is: 51 brown 22 36 49 cloud sky red. How many more steps will be required to complete the
rearrangement?
a) Three
b) Four
c) Five
d) Six
e) None of these

7). Step III of an input is : 58 dine 43 18 tower silver mat 24, which of the following will be step VI?
a) 58 dine 43 mat 24 silver 18 tower
b) 58 dine 43 mat 24 18 tower silver
c) 58 dine 43 mat 18 tower silver 24
d) There will be no such step
e) None of these

8). Input: 85 23 96 case over for 42 win. How many steps will be required to complete the rearrangement?

www.bankingpdf.com
a) Four
b) Seven
c) Five
d) Six
e) None of these

9). Step IV of an input is: 63 car 51 eyes 25 36 store lane. Which of the following is definitely the input?
a) Eyes car 25 63 36 51 store lane
b) Eyes 25 car 63 51 36 store lane
c) Eyes car 51 63 36 store lane
d) Cannot be determined
e) None of these

10). Input: field eyes 94 32 house rent 49 27 , Which of the following steps will be the last but one?
a) VI
b) V
c) VII
d) VIII
e) None of these

www.bankingpdf.com
Answers:
1. C) 2. B) 3. A) 4. C) 5. A) 6. B) 7. A) 8. C) 9. D) 10. E)

Explanation:
Direction: (01 to 05):

Direction: (06 to 10):


6). Step I: Biggest number should be placed in the first position.
Step II: words in alphabetical order should be placed second

www.bankingpdf.com
Step III: second biggest number is placed third. It continuous till all numbers are in descending order and words are
in alphabetical order
Step II: 51 brown 22 36 49 cloud sky red
Step III: 51 brown 49 22 36 cloud sky red
Step IV: 51 brown 49 cloud 22 36 sky red
Step V: 51 brown 49 cloud 36 22 sky red
Step VI: 51 brown 49 cloud 36 red 22 sky
Hence 4 more steps are required to complete the arrangement.
7). Step I: Biggest number should be placed in the first position.
Step II: words in alphabetical order should be placed second
Step III: second biggest number is placed third
It continues till all numbers in descending order and words are in alphabetical order
Step III: 58 dine 43 18 tower silver mat 24
Step IV: 58 dine 43 mat 18 tower silver 24
Step V: 58 dine 43 mat 24 18 tower silver
Step VI: 58 dine 43 mat 24 silver 18 tower
8). Step I: Biggest number should be placed in the first position.
Step II: words in alphabetical order should be placed second
Step III: second biggest number is placed third
It continues till all numbers in descending order and words are in alphabetical order
Input: 85 23 96 case over for 42 win

www.bankingpdf.com
Step I: 96 85 23 case over for 42 win
Step II: 96 case 85 23 over for 42 win
Step III: 96 case 85 for 23 over 42 win
Step IV: 96 case 85 for 42 23 over win
Step V: 96 case 85 for 42 over 23 win
Hence, five steps are required to complete the arrangement.
9). Step I: Biggest number should be placed in the first position.
Step II: words in alphabetical order should be placed second
Step III: second biggest number is placed third
It continues till all numbers in descending order and words are in alphabetical order
As the initial positions of the elements are not known, we cannot determine input.
10). Step I: Biggest number should be placed in the first position.
Step II: words in alphabetical order should be placed second
Step III: second biggest number is placed third
It continues till all numbers in descending order and words are in alphabetical order
Input: field eyes 94 32 house rent 49 27
Step I: 94 field eyes 32 house rent 49 27
Step II: 94 eyes field 32 house rent 49 27
Step III: 94 eyes 49 field 32 house rent 27
Step IV: 94 eyes 49 field 32 house 27 rent
Hence third step is last but one step

www.bankingpdf.com
820-830 Questions :

Directions (Q. 1-5): Each of the questions below consists of a question and two statements numbered I and II given
below it. You have to decide whether the data provided in the statements are sufficient to answer the questions.
Read both the statements and
Give answer (a) if the data in statement I alone are sufficient to answer the question, while the data in statement II
alone are not sufficient to answer the question
Give answer (b) if the data in statement II alone are sufficient to answer the question, while the data in statement I
alone are not sufficient to answer the question
Give answer (c) if the data either in statement I or in statement II alone are sufficient to answer the question
Give answer (d) if the data even in both statements I and II together are not sufficient to answer the question
Give answer (e) if the data in both statements I and II together are necessary to answer the question

1). What is the colour of white snow in a colour code?


I. ‘Green’ is called ‘Black’, ‘Black’ is called Blue, and ‘Blue’ is called ‘Red’.
II. ‘Red’ is called ‘White’ and ‘White’ is called ‘Orange’.

2). Who is oldest among Peter Kelvin, Joseph and Jason?


I. Jason is older than Peter and Joseph.

www.bankingpdf.com
II. Kevin is younger than Joseph.

3). Six people P, Q, R, S, T and U are seated around a circular table and are equidistant from each other. Who is 2nd
to the right of T?
I. P is to the immediate left of Q and Q sits opposite R.
II. S is to the immediate left of U.

4). How is ‘happy’ written in a code language?


I. ‘I happy today’ is written as ‘ke ne que’ and ‘today happy day’ is written as ‘ke joi ne’.
II. ‘I play’ is written as ‘qua pa’.

5). H is the mother of J. How is J related to V?


I. V is the only daughter of H.
II. V is the sister of J.

Directions (Q. 6–10): Study the following information carefully and answer the questions given below.
Seven players A, B, C, D, E, F and G from India from seven different states – Assam, Rajasthan, Telangana,
Tamil Nadu, Andra Pradesh, Odisha and Maharashtra are participate in the Olympics games but not necessarily in
the same order. All of them participate in seven different games, viz Badminton, Archery, Wrestling, Hockey, Golf,
Gymnastics and Shooting, but not necessarily in the same order. The one who participates in Wrestling does not
belong to Andra Pradesh. D is from Telangana and he does not participate in Hockey. The one who is from Tamil

www.bankingpdf.com
Nadu participates in Badminton. The one who is from Tamil Nadu participates in Badminton. The one who is from
Rajasthan does not participate in Hockey or Golf. A is from Maharashtra. G is from Assam and participates in
Gymnastics. C participates in Shooting but he is neither from Odisha nor from Andra Pradesh. B participates in
Archery and E participates in Wrestling.

6). D participates in which of the following games?


a) Wrestling
b) Archery
c) Golf
d) Cannot be determined
e) None of these

7). Who among the following participates in Hockey?


a) D
b) A
c) E
d) Cannot be determined
e) None of these

8). F is from which of the following states?


a) Assam

www.bankingpdf.com
b) Telangana
c) Tamil Nadu
d) Odisha
e) None of these

9). Which of the following combinations is correct?


a) B – Andra Pradesh - Archery
b) E – Telangana - Wrestling
c) D – Telangana - Hockey
d) All of these
e) None of these

10). The one who is from Rajasthan participates in which of the following games?
a) Hockey
b) Gymnastics
c) Golf
d) Shooting
e) None of these

Answer:

www.bankingpdf.com
1. B 2. E 3. E 4. D 5. A 6. C 7. B 8. C 9. A 10. d

Direction (01 to 05):


1).
Colour of white snow is ‘white’.
From II ‘white’ is called ‘orange’
So, the white snow is ‘orange’
So, data in statement II alone are sufficient to answer the question.
2).
I. Jason > (Peter and Joseph)
II. Joseph > Kevin
So, from both statements, Jason is oldest among all.
So, data in both statements I and II together are necessary to answer the question.
3). e
4).
I. I happy today = ke ne que … (i)
Today happy day = kejoi ne … (ii)
From (i) and (ii), the code of ‘happy’ = ke or ne
II. I play = que pa
So, from both statements, the code of happy cannot exactly determined.

www.bankingpdf.com
So, the data even in both statements I and II together are not sufficient to answer the question.
5). A
Direction (06 to 10):

Player States Game

A Maharashtra Hockey

B Andra Pradesh Archery

C Rajasthan Shooting

D Telangana Golf

E Odisha Wrestling

F Tamil Nadu Badminton

G Assam Gymnastics

www.bankingpdf.com
830-840 Questions :

Directions (Q. 1–5): In each question below are given three or four statements followed by four conclusions
numbered I, II, III and IV. You have to take the given statements to be true even if they seem to be at variance with
commonly known facts. Read all the conclusions and then decide which of the given conclusions logically follows
from the given statements, disregarding commonly known facts.

1). Statements : Some dogs are rats


All rats are trees
Some trees are not dogs
Conclusions :
(i) Some trees are dogs
(ii) All dogs are trees
(iii) All rats are dogs
(iv) All trees are dogs

a) None follows
a) b) Only i follows
b) c) Only i and ii follows
c) d) Only ii and iii follows
d) All follows

www.bankingpdf.com
2). Statements : Some boys are rains
All rains are clouds
Some clouds are cars
Conclusions :
(i) Some clouds are boys
(ii) Some cars are boys
(iii) Some cars are rains
(iv) Some rains are boys

a) None follows
a) b) Only iv follows
b) c) Only i follows
c) d) Both i and iv follows
d) All follows

3). Statements : All bricks are flowers


Some houses are flowers
All pens are houses
Conclusions :
(i) Some houses are bricks
(ii) Some pens are flowers
(iii) Some flowers are bricks

www.bankingpdf.com
(iv) No pen is flower

a) a) Only either ii or iv and iii follows


b) b) Only either ii or iv and i follows
c) c) Only either i or ii and iv follows
d) None follows
e) All follows

4). Statements : All lions are ducks


No duck is a horse
All horses are fruits
Conclusions :
(i) No lion is horse
(ii) Some fruits are horses
(iii) Some ducks are lions
(iv) Some lions are horses

a) All follows
a) b) Only either i or ii and both iii and iv follows
b) c) Only either i or iv and both ii and iii follows
c) d) Only either i or iv follows
d) None of these

www.bankingpdf.com
5). Statements : Some mountains are rivers
Some rivers are roads
Some roads are windows
Conclusions :
(i) Some windows are not roads
(ii) Some rivers are mountains
(iii) Some roads are mountains
(iv) Some windows are rivers

a) All follows
b) b) Only i and ii follows
c) c) Only iii and iv follows
d) d) Only i and iv follows
e) None follows

Directions (Q. 6–10): Study the following information carefully and answer the questions given below.
Ramesh, Ganesh, Lokesh, Dinesh, Harish and Suresh are six boys each belonging to a different state, viz
Maharashtra, Assam, Kerala, Manipur, Punjab and Rajasthan, not necessarily in the same order. Each of them got
selected in a different bank, viz Indian Bank, Allahabad Bank, IDBI Bank, Corporation Bank, Punjab National Bank
and Indian Overseas Bank, not necessarily in the same order.

www.bankingpdf.com
Ganesh belongs to Rajasthan but did not get selected in either Punjab National Bank or Indian Bank. Dinesh doesn’t
belong either to Maharashtra or to Manipur, but got selected in Allahabad Bank. The one who got selected in Punjab
National Bank doesn’t belong to Rajasthan. The one who got selected in Indian Overseas Bank belongs to Manipur.
Suresh did not get selected in Punjab National Bank. Either Lokesh or Suresh got selected in IDBI Bank but neither
of them belongs to Punjab or Manipur. Ramesh belongs to Kerala and he got selected in either Indian Bank or IDBI
Bank. Suresh doesn’t belong to Maharashtra.

6). Who among the following belongs to Punjab?


a) Ramesh
b) Ganesh
c) Lokesh
d) Dinesh
e) Cannot be determined

7). Who among the following got selected in Punjab National Bank?
a) Ramesh
b) Ganesh
c) Lokesh
d) Dinesh
e) Cannot be determined

8). The one who got selected in Corporation Bank belongs to which of the following states?

www.bankingpdf.com
a) Maharashtra
b) Rajasthan
c) Punjab
d) Assam
e) None of these

9). The one who belongs to Assam got selected in which of the following banks?
a) IDBI Bank
b) Punjab National Bank
c) Either IDBI Bank or Punjab National Bank
d) Corporation Bank
e) Cannot be determined

10). Which of the following combinations of person–state– bank is definitely false?


a) Ramesh–Kerala–Indian Bank
b) Dinesh–Punjab–Allahabad Bank
c) Suresh–Manipur–Indian Overseas Bank
d) Lokesh–Maharashtra–Punjab National Bank
e) None of these

www.bankingpdf.com
840-850 Questions :

Directions (Q. 1-5): Read the following statements and answer the questions that follow:
At a conference, four phDs specializing in different domains are seated in a row of four chairs left to right, each of a
different colour. Each person has chosen to drink a unique drink (from tea, juice, cold-drink and energy drink). The
following information is provided:
A. The PhD in English is seated one the red chair.
B. The PhD in Physics drinks juice.
C. The PhD in Anthropology is seated on the first seat on the left.
D. The PhD seated on the second seat from the right drinks cold drink.
E. The PhD in Mathematics drinks tea.
F. Juice is drunk by the person sitting on the blue chair.
G. The white chair is placed to the right to the red chair.
H. The chair colours are: white, blue, red and yellow

1). The chair color for the Anthropology PhD is :


a) Yellow
b) Blue
c) Red
d) While
e) None of these

www.bankingpdf.com
2).Which PhD drinks juice?
a) Anthropology
b) Physics
c) English
d) Mathematics
e) None of these

3). The mathematics professors sits in the chair of which color and drinks which drink?
a) Red, cold drink
b) White, Tea
c) Yellow, energy drink
d) Blue, juice
e) None of these

4). Which of the following statement is true?


a) The PhD in Mathematics sits in the white colored chair number 4 and drinks a cold drink.
b) The PhD in English sits in the yellow colored chair number 3 and drinks a cold drink.
c) The PhD in Physics sits in the blue colored chair number 3 and drinks juice.
d) The PhD in Anthropology sits in the yellow colored chair number 1 and drinks an energy drink.
e) None of theses

www.bankingpdf.com
5). Which PhD drinks cold drink?
a) Anthropology
b) Physics
c) English
d) Mathematics
e) None of these

Directions (Q. 6-10): Eight persons, Rajeev, Ramesh, Raghu, Armaan, Dinesh, Ankit, Atul and Arvind arrived at a
shopping mall in their respective cars. Each of these eight people have his own car out of the following brands: Audi,
Alto, Verna, i-10, Maruti, Nano, Swift and Tata Safari. No two people have the same brand of car. Armaan was the
3rd person to arrive in his Audi but after the person who owns i-10 car. Ankit was not the last person to arrive. Atul
arrived after Raghu but just before Rajeev, who owns Maruti. The person who owns Alto was the last person to
arrive. Neither raghu nor Dinesh own i-10. The person who owns Swift arrived after the person who owns Verna.
Dinesh, who owns Tata Safari, was first to arrive. Arvind owns Nano and he came after Raghu but not just before the
person having Alto.

6). Who was the second last person to arrive?


a) Raghu

www.bankingpdf.com
b) Rajeev
c) Ramesh
d) Atul
e) Cannot be determined

7). Who owns the i-10 car?


a) Raghu
b) Atul
c) Rajeev
d) Ankit
e) None of these

8). Which all brands of cars arrived after the Verna?


a) Alto, Swift, Nano, Maruti
b) Nano, Alto, Maruti
c) Maruti, Alto
d) Audi, Swift, Nano
e) None of these

www.bankingpdf.com
9). Which of the following statements is incorrect?
a) Raghu owns a Swift
b) Three cars arrived after the Nano.
c) Two persons arrived between Ankit and Arvind
d) Ramesh was the last person to arrive.
e) None of these

10). Had the eight persons arrived in the alphabetical order of their names, who must have arrived just after the
person who arrived in Verna?
a) Dinesh
b) Arvind
c) Raghu
d) Rajeev
e) None of these

Answers:
1)a 2)b 3)b 4)d 5)c 6)b 7)d 8)c 9)e 10)a

Explanation:
Directions (Q. 1-5):

www.bankingpdf.com
Subject Chair colour Drink of choice Chair number

Anthropology Yellow Energy Drink 1

Physics Blue Juice 2

English Red Cold Drink 3

Mathematics White Tea 4

Directions (Q. 6-10):

www.bankingpdf.com
850-860 Questions :

Directions (Q. 1-5): Study the following information carefully and answer the given questions.
In a certain code language ‘speak the truth’ is written as ‘ke la ta’, ‘always seek knowledge’ is written as ‘bi ti na’, ‘
knowledge is truth’ is written as ‘ti ta si’ and ‘never seek violence’ is written as ‘li bi za’.

1). What does the code ‘ta’ stand for?


a) truth
b) the
c) speak
d) the or speak
e) N one of these
2). Which of the following can be a code for ‘violence is always seek’?
a) si na bi li
b) za li bi na
c) bi na ta za
d) Can’t be determined
e) None of these
3). Which of the following codes stands for ‘always’?
a) ti
b) na

www.bankingpdf.com
c) bi
d) ke
e) None of these
4). ‘ke la li za’ is the code for which of the following?
a) knowledge is always speak
b) the violence is truth
c) never speak the violence
d) Can’t be determined
e) None of these
5). Which of the following is the code for ‘is’?
a) ta
b) bi
c) si
d) ti
e) None of these
Directions (Q. 6 - 10): Study the following information carefully and answer the given questions.
A cubical box is painted in six different colours on different faces. Green face is painted adjacent to Pink face. White
face is painted between Violet and Grey face. Black face is painted adjacent to Green face. Violet face is painted
opposite the Grey face. Pink face and Black face are painted opposite each other. Grey is painted on the base.

6). Which of the following group of colours is adjacent to Black face?

www.bankingpdf.com
a) Green, Pink, White
b) Pink, Violet, Green
c) Grey, White, Pink
d) Violet, Green, White
e) None of these
7). Which of the following colours is opposite to White face?
a) Black
b) Green
c) Violet
d) Grey
e) None of these
8). Which of the following colours is between Grey and Violet?
a) Black
b) Pink
c) Green
d) White
e) All these colours
9). Which of the following colours is/are adjacent to Pink and Violet faces?
a) Green and White
b) White and Grey
c) Grey

www.bankingpdf.com
d) Black
e) None of these
10). If the Green face and Black face are interchanged then which of the following colours is not adjacent to Green
face?
a) Grey
b) Pink
c) Violet
d) White
e) None of these

Answers:
1)a 2)a 3)b 4)c 5)c 6)d 7)b 8)e 9)a 10)b

Explanation:

Directions (1-5):
speak the truth - ke la ta ... (i)
always seek knowledge - bi ti na ... (ii)
knowledge is truth - ti ta si ... (iii)
never seek violence - li bi za ... (iv)

www.bankingpdf.com
From (i) and (iii),
Truth - ta ... (v)
From (ii) and (iv),
Seek - bi ... (vi)
From (ii) and (iii),
Knowledge - ti ... (vii)
From (iii), (v) and (vii),
Is - si
From (i) and (v)
Speak/the - ke/la ... (viii)
From (ii), (vi) and (vii)
Always - na ... (ix)
From (iv) and (vi)
never/violence - li/za ... (xii)

Directions (6 -10)

www.bankingpdf.com
Violet face is opposite Grey
Pink face is opposite Black

Green face is opposite White

www.bankingpdf.com
860-870 Questions :

Directions (1 – 5): Study the following information carefully and answer the questions given below.
Eight friends P, Q, R, S, T, U, V and W are sitting around a circular table at equal distance from each other, but not
necessarily in the same order. All of them are wearing clothes of different brands – Van Heusen, Allen Solly, Peter
England, Flying Machine, Newport, Park Avenue, Mufti and FabIndia, but not necessarily in the same order.
P is third to the left of the person who is wearing clothes of FabIndia brand. One who is wearing Newport clothes is
seated between those two persons who are wearing Peter England and FabIndia clothes while the person who is
wearing Newport clothes sits opposite S. U sits second to the left of V and does not wear Flying Machine or Van
Heusen clothes. T is not the immediate neighbour of S, who is wearing Allen Solly. The one who is wearing Mufti
clothes sits to the immediate left of P. There are three persons between P and R, who is wearing Flying Machine
clothes. The person who is wearing Park Avenue clothes is second to the right of T. V sits to the immediate right of
S, who is not wearing Flying Machine or Park Avenue clothes. S is not immediate neighbour of T or P. The person
who is wearing Peter England is not immediate neighbour of V or T. An immediate neighbour of Q is wearing Van
Heusen clothes but that person is not W. W and T are sitting together.

1). Who among the following sits second to the right of W?


a) The person who is wearing Flying Machine clothes
b) U
c) P
d) T

www.bankingpdf.com
e) None of these

2). What is the position of U with respect to P?


a) Second to the right
b) Third to the left
c) Third to the right
d) Second to the left
e) Immediate left

3). What is the brand of clothes that T is wearing?


a) Mufti
b) Van Heusen
c) Flying Machine
d) FabIndia
e) Allen Solly

4). How many persons are seated between the person who is wearing Park Avenue and P? (If counting from P in
clockwise direction)
a) Four
b) Two

www.bankingpdf.com
c) Three
d) One
e) None of these

5). Which of the following is true regarding U?


a) U is wearing FabIndia clothes
b) U sits second to the right of V
c) U sits second to the right of T and is wearing Park Avenue clothes
d) U sits third to the left of W
e) None of these

6). Statements: D > J < C, V > U ≥ K = C


Conclusions:
I. U > D
II. V ≤ C
III. V > D
IV. D ≥ V

a) None follows
b) Only III follows
c) Only IV follows

www.bankingpdf.com
d) Only either III or IV follows
e) None of these

7). Statement: S = M ≥ N = Y < Z = R


Conclusions:
I. S ≥ Y
II. N < R
III. Z > N
IV. S ≥ N

a) Only I, II and III follows


b) Only II, III and IV follows
c) Only I, III and IV follows
d) All I, II, III and IV follows
e) None of these

8). Statement: P > M ≥ R = K ≤ H < J


Conclusions:
I. J > R
II. P ≥ H
III. P > K
IV. P ≥ K

www.bankingpdf.com
a) Only I and III follows
b) Only I and IV follows
c) Only II and III follows
d) Only I, II and IV follows
e) None of these

9). Statements: Q ≥ R = D, D ≥ B > M


Conclusions:
I. R ≤ B
II. Q > M
III. R ≥ B
IV. Q ≥ B

a) Only I and III follows


b) Only I and IV follows
c) Only II and III follows
d) Only II, III and IV follows
e) None of these

10). Statement: E > M = G ≤ O ≤ L = J


Conclusions:
I. J > M

www.bankingpdf.com
II. M = J
III. O > E
IV. J ≥ M

a) None follows
b) Only I follows
c) Only II follows
d) Only III follows
e) Only IV follows

Answers:
1)a 2)b 3)d 4)b 5)c 6)a 7)d 8)a 9)d 10)e

Directions (Q. 1-5):

www.bankingpdf.com
6). D > J < C, V > U ≥ K = C
U>D
V≤C
V>D
D≥V
None follows

7). S = M ≥ N = Y < Z = R
S≥Y
N<R

www.bankingpdf.com
Z>N
S≥N
All follows

8). P > M ≥ R = K ≤ H < J


J > R follows
P ≥ H not follows
P > K follows
P ≥ K not follows

9). Q ≥ R = D, D ≥ B > M
R > B not follows
Q > M follows
R ≥ B follows
Q ≥ B follows

10). E > M = G ≤ O ≤ L = J
J > M not follows
M = J not follows

www.bankingpdf.com
O > E not follows
J ≥ M follows

870-880 Questions :

Directions (Q. 1-5): Study the following information carefully to answer the given questions:

A man has six sons A, B, C, D, E and F, who stay in four metro cities. They work in different companies, viz Infosys,
Twitter, Tata, Google, Walmart and Facebook but not necessarily in the same order. D stays in Mumbai and works
with Google. Those who stay in Kolkata work with Twitter and Walmart. One of the two Mumbaiars works with
Facebook. The one who works with Infosys lives in Chennai. F does not live in Chennai. A works with Tata. B works
with Twitter. E works with Walmart.

1). Who among the following works with Infosys?


a) A
b) C
c) D
d) E
e) None of these

www.bankingpdf.com
2). Which of the following pairs lives in Kolkata?
a) DE
b) FD
c) BE
d) CA
e) None of these

3). F works with which of the following companies?


a) Walmart
b) Facebook
c) Tata
d) Can’t be determined
e) None of these

4). In which of the following metros does C live?


a) Delhi
b) Chennai
c) Kolkata
d) Can’t be determined
e) None of these

www.bankingpdf.com
5). Which of the following combinations is true?
a) B – Mumbai – Walmart
b) F – Kolkata – Walmart
c) E – Delhi – Facebook
d) C – Chennai – Infosys
e) None of these

Directions (Q. 6-10): Study the following information carefully to answer the given questions:
Seven participants F, B, G, A, D, C and E from seven different states, viz Rajasthan, Karnataka, Tripura, Assam,
Punjab, Chhattisgarh and Gujarat, but not necessarily in the same order, participate in a dance competition. All of
them perform seven different folk dances, viz Bagurumba, Sammi, Panthi, Garba, Hojagiri, Kalbelia and
Yakshagana, but not necessarily in the same order.
The one who performs Panthi does not belong to Punjab. B is from Tripura and does not perform Garba. The one
who is from Assam performs Bagurumba. The one who is from Karnataka performs neither Garba nor Hojagiri. F is
from Gujarat. D is from Rajasthan and performs Kalbelia dance. G performs Yakshagana but he is neither from
Chhattisgarh nor from Punjab. A performs Sammi and E performs Panthi.

6). Who performs Garba?


a) C
b) E

www.bankingpdf.com
c) D
d) F
e) None of these

7). Which of the following combinations is correct?


a) Rajasthan – Sammi
b) Punjab – Kalbelia
c) Chhattisgarh– Panthi
d) Chhattisgarh– Kalbelia
e) None of these

8). B performs which of the following folk dances?


a) Panthi
b) Bagurumba
c) Hojagiri
d) Sammi
e) None of these

9). Who is from Karnataka?


a) C
b) G
c) E

www.bankingpdf.com
d) Cannot be determined
e) None of these

10). C is from which state?


a) Rajasthan
b) Karnataka
c) Chhattisgarh
d) Assam
e) None of these

Answers:

1)b 2)c 3)b 4)b 5)d 6)d 7)c 8)c 9)b 10)d

Explanations:
Directions (Q. 1-5):

www.bankingpdf.com
Directions (Q. 6-10):

www.bankingpdf.com
880-890 Questions :

Directions (Q. 1-5): Study the following information and answer the questions that follow.
Seven institutes ABC, DEF, GHI, JKL, MNO, PQR and STU provide coaching for seven different competitive exams,
viz Engineering, NET, CAT, SSC, Banking, Medical and TET, but not necessarily in the same order. There is one day
weekly, off in each institute from Monday to Sunday but not necessarily in the same order. No two institutes have the
same weekly off day.
Institute GHI provides coaching for NET and is not closed either on Friday or on Wednesday.
Institute JKL provides coaching for Engineering, and Thursday is its weekly off day.
Institute MNO and PQR do not provide coaching for Banking and neither of these has Wednesday as weekly off day.
Institute DEF provides coaching for SSC and remains closed on Sunday.

www.bankingpdf.com
The one which provides coaching for Medical has Tuesday as weekly off day.
Institute STU provides coaching for TET and remains closed on Monday.
Institute MNO does not provide coaching for CAT.

1). Institute GHI remains closed on which of the following days?


a) Sunday
b) Monday
c) Tuesday
d) Saturday
e) None of these

2). Which of the following institutes provides coaching for Banking?


a) ABC
b) MNO
c) PQR
d) MNO or PQR
e) None of these

3). Which of the following combinations is correct?


a) Wednesday – DEF – SSC
b) Wednesday – DEF – Banking

www.bankingpdf.com
c) Friday – PQR – CAT
d) Thursday – JKL – Medical
e) None of these

4). Which institute provides coaching for Medical?


a) ABC
b) PQR
c) MNO
d) JKL
e) None of these

5). On which of the following days does Institute PQR remain closed?
a) Tuesday
b) Wednesday
c) Sunday
d) Cannot be determined
e) None of these

Directions (Q. 6-10): The following questions are based on the five three-digit numbers given below:

www.bankingpdf.com
261 489 643 586 729

6). If 2 is added to the first digit of each of the numbers, how many numbers thus formed will be divisible by three?
a) None
b) One
c) Two
d) Three
e) All

7). If the third digit of the second smallest number is divided by the third digit of the largest number then what will be
the result?
a) 1
b) 0
c) 3
d) 2
e) None of these

8). If all the digits in each of the numbers are arranged in descending order within the number, which of the following
will be the largest number?
a) 261
b) 489

www.bankingpdf.com
c) 643
d) 729
e) 586

9). If all the numbers are arranged in ascending order from left to right, which of the following will be the sum of all
three digits of the number which is second from the right of the new arrangement?
a) 20
b) 21
c) 13
d) 15
e) 9

10). If the first and the third digit of all the numbers are interchanged, then which of the following will be the second
largest number?
a) 729
b) 643
c) 261
d) 489
e) 586

Answers:

www.bankingpdf.com
1). d) 2). a) 3). c) 4). c) 5). e) 6). c) 7). a) 8). b) 9). c) 10). a)

Directions (Q. 1-5):


Institute Competitive Examination Weekly Off

ABC Banking Wednesday

DEF SSC Sunday

GHI NET Saturday

JKL Engineering Thursday

MNO Medical Tuesday

PQR CAT Friday

STU TET Monday

Directions (6-10):
261 489 643 586 729

www.bankingpdf.com
6). After adding 2 to the first digit in each of the numbers. Thus 843 and 786.
Answer: c)
7). The third digit of the second smallest number is 9. The third digit of the largest number is 9. Then, required
answer = 9/9 = 1
Answer: a)
8). After rearrangement, 621 984 643 865 972
Answer: b)
9). After rearrangement in ascending order of the numbers from left to right.
261, 489, 586, 643, 729
Now, sum of the digits of the second number from the right = 6 + 4 + 3 = 13
Answer: c)
10). After rearrangement,
162, 984, 346, 685, 927
Thus, second highest number is 927, which comes from 729.
Answer: a)

www.bankingpdf.com
890-900 Questions :

Directions (Q. 1-5): Study the following arrangement carefully and answer the questions given below:

K7%5AB1$GD94UEAC6H@8#IJM

1). How many such consonants are there in the above arrangement each of which is immediately preceded by a
symbol and immediately followed by a letter?
a) None
b) One
c) Two
d) Three
e) None of these

2). How many such numbers are there in the above arrangement each of which is immediately followed by a vowel?
a) One
b) Two
c) Three
d) Four
e) None of these

3). Which of the following is fifth to the right of the nineteenth from the left end of the above arrangement?
a) #
b) K

www.bankingpdf.com
c) M
d) 5
e) None of these

4). If all the symbols are dropped from the above arrangement, which of the following will be the tenth from the right
end?
a) U
b) C
c) E
d) 9
e) None of these

5). How many such symbols are there in the above arrangement each of which is immediately preceded by a letter
and immediately followed by a number?
a) None
b) Two
c) Three
d) One
e) None of these

Directions (Q. 6-10): Study the following information to answer the given questions.

www.bankingpdf.com
Six friends—Rangana, Rangitha, Ranjana, Ranjika, Ranjini and Ranjita—are sitting around a circular table facing
towards the centre in a hotel. They ordered different items like Appam, Dosa, Idli, Chappathi, Pongal and Juice for
their breakfast, not necessarily in the same order. Each of them wears a dress of a different brand, viz Park Avenue,
Peter England, Parx, Arrow, Blackberry and ATB, not necessarily according to the order given.
(i) The persons who ordered Appam, Idli, and Juice are wearing neither Park Avenue nor Peter England dress.
(ii) The persons in Parx and Blackberry dress have ordered neither Appam nor Idli.
(iii) Rangana is not wearing a Park Avenue dress and is not on the immediate left of the person who has ordered
Chappathi.
(iv) The only person who is sitting between Ranjini and Ranjita likes Dosa. The person who is on the left side of the
person in Park Avenue dress does not eat Pongal.
(v) Ranjika ordered Chappathi and he is wearing Parx dress. He is facing the person who has ordered Dosa.
(vi) The one who has ordered Appam is sitting opposite the person in ATB dress while the person in Parx dress is on
the left side of the person who ordered Juice.
(vii) The one who ordered Pongal is on the immediate right of the person in Park Avenue dress but on the immediate
left of the person who ordered for Idli.
(viii) Ranjana has not ordered Idli while Ranjita has not ordered Appam.

6). Who among the following is wearing Park Avenue dress?


a) Rangana
b) Rangitha
c) Ranjana

www.bankingpdf.com
d) Ranjini
e) None of these

7). The only person who is sitting between Ranjini and Ranjika is wearing the dress of which of the following brands?
a) Arrow
b) ATB
c) Peter England
d) Blackberry
e) None of these

8). Who among the following has ordered Juice?


a) Rangitha
b) Ranjika
c) Ranjana
d) Ranjini
e) None of these

9). Which of the following is correctly matched?


a) Rangana—Blackberry—Juice
b) Rangitha—Arrow—Idli
c) Ranjini—Arrow—Idli
d) Ranjita—Peter England—Juice

www.bankingpdf.com
e) None of these

10). The person who ordered Pongal is wearing the dress of which of the following brands?
a) Arrow
b) Blackberry
c) ATB
d) Peter England
e) None of these

Answers:
1). b) 2). b) 3). b) 4). a) 5). d) 6). c) 7). d) 8). e) 9). a) 10). d)

Explanation:
1). $GD
Answer: b)
2). 5A, 4U
Answer: b)
3). Fifth to the right of nineteenth from the left = (19 + 5=) 24th from the left, ie K.
Answer: b)
4). After rearrangement,
K 7 5 A B 1 G D 9 4 U E A C 6 H 8 I J M.

www.bankingpdf.com
Hence tenth from the right end is U.
Answer: a)
5). H @ 8
Answer: d)

Directions (Q. 6-10):


Ranjika orders Chappathi and wears Parx dress. The person opposite Ranjika orders Dosa.
Now, the persons wearing Park Avenue and Peter England do not order Appam, Idli, Juice or Chappathi. So, they
order Dosa or Pongal. But one who orders Pongal is on the immediate right of the person wearing Park Avenue
dress. So, the person wearing Park Avenue dress orders Dosa while the person who orders Pongal wears Peter
England dress and is sitting on the right of the person in Park Avenue dress.
Now, the person who orders Pongal is on the immediate left of the person who orders Idli. Clearly, using (vi), the
person on the left of the person in Park Avenue dress orders Appam. But he is sitting opposite the person wearing
ATB dress. So the person who orders Idli wears ATB dress. Now, the person who orders Appam must be wearing
either Blackberry or Arrow. But the person in Blackberry dress does not order Appam. So, he wears Arrow. The
person who orders Juice wears Blackberry dress. Ranjita orders Pongal and wears Peter England dress. Rangitha
orders Idli and wears ATB dress.

www.bankingpdf.com
www.bankingpdf.com
900-910 Questions :

1). How many meaningful English words can be made with the letter IFEL using each letter only once in each word?
a) None
b) One
c) Two
d) Three
e) More than three

2). Each consonant in the word TIRADES is replaced by the previous letter in the English alphabet and each vowel is
replaced by the next letter in the English alphabet and the new letters are rearranged alphabetically. Which of the
following will be the 4th from the right end?
a) F
b) J
c) Q
d) C
e) None of these

3). Pointing to a girl, Mahesh said, “She is the only daughter of my grandfather’s only child”. How is the girl related to
Mahesh?
a) Daughter
b) Niece

www.bankingpdf.com
c) Sister
d) Data inadequate
e) None of these

4). In a row of 25 children facing South R is 16th from the right end and B is 18th from the left end. How many
children are there between R and B?
a) 2
b) 3
c) 4
d) Data inadequate
e) None of these

Directions (Q. 5-10): Study the following information carefully and answer the given questions.
A word and number arrangement machine when given an input line of words and numbers rearranges them following
a particular rule in each step. The following is an illustration of input and rearrangement.
Input: new 11 bold 22 carve hundred 32 29 45 houses it 38
Step I: 11 22 new bold carve hundred 32 29 45 houses it 38
Step II: it new 11 22 bold carve hundred 32 29 45 houses 38
Step III: 29 32 it new 11 22 bold carve hundred 45 houses 38
Step IV: bold carve 29 32 it new 11 22 hundred 45 houses 38
Step V: 38 45 bold carve 29 32 it new 11 22 hundred houses

www.bankingpdf.com
Step VI: houses hundred 38 45 bold carve 29 32 it new 11 22
Step VI is the last step of the above input, as the desired arrangement is obtained.
As per the rules followed above find the appropriate step for the given input
Input: ice money 21 13 good 18 12 qualify 35 eligible 41 browse candidates 10

5). Which of the following represents the position of ‘ice’ in Step VI?
a) Third from the left
b) Fifth from the right
c) Sixth from the right
d) Fourth from the left
e) None of these

6). Which step will be the last but one?


a) IX
b) VI
c) V
d) VII
e) None of these

7). Which word/number would be at the 5th position from the right in Step V?
a) ice

www.bankingpdf.com
b) qualify
c) 10
d) 12
e) money

8). How many steps will be required to complete the arrangement?


a) VI
b) VII
c) VIII
d) X
e) IX

9). How many elements (words or numbers) are there between ‘21’ and ‘12’ in Step VII?
a) Eight
b) Five
c) Three
d) Six
e) None of these

10). Which step number is the following output? ‘money browse 13 18 ice good 10 12 21 qualify 35 eligible 41
candidates’

www.bankingpdf.com
a) III
b) VI
c) IV
d) V
e) None of these

Answers:
1)b 2)b 3)c 4)e 5)c 6)d 7)d 8)c 9)a 10)c

Solution:
1). b)
2). b) T I R A D E S
SJQBCFR
Alphabetically rearranged = B C F J Q R S
3). c)
4). e) 7

(5-10):
The machine arranges words and numbers in the following manner:
Step I. The first two numbers are arranged in ascending order from the left.

www.bankingpdf.com
Step II. The first two words are arranged according to the number of letters present in the word.
This process follows in each alternate step until all the numbers and words are arranged.

Input: ice money 21 13 good 18 12 qualify 35 eligible 41 browse candidates 10


Step I: 10 12 ice money 21 13 good 18 qualify 35 eligible 41 browse candidates
Step II: ice good 10 12 money 21 13 18 qualify 35 eligible 41 browse candidates
Step III: 13 18 ice good 10 12 money 21 qualify 35 eligible 41 browse candidates
Step IV: money browse 13 18 ice good 10 12 21 qualify 35 eligible 41 candidates
Step V: 21 35 money browse 13 18 ice good 10 12 qualify eligible 41 candidates
Step VI: qualify eligible 21 35 money browse 13 18 ice good 10 12 41 candidates
Step VII: 41 qualify eligible 21 35 money browse 13 18 ice good 10 12 candidates
Step VIII: candidates 41 qualify eligible 21 35 money browse 13 18 ice good 10 12

910-920 Questions :

Directions (Q. 1-5): In each question below are given three or four statements followed by four conclusions numbered
I, II, III and IV. You have to take the given statements to be true even if they seem to be at variance with commonly
known facts. Read all the conclusions and then decide which of the given conclusions logically follows from the given
statements, disregarding commonly known facts.

1). Statements: Some pulses are fruits

www.bankingpdf.com
Some fruits are sprouts
All sprouts are nuts
No seed is nut
Conclusions:
(i) Some nuts are pulses
(ii) Some nuts are fruits
(iii) No seed is sprout

a) a) Only i and ii follows


b) b) Only ii and iii follows
c) c) Only i , ii and iii follows
d) d) Only ii, iii and iv follows
e) None of these

2). Statements: All chairs are books


Some books are sticks
All sticks are branches
Conclusions:
(i) Some branches are chairs
(ii) Some branches are books
(iii) Some sticks are chairs

www.bankingpdf.com
a) Only i follows
a) b) Only ii follows
b) c) Only iii follows
c) d) Only either i or iii follows
d) e) Only either i or iii and ii follows

3). Statements: All bulbs are plugs


All plugs are tables
All tables are switches
Conclusions:
(i) Some switches are bulbs
(ii) Some tables are bulbs
(iii) All plugs are switches

a) None follows
b) All follow
a) c) Only ii and iii follow
b) d) Only iii follow
c) e) None of these

4). Statements: All knives are oils


No oils is sword

www.bankingpdf.com
Some swords are tails
Conclusions:
(i) Some tails are oils
(ii) Some swords are knives
(iii) No tail is oils

a) Only i follows
a) b) Only ii follows
b) c) Only iii follows
c) d) Only either i or iii follows
d) e) Only either i or iii and ii follows

5). Statements: Some fruits are parts


All parts are jungles
All jungles are greens
Conclusions:
(i) All fruits are jungles
(ii) Some greens are fruits
(iii) Some jungles are fruits

a) None follows
b) Only i follows

www.bankingpdf.com
a) c) Only ii follows
b) d) Only iii follows
c) e) Only ii and iii follows.

Directions (Q. 6-10): In the following questions, the symbols !, @, ©, % and * are used with the following meaning as
illustrated below:

‘P © Q’ means ‘P is not smaller than Q’.


‘P % Q’ means ‘P is neither smaller than nor equal to Q’.
‘P * Q’ means ‘P is neither greater than nor equal to Q’.
‘P ! Q’ means ‘P is not greater than Q’.
‘P @ Q’ means ‘P is neither greater than nor smaller than Q’.

6). Statements: D ! T, T @ R, R © M, M % K
Conclusions:
I. R@D
II. R%D
III. K*T
IV. M!T

a) Only either I or II follows


b) Only III and IV follows

www.bankingpdf.com
c) Only either I or II and III follows
d) Only either I or II and IV follows
e) Only either I or II and III and IV follows

7). Statements: J @ F, F ! N, N % H, H © G
Conclusions:
I. G*N
II. N©J
III. F*J
IV. J!G

a) Only I and II follows


b) Only I, II and III follows
c) Only II, III and IV follows
d) All I, II, III and IV follows
e) None of these

8). Statements: R * K, K % D, D @ V, V ! M
Conclusions:
I. R*D
II. V*R
III. D@M

www.bankingpdf.com
IV. M%D

a) None follows
b) Only III follows
c) Only IV follows
d) Only either III or IV follows
e) None of these

9). Statements: B © T, T * R, R % F, F @ K
Conclusions:
I. B%R
II. F*T
III. R%K
IV. K&T

a) None follows
b) Only I follows
c) Only II follows
d) Only III follows
e) Only IV follows

10). Statements: F % N, N © W, W ! Y, Y * T
Conclusions:

www.bankingpdf.com
I. F%W
II. T%N
III. N%Y
IV. T%W

a) Only I and III follows


b) Only I and IV follows
c) Only II and III follows
d) Only I, II and IV follows
e) None of these

Answers:
1)b 2)b 3)b 4)d 5)e 6)e 7)a 8)d 9)d 10)b

Explanation:

1).

www.bankingpdf.com
Answer: b)
2).

Answer: b)
3).

www.bankingpdf.com
Answer: b)
4).

Answer: d)
5).

Answer: e)

6). D ≤ T, T = R, R ≥ M, M > K
R=D
R>D

www.bankingpdf.com
K<T
M≤T
Only either I or II and III and IV follows
Answer: e)

7). J = F, F ≤ N, N > H, H ≥ G
G < N follows
N ≥ J follows
F < J not follows
J ≤ G not follows
Answer: a)

8). R < K, K > D, D = V, V ≤ M


R<D
V<R
D=M
M>D
Only either III or IV follows
Answer: d)

www.bankingpdf.com
9). B ≥ T, T < R, R > F, F = K
B > R not follows
F < T not follows
R > K follows
K = T not follows
Answer: d)

10). F > N, N ≥ W, W ≤ Y, Y < T


F > W follows
T > N not follows
N > Y not follows
T > W follows
Answer: b)

www.bankingpdf.com
920-930 Questions :

Directions (Q. 1-5): Study the following information and answer the questions that follow.
Jaipal, Jairaj, Jairam, Jalal, Jalil, Jashan and Jashun organise seminars in seven different schools, viz A, B, C, D, E,
F, G, on different days of the week from Monday to Sunday, but not necessarily in the same order.
Jairaj organises seminar in school D on Wednesday. Jalil does not organise seminar for school G or E but organises
on the next day of Jalal’s seminar, who organises seminar for school B. Jashun organises seminar for school C on
Friday. Jashan organises seminar on Monday, but not for school E or A. Jairam organises seminar for school G but
not on Tuesday.
1). On which of the following days does Jalil organise the seminar?
a) Tuesday
b) Sunday
c) Saturday
d) Cannot be determined
e) None of these

2). Jairam organises seminar on which of the following days?


a) Saturday
b) Sunday
c) Tuesday
d) Thursday

www.bankingpdf.com
e) None of these

3). Which of the following combination is true?


a) Jalal – Tuesday - E
b) Jalil – Wednesday - C
c) Jashan – Monday - F
d) Jashun – Thursday - A
e) None of these

4). Who among the following organises seminar in Saturday?


a) Jalil
b) Jaipal
c) Jairaj
d) Jalal
e) None of these

5). Who among the following organises seminar in school E and on which day?
a) Jalal, Saturday
b) Jaipal, Tuesday
c) Jashan, Monday
d) Jairam, Thursday
e) None of these

www.bankingpdf.com
6). Pointing to a boy, a man said, “I have no brother but that boy’s father is my father’s son.” How is the boy related to
that man?
a) Father
b) Grand son
c) Nephew
d) Son
e) None of these

7). How many such digits are there in the number 62143879 each of which is as far away from the beginning of the
number as when the digits are rearranged in ascending order?
a) None
b) One
c) Two
d) Three
e) None of these

8). In a certain code language DENTICLE is written as TNEDELIC. How is CONFUSED written in that code
language?
a) ENCODEUS
b) FNOCDEUS
c) OCFNUS
d) USEDCONF

www.bankingpdf.com
e) None of these

9). How many meaningful English words can be formed with the letters ABER using each letter only once in each
word?
a) None
b) One
c) Two
d) Three
e) None of these

10). What should come next in the following letter series?


RQPONMLQPONMLPONML??
a) P, O
b) N, M
c) O, P
d) O, N
e) None of these

Answers:
1)b 2)d 3)c 4)d 5)b 6)d 7)d 8)b 9)c 10)d

www.bankingpdf.com
Explanation:
Directions (Q. 1-5)

Person Day School

Jaipal Tuesday E

Jairaj Wednesday D

Jairam Thursday G

www.bankingpdf.com
Jalal Saturday B

Jalil Sunday A

Jashan Monday F

Jashun Friday C

6). Boy’s father = Man’s father’s son = man


The man is the boy’s father.
The boy is the man’s son.
Answer: d)
7).
6 2 1 4 3 8 7 9

1 2 3 4 6 7 8 9

Answer: d)
8). DENTICLE – TNEDELIC, first four words in reverse format and last four words also in reverse format
Similarly,
CONFUSED – FNOCDEUS
Answer: b)

www.bankingpdf.com
9). The meaningful words that can be formed are BEAR and BARE.
Answer: c)
10). RQPONML/QPONML/PONML/ON
Answer: d)

930-940 Questions :

Directions (Q. 1-5): Study the following information carefully and answer the questions given below.
Eight friends A, B, C, D, E, F, G and H are sitting at equal distances around a circular table facing the centre.
The angle made at the centre of the circle by straight lines from A and D is 1800, from A and H and A and C are
1350 each. F is third to the left of C. G is third to the right of H, who is second to the right of B.
1). What angle is made between B and C?
a) 900
b) 1800
c) 450
d) 1350
e) None of these

2). Who among the following is second to the right of G?

www.bankingpdf.com
a) A
b) D
c) B
d) E
e) None of these

3). How many persons sit between H and E?


a) Once
b) Two
c) Three
d) Four
e) None of these

4). What angle is made between C and G?


a) 1800
b) 900
c) 1200
d) 450
e) None of these

www.bankingpdf.com
5). Which of the following statements is true?
a) The angle made between A and E is 900
b) Two persons sit between D and F
c) H is second to the left of C
d) All are true
e) None of these

6). South is related to North-West in the same way as West is related to which of the following?
a) North
b) North-West
c) West
d) South-West
e) None of these

7). If Deepa says, “Leema’s father Kumar is the only son of my father-in-law Mahesh”, then how is Teena, who is
sister of Leema, related to Mahesh?
a) Daughter
b) Sister
c) Grand daughter
d) Daughter-in-law

www.bankingpdf.com
e) None of these

8). How many such pairs of letters are there in the word BEAUTIFUL each of which has as many letters between
them in the word as in the English alphabet?
a) One
b) Two
c) Three
d) Four
e) More than four

9). If ‘WIN’ is written as ‘@#%’ and ‘GROW’ is written as ‘©+$@’ then how is WRONG written in that code language?
a) @+$%©
b) ©+$%@
c) @#$%©
d) @$%+©
e) None of these

10). If A $ B means ‘A is father of B’, A @ B means ‘A is mother of B’ and ‘A # B’ means ‘A is husband of B’ then
which of the following means ‘D is grandson of P’?
a) D # N @ L $ P
b) N @P $ D $ L
c) P $ L @D # M

www.bankingpdf.com
d) L # D @N @P
e) None of these

Answers:
1)b 2)a 3)c 4)d 5)c 6)e 7)c 8)d 9)a 10)c

Explanation:

Directions(Q. 1-5):

www.bankingpdf.com
940-950 Questions :

1). The finance minister was afraid of any credit downgrade by international rating agencies. Which of the following
inferences cannot be drawn from the above statement? (An inference is something which is not directly stated but
can be inferred from the given facts.)
a) This would tarnish the country’s image.
b) The rupee would go crashing.
c) There would be a rise in inflation.
d) The voters would give his party a raw deal.
e) All the above inferences can be drawn

2). The segment of cars using Automated Manual Transmission (AMT) technology has not taken off. Which of the
following can be a probable cause(s) of the above phenomenon?
(I) There is a price gap of at least a lakh rupees between automatic and manual cars.
(II) The fuel efficiency in existing automatic cars is 25–30 per cent lower than in manual ones.
(III) Their maintenance cost is exorbitant.
a) Only II
b) Only I
c) Only I and II

www.bankingpdf.com
d) Only II and III
e) All I, II and III

3). Sony India has decided to sell off the Vaio brand and laptop business. Which of the following can be a proper
course(s) of action?
(I) Sony India should put more resources in developing the mobile business, which is the future growth area.
(II) The employees in laptop business should be moved to mobile division.
(III) People should sell their Vaio laptops.
a) Only I
b) Only II
c) Only I and II
d) Only II and III
e) All I, II and III

4). An exceptional monsoon will raise food grain production to a record 263.2 million this year.
Which of the following substantially weakens the argument given in the above statement?
a) Farm sector growth would be 4.6 per cent compared to 1.9 per cent last year.
b) Most of the land in India has proper irrigation facilities.
c) A rise in the production of pulses, whose global supplies are limited, will help tame prices.
d) India mostly produces cash crops.

www.bankingpdf.com
e) None of these

5). The rise of global CEOs who spent their formative years in India is acknowledgement that the country is doing at
least some things right. Which of the following assumptions is implicit in the above statement? (An assumption is
something supposed or taken for granted.)
a) India has produced a large number of CEOs.
b) India is not known for right policies.
c) Some of the qualities these leaders possess — humility, modesty and a strong work ethic — were acquired well
before they left the territorial frontiers of India.
d) The stability of family upbringing is a major advantage Indians have.
e) None of these

Directions (Q. 6-10): Study the following information carefully and answer the given questions.
Badrinath, Gaurinath, Kashinath, Raghunath, Loknath, Sainath, Ramnath and Taraknath are the members of a
committee sitting around a circular table but not facing the centre. Each member has a different zodiac sign, viz
Aquarius, Virgo, Taurus, Cancer, Sagittarius, Gemini, Pisces and Capricorn, but not necessarily in the same order.
Ramnath is third to the right of Kashinath. The one whose sunsign is Aquarius is second to the left of the one whose
sunsign is Taurus. Taraknath’s sunsign is Taurus and is sitting exactly between Kashinath and Badrinath. The one
whose sunsign is Pisces sits second to the right of Gaurinath. The one whose sunsign is Sagittarius is second to the
right of the person whose sunsign is Gemini. Kashinath sits third to the left of the person whose sunsign is Virgo.

www.bankingpdf.com
Neither Sainath nor Badrinath is the immediate neighbour of Gaurinath. Sainath is fourth to the left of Badrinath.
Gaurinath’s sunsign is neither Cancer nor Sagittarius. The person whose sunsign is Aquarius is sitting second to the
right of the person whose sunsign is Cancer. Raghunath’s sunsign is Aquarius and is not an immediate neighbour of
Gaurinath.

6). Which of the following is Gaurinath’s sunsign?


a) Pisces
b) Capricorn
c) Gemini
d) Cannot be determined
e) None of these

7). Who sits third to the right of Badrinath?


a) Kashinath
b) Taraknath
c) Raghunath
d) Sainath
e) None of these

8). What is Taraknath’s position with respect to Sainath?


a) Third to the left

www.bankingpdf.com
b) Fourth to the left
c) Second to the right
d) Third to the right
e) Second to the left

9). How many persons are there between Kashinath and Gaurinath?
a) None
b) Two
c) Three
d) Four
e) None of these

10). What is the sunsign of Kashinath?


a) Gemini
b) Taurus
c) Aquarius
d) Cannot be determined
e) None of these

Answers:
1)e 2)e 3)c 4)e 5)b 6)b 7)e 8)d 9)c 10)a

www.bankingpdf.com
Solution:
1). Credit downgrade leads to all of these.
Answer: e)

2). All the three reasons go against the popularity of AMT cars.
Answer: e)

3). Alternatives have to be explored. Hence (I) and (II) follow. But III is a panic reaction.
Answer: c)

4). Choices a) and c) are obviously irrelevant. Choice d) too does not make sense even if it were true. All we are
concerned about is the food grain component; cash crops do not concern the statement irrespective of what their
contribution is. Choice b) may be a googly. If irrigation facilities are there, we are not dependent on rainfall! Well, that
may be true if only certain areas have deficit rainfall. But in general, you do need a good rainfall to have a good water
level and to keep your canals functioning.
Answer: e)

www.bankingpdf.com
5). This is implicit in the last part of the statement — “the country is doing at least some things right”.
Answer: b)

950-960 Questions :

Directions (Q. 1-5): Study the following information and answer the questions that follow.
Seven people, namely Daaruk, Daarun, Darpan, Darshan, Deepak, Deepan, and Deepesh have an anniversary but
not necessarily in the same order, in seven different months (of the same year), namely February, March, April, June,
September, October and November. Each of them also likes a different chocolate, namely Perk, Jelly, Kitkat,
Milkybar, Dairy milk, Snickers and Munch but not necessarily in the same order.
Deepan has an anniversary in the month which has more than 30 days. Only one person has an anniversary
between Deepan and the one who likes Perk. Both Deepesh and Darpan have an anniversary in one the months
after the one who likes Perk. Deepesh has an anniversary immediately before Darpan. The one who likes Kitkat has
an anniversary in the month which has less than 30 days. Only three people have an anniversary between the one
who likes Kitkat and the one who likes Munch. Only two people have an anniversary between Deepesh and the one
who likes Milkybar. Darshan has an anniversary immediately after the one who likes Milkybar. Only two people have
an anniversary between Darshan and Deepak. Daaruk has an anniversary immediately before the one who likes
Jelly. Darpan does not like Snickers.

www.bankingpdf.com
1). Which of the following represents the month in which Deepesh has an anniversary?
a) October
b) March
c) April
d) September
e) Cannot be determined

2). Which of the following does Darpan like?


a) Perk
b) Jelly
c) Milkybar
d) Dairy milk
e) Munch

3). As per the given arrangement, Kitkat is related to April and Milkybar is related to September following a certain
pattern. Which of the following is Munch related to following the same pattern?
a) February
b) June
c) November
d) October

www.bankingpdf.com
e) March

4). Which of the following represents the people who have an anniversary in April and November respectively?
a) Daarun, Daaruk
b) Daarun, Darpan
c) Deepak, Darpan
d) Deepak, Daaruk
e) Daarun, Deepesh

5). How many people have an anniversary between the months in which Deepak and Daaruk have an anniversary?
a) None
b) One
c) Two
d) Three
e) More than Three

Directions (6-10): In these questions, certain symbols have been used to indicate relationships between elements as
follows:
1) A @ B means P is not smaller than Q.

www.bankingpdf.com
2) A # B means P is not greater than Q.
3) A % B means P is neither greater than nor smaller than Q.
4) A + B means P is neither smaller than nor equal to Q.
5) A $ B means P is neither greater than nor equal to Q.

6). Statements: Y $ W , W @ O , O # H
Conclusions:
I. Y+O
II. H%W
III. H+Y

a) None follows
b) Only I follow
c) Only II follow
d) Only III follow
e) Only II & III follows

7). Statements: B + M , M @ Z , Z $ C
Conclusions:
I. C+M

www.bankingpdf.com
II. B+Z
III. C+B

a) Only II follows
b) Only I and III follows
c) Only I follow
d) Only III follow
e) None of these

8). Statements: N @ S , S % M , M + H
Conclusions:
I. M#N
II. N+H
III. M+N

a) Only II follows
b) Only I and II follows
c) Only III follows
d) Only II and III follows
e) None of these

www.bankingpdf.com
9). Statements: L @ U , U + A , A $ G
Conclusions:
I. G$L
II. L#G
III. L+A

a) None follows
b) Only I follow
c) Only II follow
d) Only III follow
e) Only II & III follows

10). Statements: J # W , W + A , A @ F
Conclusions:
I. F$W
II. F%W
III. J +F

a) None follows
b) Only I follow
c) Only II follow

www.bankingpdf.com
d) Only III follow
e) Only II & III follows

Answers:
1)a 2)d 3)c 4)b 5)d 6)a 7)a 8)b 9)d 10)b

Directions (Q. 1-5):


Person Chocolate Month

Deepak Kitkat February

Deepan Snickers March

Daarun Milkybar April

Darshan Perk June

Daaruk Munch September

Deepesh Jelly October

www.bankingpdf.com
Darpan Dairy milk November

6). Y < W , W ≥ O , O ≤ H
Y>O
H=W
H>Y
None follows
Answer: a)

7). B > M , M ≥ Z , Z < C


C > M not follows
B > Z follows
C > B not follows
Answer: a)

8). N ≥ S , S = M , M > H
M ≤ N follows
N > H follows
M > N not follows

www.bankingpdf.com
Answer: b)

9). L ≥ U , U > A , A < G


G < L not follows
L ≤ G not follows
L > A follows
Answer: d)

10). J ≤ W , W > A , A ≥ F
F < W follows
F = W not follows
J > F not follows
Answer: b)

960-970 Questions :

Directions (Q. 1-5): Study the following information carefully and answer the given questions.

www.bankingpdf.com
P, Q, R, S, I, J, K and L are eight friends sitting around a square table, two on each side. All of them are facing away
from the centre and each is opposite another. There are three female members and they are not seated next to one
another.
K sits between I and Q. L is a female member who sits second to the left of K. Q is not a female member but sits
opposite P, who is a female. S sits third to the left of J and is not a male member.

1). Who among the following sits on the immediate left of Q?


a) J
b) K
c) L
d) R
e) None of these

2). Which of the following statements is true about J and K?


a) Both are opposite each other.
b) Both are males.
c) J is a female but K is a male.
d) Both are females.
e) None of these

www.bankingpdf.com
3). Which of the following groups includes only females?
a) LPJ
b) PSR
c) KLQ
d) PSL
e) None of these

4). Who among the following is sitting between R and J?


a) P
b) S and I
c) S
d) P and I
e) None of these

5). Who among the following sits third to the right of Q?


a) P
b) S
c) J

www.bankingpdf.com
d) I
e) None of these

Directions (Q. 6-10): Study the following information carefully to answer the given questions.
A word and number arrangement machine when given an input line of words and numbers rearranges them following
a particular rule in each step. The following is an illustration of the input and its rearrangement.
Input: site 72 easy owl 28 11 pull 81 40 cut
Step I: easy site 72 owl 28 pull 81 40 cut 11
Step II: easy owl site 72 pull 81 40 cut 11 28
Step III: easy owl cut site 72 pull 81 11 28 40
Step IV: easy owl cut pull site 81 11 28 40 72
Step V: easy owl cut pull site 11 28 40 72 81
Step V is the last step of the above input. As per the rules followed in the above steps, find out in each of the
following questions, the appropriate step for the given input below and answer the questions based on it.
Input: curtail 53 vitiate 49 33 artifice 45 aptitude 23 ice 69 entourage bevy

6). How many steps will be required to complete the arrangement of the above input?
a) Four
b) Five
c) Six

www.bankingpdf.com
d) Seven
e) None of these

7). What will be the position of ‘69’ in Step IV?


a) 9th from the left
b) Fifth from the right
c) Extreme left
d) Extreme right
e) None of these

8). Which step would be the following output? ‘aptitude artifice entourage curtail 53 vitiate 49 ice 69 bevy 23 33 45’
a) IV
b) V
c) VI
d) III
e) VII

9). Which word/number would be at the 9th position from the right end in Step V?
a) curtail

www.bankingpdf.com
b) vitiate
c) bevy
d) 53
e) 49

10). Which of the following steps would be the last step but one?
a) V
b) VI
c) VII
d) IV
e) None of these

Answers:
1)c 2)b 3)d 4)a 5)b 6)c 7)e 8)d 9)c 10)a

Explanation:

Directions (Q. 1-5):

www.bankingpdf.com
Directions (Q. 6-10):
The machine rearranges one word and one number in each step. The words starting with a vowel are arranged first
in alphabetical order from the left. When this is done, the words starting with a consonant are arranged in
alphabetical order. The numbers are arranged in ascending order from the right end.
Input: curtail 53 vitiate 49 33 artifice 45 aptitude 23 ice 69 entourage bevy
Step I. aptitude curtail 53 vitiate 49 33 artifice 45 ice 69 entourage bevy 23
Step II. aptitude artifice curtail 53 vitiate 49 45 ice 69 entourage bevy 23 33
Step III. aptitude artifice entourage curtail 53 vitiate 49 ice 69 bevy 23 33 45
Step IV. aptitude artifice entourage ice curtail 53 vitiate 69 bevy 23 33 45 49
Step V. aptitude artifice entourage ice bevy curtail vitiate 69 23 33 45 49 53

www.bankingpdf.com
Step VI. aptitude artifice entourage ice bevy curtail vitiate 23 33 45 49 53 69

970-980 Questions :

Directions (Q. 1-3): Study the following information carefully and answer the given questions.
There are five colour bottles Red, Blue, Green, White and Yellow arranged on a table but not necessarily in the same
order. Blue bottle is to the north of White bottle. Green bottle is to the south of Blue bottle. Red bottle and Yellow
bottle are in the east and west respectively of Green bottle.

1). In which direction is White bottle with respect to Green bottle?


a) South
b) North
c) Either a) or b)
d) East
e) North-east

2). If Green bottle is exactly between Blue and White then in which direction is Red bottle with respect to White
bottle?
a) North
b) North-east

www.bankingpdf.com
c) South-east
d) South
e) None of these

3). In which direction is Yellow bottle with respect to Blue bottle?


a) South
b) South-east
c) West
d) South-west
e) None of these

4). If ‘P + Q’ means ‘P is father of Q’, ‘P × Q’ means ‘P is mother of Q’ and ‘P # Q’ means ‘P is husband of Q’ then
which of the following means ‘L is grandfather of H’?
a) H × T + L × N
b) L + H # P × T
c) R # H × P + L
d) L + T # R × H
e) None of these

www.bankingpdf.com
5). In a certain code language GARDEN is written as SZTODE. How is FRUGAL written in that code language?
a) GBSHBM
b) WSHMZH
c) UQGMZH
d) VQUMZH
e) None of these

Directions (Q. 6-10): Study the following information carefully and answer the given questions.
Indradutt, Indrajeet, Indrakanta, Indraneel, Indrasena, Indrasuta and Indratan are seven shopkeepers of different
shops, viz Electronic Appliances, Jewellery, Books, Sportswear, Toys, Vegetables and Medicines. They go to
supermarkets, viz. Food Bazaar, More, Reliance Fresh and Hearty Mart only on Sunday but not necessarily in the
same order. At least one shopkeeper goes to one supermarket, but no super market is visited by more than two
shopkeepers.
Indrasuta, who is a shopkeeper of Books, goes alone to Reliance Fresh.
The one who is a shopkeeper of Sportswear does not go to Food Bazaar. Also, he never goes either with Indrasena
or with Indratan.
Indraneel goes to Hearty Mart with the person who is the shopkeeper of Medicines.
Indrakanta goes to More. Indratan is not a shopkeeper of Medicines.
The one who is the shopkeeper of Sportswear goes to the market with the person who is the shopkeeper of
Electronic Appliances.

www.bankingpdf.com
The one who is the shopkeeper of Toys goes to Hearty Mart.
Indradutt is a shopkeeper of neither Jewellery nor Medicines.
The one who is the shopkeeper of Vegetables goes to Food Bazaar neither with Indrasena nor with Indraneel.
The person who is the shopkeeper of Jewellery goes to the supermarket with Indradutt.

6). Indradutt goes which of the following supermarkets?


a) Hearty Mart
b) More
c) Food Bazaar
d) Reliance Fresh
e) None of these

7). Which of the following statements is/are not true?


a) Indraneel is a shopkeeper of Medicines and goes to Hearty Mart.
b) Indrakanta is a shopkeeper of Sportswear and goes to Food Bazaar.
c) Indratan is a shopkeeper of Jewellery and goes to Food Bazaar
d) Both options a and b
e) All statements are false

www.bankingpdf.com
8). Who among the following is shopkeeper of Toys?
a) Indrajeet
b) Indraneel
c) Indrasuta
d) Indradutt
e) None of these

9). Which of the following statements is definitely incorrect?


a) Indrakanta is a shopkeeper of Sportswear.
b) Indrasena is a shopkeeper of Medicines.
c) Indratan is shopkeeper of Electronic Appliances.
d) All are incorrect
e) None of these

10). Food Bazaar is visited by which of the following person(s)?


a) Indratan and Indrasena
b) Only Indraneel
c) Indradutt and Indrajeet
d) Only Indrasuta
e) None of these

www.bankingpdf.com
Answers:
1)c 2)b 3)d 4)d 5)d 6)a 7)d 8)b 9)c 10)e

Q (1-3)

4).

www.bankingpdf.com
5). GARDEN = SZTODE
FRUGAL = VQUMZH

Directions (Q. 6-10):


Shopkeeper Super Market Shop

Indradutt Food Bazaar Vegetables

Indrajeet More Electronic Appliances/Sportswear

Indrakanta More Sportswear/Electronic Appliances

Indraneel Hearty Mart Toys

Indrasena Hearty Mart Medicines

www.bankingpdf.com
Indrasuta Reliance Fresh Books

Indratan Food Bazaar Jewellery

980-990 Questions :

Directions (Q. 1-5): Study the following information carefully and answer the questions given below:

A,B,C,D,E,F,G and H are eight friends sitting around a circle in one arrangement facing outwards, and in a straight line
in another arrangement where they face north. One of the immediate neighbours of H in the straight line sits opposite
H in the circle. E sits third to the right of B in the circle, while fourth to his left in the straight line. F and C are the
immediate neighbours of B in both the arrangements, but C is not at the extreme ends of the line. The one who sits on
the extreme left end sits second to the right of E in the circle. H is not on the immediate left of F in both the
arrangements. G sits on the immediate left of H in the circle, but they are not immediate neighbours of each other in
the straight line. D sits third to the right of F in the straight line. The one who sits on the immediate left of B in the
straight line is sitting on the immediate right of B in the circle.

1). Who is sitting opposite to F in a circle?


a) H
b) D
c) E
d) G
e) None of these
2). Which of the following pair is sitting at the extreme ends of a straight line?
a) H,D
b) D,A
c) G,D

www.bankingpdf.com
d) G,C
e) None of these
3). Immediate neigbours of A in a circle are
a) B,E
b) E,C
c) E,B
d) E,F
e) None of these
4). Who is sitting forth to the left of C in a circle?
a) A
b) B
c) C
d) D
e) E
5). Which of the following statement is true about A in a straight line arrangement?
a) A is sitting in the middle of line
b) C is sitting third to the left of A
c) None of the following is true
d) G and F are immediate neighbours of A
e) C is sitting third to the right of A

Directions (6-10): Study the following information and answer the following questions:
Seven friends Ravi, Raju, Raja, Rajan, Rahul, Rajesh and Ranjan study in three different colleges A,B, C and four
different branches of engineering -Mechanical, Chemical, Electronics and Electrical not necessarily in the same order.
At least one of them and not more than three of them study in any one college and any one branch. Raju studies
mechanical in college B. Ravi and Rajan study in college A but in different branches. Rahul studies chemical and he
is in college C with Raja. Neither Rajesh nor Ranjan studies in college A. The only person who studies electronics is
in college C. Ranjan Studies electrical. Ravi does not study chemical. Rajan and Raju are in same branch and only
two people study chemical. Rajesh studies in college B.

6). Who studies electronics ?


a) Raju

www.bankingpdf.com
b) Rajan
c) Raja
d) Ravi
e) Rahul
7). In which branch Rajesh study ?
a) Mechanical
b) Chemical
c) Electrical
d) Electronics
e) Can't be determined
8). Which college has maximum number of students ?
a) A
b) B
c) Either A or B
d) C
e) Either B or C
9). Which of following pair are in same college ?
a) Ravi, Raju
b) Raja, Rajan
c) Rajesh, Raju
d) Rahul, Rajan
e) None of these
10). In which branch Ravi studies ?
a) Mechanical
b) Chemical
c) Electrical
d) Electronics
e) Can't be determined

Answers:

www.bankingpdf.com
1)d 2)c 3)d 4)e 5)e 6)c 7)b 8)e 9)c 10)c

Explanation:

Directions (Q. 1-5):

Directions (Q. 6-10):


Person Dept College

Ravi Electrical A

www.bankingpdf.com
Raju Mech B

Raja Electronics C

Rajan Mech A

Rahul Chemical C

Rajesh Chemical B

Ranjan Electrical C/B

www.bankingpdf.com
990-1000 Questions :

Directions (1 – 5) : Study the following information carefully and answer the questions given below:
Eight friends D, E, F, G, H, I, J and K are sitting around a circular table at equal distance between each other, but not
necessarily in the same order. All friends are facing the centre.
D sits either second to the left of G or second to the right of I and there is an angle between D and J, which is 180° at
the centre. K is third to the left of J. F is third to the right of E. There is an angle between I and G which is 135° at the
centre. F is not an immediate neighbor of either D or G. There is an angle between H and I which is 45°.

1). What is the position of H with respect to E?


a) Third to the left
b) Second to the left
c) Immediate right
d) Fourth to the right
e) None of these

2).How many persons are there between I and E?


a) Two
b) Three
c) Four

www.bankingpdf.com
d) Can’t be determined
e) None of these

3).If K and F interchange their positions, who will sit on the immediate right of F?
a) E
b) J
c) I
d) G
e) None of these

4).What is the measurement of the angle made between D and H?


a) 180°
b) 90°
c) 135°
d) 45°
e) None of these

5).In which of the following groups is the first person second to the right of the second person?
a) H,K
b) I,F
c) J,I
d) F,G

www.bankingpdf.com
e) None of these

Directions (Q. 6- 10): Study the following information carefully and answer the questions given below:
Eight girls Kajal, Komal, Kanak, Kiran, Kanika, Keshar, Kanchan and Kamala are sitting around a circular table facing
the centre. All of them have a different hobby, viz Watching TV, Singing, Reading, Writing, Making Toys, Dancing ,
Swimming and Cooking but not necessarily in the same order.
Keshar is third to the right of Kiran, who does not like Swimming or Cooking. The one who likes Dancing is
an immediate neighbor of Kanchan and Kanak. The one who likes Swimming is second to the left of Keshar, who
likes writing. The one who likes cooking is opposite Keshar. Kamala is not an immediate neighbor of either Kanak or
Keshar but is third to the right of Komal. The persons who like dancing and reading sit opposite each other. Kamala
likes Singing. The one who likes Swimming sits third to the left of the one who likes Reading. The one who likes
Watching TV and the one who likes Cooking are immediate neighbours. Kajal does not like Watching TV. Kanika is
second to the right of Kajal, who is an immediate neighbor of Keshar and Kamala.

6).Which of the following is the hobby of Komal?


a) Reading
b) Watching TV
c) Making Toys
d) Dancing
e) None of these

7).Who among the following likes Swimming?

www.bankingpdf.com
a) Kanak
b) Kanika
c) Kanchan
d) Can’t be determined
e) None of these

8).If Kanika is an immediate neighbor of Kanak then who among the following likes Cooking?
a) Kanak
b) Kanchan
c) Kajal
d) Can’t be determined
e) None of these

9).If Kanchan is third to the right of Kanika, then who among the following is on the immediate left of Komal?
a) Kiran
b) Kanchan
c) Kanak
d) Kamala
e) None of these

10).Which of the following statements is / are true?


a) Kanak sits opposite Keshar and likes Cooking

www.bankingpdf.com
b) Kanika likes reading and is on the immediate left of Kamala
c) Kajal is an immediate neighbor of Keshar and Kamala and likes reading.
d) None is true
e) None of these

Answers:
1)a 2)b 3)a 4)d 5)d 6)c 7)d 8)a 9)b 10)c

Explanation:

Questions (01 to 05):

www.bankingpdf.com
Questions (06 to 10):

www.bankingpdf.com

You might also like